Download as pdf or txt
Download as pdf or txt
You are on page 1of 278



Question bank
PAST-PAPERS
‫‪‬إ‪‬اء‪‬‬
‫‪ ‬آﯾﺔ ﻣﻦ ﻛﺘﺎب ﷲ ﺣﻔﻈﻨﺎھﺎ وﻟﻜﻞ ﻣﻌﻠﻮﻣﺔ ﻗﺮأﻧﺎھﺎ وﻧﺴﯿﻨﺎھﺎ ﻟﻌﻞ ﷲ ﯾﺤﻤﯿﻨﺎ ﻣﻦ اﻟﻨﺴﯿﺎن وﯾﺮدھﺎ إﻟﯿﻨﺎ‬
‫‪ ‬ﺷﺨﺺ دﻋﻮﻧﺎ ﻟﮫ ﻓﻲ ظﮭﺮاﻟﻐﯿﺐ ﻟﻌﻞ ﷲ أن ﯾﺴﺘﺠﯿﺐ دﻋﺎﺋﻨﺎ‬
‫‪ ‬ﻣﺮﯾﺾ ﻗﺪ آﻟﻤﮫ ﻣﺎ أﻟﻤﮫ ﻟﻌﻞ ﷲ ﯾﺸﻔﯿﮫ وﯾﻌﯿﺪ إﻟﯿﮫ ﺻﺤﺘﮫ وﻋﺎﻓﯿﺘﮫ‬
‫‪ ‬ﻟﺤﻈﺔ ﺗﺂﻟﻢ ﺑﮭﺎ أﺧﻲ وﺟﺪي وأﺣﺒﺘﻲ أﺛﻨﺎء ﻋﻼﺟﮭﻢ ﻟﻌﻞ ﷲ ﻻ ﯾﺮﯾﮭﻢ اﻵﻟﻢ وﻻ ﯾﻀﯿﻤﮭﻢ ﻣﺮة أﺧﺮى‬
‫‪ ‬ﻣﻮﻗﻒ اﺣﺘﺠﺖ ﺑﮫ ﻋﯿﻨﺎي ﻓﻠﻢ ﺗﻠﺒﯿﺎﻧﻲ ﻟﻌﻞ ﷲ ﻻ ﯾﻀﺮﻧﻲ ﺑﮭﻤﺎ ﻓﻲ ﺣﯿﺎﺗﻲ‬
‫‪ ‬ﺣﺎﻟﻢ ﺻﺎﺣﺐ أھﺪاف وﻣﺒﺎدئ ﻟﻌﻞ ﷲ ﯾﺤﻘﻖ ﻟﮫ ﻣﻘﺎﺻﺪه ﺣﺘﻰ ﯾﺮﺿﯿﮫ وﯾﺮﺿﻰ ﻋﻨﮫ‬
‫‪ ‬ﻣﺠﺘﮭﺪ ﻟﻌﻞ ﷲ ﯾﺠﻌﻞ ﻟﮫ ﻣﻦ اﻟﺘﻮﻓﯿﻖ أوﻓﺮ ﺣﻆ و ﻧﺼﯿﺐ وﯾﮭﻮن ﻟﮫ اﻷﺳﺒﺎب ﻛﻠﮭﺎ ﻣﻦ ﺣﯿﺚ ﻻ‬
‫ﯾﺪري ﻣﻦ ﻏﯿﺮ ﺿﺮاء ﻣﻀﺮة وﻻ ﻓﺘﻨﺔ ﻣﻀﻠﮫ‬
‫‪ ‬ﻣﻦ ﻓﻘﺪ ﺷﯿﺌﺎ ﻓﻲ اﻟﺤﯿﺎة ﻟﻌﻞ ﷲ ﯾﻌﯿﺪه إﻟﯿﮫ أو ﯾﻌﻮﺿﮫ ﺑﻤﺎ ﯾﺴﻌﺪه‬
‫‪ ‬ﺷﺨﺺ اﺑﺘﻌﺪﻧﺎ ﻋﻨﮫ ﻟﻌﻞ ﷲ ﯾﺠﻤﻌﮫ ﺑﻤﻦ ھﻢ أﻓﻀﻞ ﻣﻨﺎ‬
‫وأ‪‬ا وﻟﯿﺲ آﺧﺮا ﻷھﻠﻨﺎ ﻟﻌﻞ ﷲ ﯾﺠﻌﻠﮭﺎ ﻟﮭﻢ ﺻﺪﻗﺔ ﺟﺎرﯾﮫ ﺗﺮد إﻟﯿﮭﻢ ﻗﻠﯿﻼ ﻣﻦ ﻓﻀﻠﮭﻢ ﻋﻠﯿﻨﺎ وﺗﻜﻮن ﺳﺒﺒﺎ‬
‫ﯾﻠﻢ ﺷﻤﻠﻨﺎ ﺗﺤﺖ ﺳﻘﻒ واﺣﺪ ﺑﻌﺪ أن ھ ﱠﺠ َﺮﺗﻨﺎ اﻟﺤﺮوب واﻟﻤﺼﺎﺋﺐ‬

‫‪PT Part I 2019 index‬‬


‫‪Branch‬‬ ‫‪No. of Q‬‬
‫‪Cardiology‬‬ ‫)‪(386‬‬
‫‪Clinical haematology/oncology‬‬ ‫)‪(392‬‬
‫‪Clinical‬‬
‫)‪(513‬‬
‫‪pharmacology/therapeutics/toxicology‬‬
‫‪Dermatology‬‬ ‫)‪(138‬‬
‫‪Endocrinology‬‬ ‫)‪(414‬‬
‫‪Gastroenterology‬‬ ‫)‪(403‬‬
‫‪General Revision‬‬ ‫)‪(122‬‬
‫‪Infectious diseases/sexually‬‬
‫)‪(412‬‬
‫‪transmitted diseases‬‬
‫‪Nephrology‬‬ ‫)‪(335‬‬
‫‪Neurology‬‬ ‫)‪(413‬‬
‫‪Ophthalmology‬‬ ‫)‪(81‬‬
‫‪Psychiatry‬‬ ‫)‪(268‬‬
‫‪Respiratory Medicine‬‬ ‫)‪(457‬‬
‫‪Rheumatology‬‬ ‫)‪(380‬‬
Clinical Sciences
Cell/molecular and membrane
(72)
biology
Clinical anatomy (67)
Clinical biochemistry and metabolism (118)
Clinical physiology (92)
Genetics (107)
Immunology (118)
Statistics (158)

Past-Papers index
Passing mark range between 63-66%
Year No. of Q
MRCP 1 May 19 100
MRCP 1 Jan 19 100
MRCP 1 Sept 18 100
MRCP 1 May 18 100
MRCP 1 Jan 18 100
MRCP 1 Sept 17 100
MRCP 1 May 17 100
MRCP 1 Jan 17 100
MRCP 1 Sept 16 100
MRCP 1 May 16 100
MRCP 1 Jan 2016 100
MRCP 1 Sept 2015 100
MRCP 1 May 2015 100
MRCP 1 Jan 2015 100

A. H. Murad
‫ﻻ ﺗﻨﺴﻮﻧﺎ ﻣﻦ ﺻﺎﻟﺢ دﻋﺎﺋﻜﻢ‬
‫ﺗﻢ ﺑﺤﻤﺪ ﷲ وﺗﻮﻓﯿﻘﮫ وﻣﻨﮫ‬
‫ﺟﻌﻞ ﷲ ﻋﻤﻠﻨﺎ ﻣﺘﻘﺒﻼ ﺧﺎﻟﺼﺎ ﻟﻮﺟﮭﮫ اﻟﻜﺮﯾﻢ‬
0:00:43/03:00:00

You are reviewing the case of a 23-year-old man who suffered a sudden cardiac arrest at
work and died. He was in asystole by the time the ambulance had arrived on scene. He had a
family history of sudden cardiac death, with his father having died suddenly at the age of 38.
A post-mortem examination is performed, with a myocardial biopsy revealing fibrofatty
infiltration of the right ventricle. The right ventricle is dilated.
Which of the following is the most likely diagnosis?

A Arrhythmogenic right ventricular cardiomyopathy

B Brugada syndrome

C Hypertrophic obstructive cardiomyopathy (HOCM)

D Long QT syndrome

E Viral myocarditis

Explanation 

A Arrhythmogenic right ventricular cardiomyopathy

Young, previously healthy patients may suffer a cardiac arrest as the presenting feature of
arrhythmogenic right ventricular cardiomyopathy. The aetiology is unknown. Over time,
cardiac myocytes suffer increased apoptosis and are progressively replaced by fibrofatty
tissue. Progressive right ventricular failure ensues, with an increased risk of ventricular
arrhythmias over time. Implantable cardioverter–defibrillator (ICD) implantation is mandated
in high-risk patients. Medical management is with standard heart failure medications, such as
angiotensin-converting enzyme inhibitors and diuretics, although cardiac transplantation is
the ultimate treatment for many patients.

B Brugada syndrome

Brugada syndrome is a cause of fatal ventricular arrhythmia, although it occurs because of


sodium channel dysfunction, rather than increased death of cardiac myocytes. No medical
therapy has yet proven to be effective.

C Hypertrophic obstructive cardiomyopathy (HOCM)

HOCM is usually associated with septal and left ventricular hypertrophy, rather than the right
ventricular changes seen here. Beta-blockers may be used to reduce symptoms of angina,
although they do not significantly impact on ventricular arrhythmias, which are thought to be
related to the degree of septal hypertrophy. The risk of sudden cardiac death in these
patients can be calculated using the European Society of Cardiology HCM Risk-SCD
calculator. High-risk patients are typically offered an implantable cardiac defibrillator.

D Long QT syndrome

Long QT syndrome presents with sudden cardiac death due to ventricular arrhythmias,
typically without any changes in cardiac anatomy or QRS morphology on the
electrocardiogram. Beta-blockers are the first-line therapy for long QT syndrome, with ICD
implantation for patients with a history of collapse thought to be related to ventricular
arrhythmias.

E Viral myocarditis

The history here of sudden cardiac death without a significant prodromal illness does not fit
with a diagnosis of viral myocarditis.
51574

Rate this question:

Next Question

Previous Question Tag Question

Feedback End Review

Difficulty: Average

Peer Responses %

Q. Answered Flagged

Q1

Q2

Q3

Q4

Q5
Q. Answered Flagged

Q6

Q7

Q8

0:00:43/03:00:00

A 54-year-old man is reviewed in the clinic, following his admission for upper gastrointestinal
(GI) haemorrhage. He has been suffering from dyspepsia, night sweats and weight loss over
the past six months, although he is still able to hold down his job as a solicitor.
The endoscopy revealed a suspicious area in the gastric antrum which was biopsied. The
biopsy has revealed extensive lymphocytic infiltration, some of which appear atypical. A
computed tomography (CT) scan of the thorax and abdomen has revealed no evidence of
spread. Helicobacter pylori testing is positive.
Physical examination in the clinic and routine bloods are unremarkable.
Which of the following is the most appropriate initial intervention?

A Gastrectomy

B H. pylori eradication

C Imatinib

D Radiotherapy

E Rituximab

Explanation 

B H. pylori eradication

This patient has a gastric MALToma, which is likely to be related to chronic inflammation as a
result of H. pylori infection. In early-stage disease, H. pylori eradication leads to remission in
65–70% of patients and is therefore the first-line option here. In patients who do not achieve
complete remission, disease may remain localised for many years. Typically, biopsies are
performed every three months initially to establish response to eradication therapy.

A Gastrectomy

Gastrectomy is reserved for those who have failed treatment with non-surgical modalities,
including both radiotherapy, chemotherapy and targeted immune interventions.

C Imatinib

Imatinib is a small-molecule kinase inhibitor, which was originally developed for the treatment
of chronic myeloid leukaemia. It is used in the treatment of gastric MALToma, usually in
combination with rituximab. It is incorrect here, as it is not a first-line therapy for MALToma.
D Radiotherapy

Radiotherapy vies with combination imatinib and rituximab as the next step in patients who
fail to respond to H. pylori eradication. It is particularly useful where all the affected mucosa
can be targeted in one field of radiation.

E Rituximab

Rituximab is an anti-CD20 monoclonal antibody which targets B lymphocytes. It is the


immune intervention of choice, in combination usually with imatinib, for patients who fail to
respond to H. pylori eradication.
51586

Rate this question:

Next Question

Previous Question Tag Question

Feedback End Review

Difficulty: Average

Peer Responses %

Q. Answered Flagged

Q1

Q2

Q3

Q4

Q5

Q6

Q7
Q. Answered Flagged

Q8
0:00:43/03:00:00

A 54-year-old woman is commenced on bendroflumethiazide, in addition to ramipril, for the


control of hypertension.
Which of the following is the site of action of bendroflumethiazide?

A Ascending limb of the loop of Henle

B Collecting duct

C Descending limb of the loop of Henle

D Distal convoluted tubule

E Proximal convoluted tubule

Explanation 

D Distal convoluted tubule

Thiazide diuretics, such as bendroflumethiazide, inhibit the sodium chloride transporter in the
distal convoluted tubule. This transporter only reabsorbs around 5% of filtered sodium,
although delivery of this extra sodium load into the distal segment of the distal tubule leads
to increased potassium loss, which may result in symptomatic hypokalaemia in some cases.
Thiazides are now discouraged in the management of hypertension, in favour of thiazide-like
diuretics, such as indapamide, which are not thought to impact on glucose tolerance.

A Ascending limb of the loop of Henle

Loop diuretics inhibit the sodium–potassium–chloride cotransporter in the thick ascending


limb of the loop of Henle. This transporter normally reabsorbs about 25% of the sodium load,
and inhibition drives significant salt and water loss.

B Collecting duct

The collecting duct is mainly responsible for absorption of free water, via aquaporin-2
channels, the expression of which is driven by vasopressin. Potassium-sparing diuretics also
have some action on the proximal portion of the collecting duct.

C Descending limb of the loop of Henle

The descending limb of the loop of Henle has low permeability to ions but is highly
permeable to water. It is not a major site of diuretic action.
E Proximal convoluted tubule

Carbonic anhydrase inhibitors inhibit the transport of bicarbonate out of the proximal
convoluted tubule, which leads to less sodium reabsorption at this point and greater sodium,
bicarbonate and water loss in the urine.
51592

Rate this question:

Next Question

Previous Question Tag Question

Feedback End Review

Difficulty: Average

Peer Responses %

Q. Answered Flagged

Q1

Q2

Q3

Q4

Q5

Q6

Q7

Q8

Q9

0:00:43/03:00:00

A 29-year-old man, who has been teaching at a school in the Thai jungle, comes to the
Emergency Department for review, having just returned to the United Kingdom. He tells you
he suffered from 2–3 days of bloody diarrhoea and lower abdominal pain, which only
improved after taking a course of oral metronidazole.
Examination in the clinic is entirely normal.
From which of the following infections is he most likely to have been suffering?

A Bacillus cereus

B Campylobacter jejuni

C Entamoeba histolytica

D Giardia lamblia

E Salmonella typhi

Explanation 

C Entamoeba histolytica

Amoebic dysentery presents with bloody diarrhoea and lower abdominal pain. Symptoms
can last between 1–3 weeks and then resolve. It is only later that patients may present with
symptoms of a liver abscess. Metronidazole kills trophozoites, leading to a symptomatic
response, but does not eradicate cysts.

A Bacillus cereus

Bacillus cereus food poisoning is responsible for either profuse vomiting or severe diarrhoea,
with the onset around six hours after ingestion of contaminated food, most often reheated
rice products.

B Campylobacter jejuni

Campylobacter infection does lead to vomiting and bloody diarrhoea, from eating
contaminated poultry or dairy products. It does not respond to metronidazole. For patients
who are symptomatic, a quinolone or cephalosporin is usually considered.

D Giardia lamblia
Giardiasis does respond to treatment with metronidazole, although it produces symptoms
more akin to irritable bowel syndrome, rather than the bloody diarrhoea described here.

E Salmonella typhi

S. typhi leads to vomiting and diarrhoea, but a response to metronidazole, as described here,
would not be expected. Quinolones are more likely to be used in patients with severe
symptoms.
51633

Rate this question:

Next Question

Previous Question Tag Question

Feedback End Review

Difficulty: Average

Peer Responses %

Q. Answered Flagged

Q1

Q2

Q3

Q4

Q5

Q6

Q7

Q8

Q9

0:00:43/03:00:00

A 75-year-old woman is reviewed on the ward some six hours after a left hemiarthroplasty
under spinal anaesthetic. She has severe chronic obstructive pulmonary disease (COPD) and
currently takes high-dose inhaled corticosteroids, a long-acting muscarinic antagonist and a
long-acting beta2 agonist. The nurses are concerned about her oxygen (O 2) saturation of 91%
on air.
Which of the following is the most appropriate target for her O 2 saturation?

A 86%

B 90%

C 94%

D 98%

E 99%

Explanation 

B 90%

In severe COPD, a target O 2 saturation of 88–92% is entirely appropriate. Increasing her


inspired O 2 is not appropriate here, as it may precipitate significant carbon dioxide (CO 2)
retention, leading to decompensated type 2 respiratory failure.

A 86%

Eighty–six percent is below the threshold, even for severe COPD, and is likely to drive
significant shortness of breath and increase the risks of cardiac arrhythmia.

C 94%

Ninety–four percent may well be unattainable here due to emphysema and reduced area for
gas exchange. Chasing a saturation of 94% by increasing inspired O 2 will run significant risks
of CO 2 retention.

D 98%

Ninety–eight percent is unrealistic, given this patient’s severe COPD. Increasing inspired
oxygen will not achieve an O 2 saturation very much above that seen here because of the
effect it would have in decreasing the respiratory drive.
E 99%

Ninety–nine percent implies a fully functioning unimpaired respiratory system, with respect to
gas exchange, and is therefore incorrect.
51666

Rate this question:

Next Question

Previous Question Tag Question

Feedback End Review

Difficulty: Average

Peer Responses %

Q. Answered Flagged

Q1

Q2

Q3

Q4

Q5

Q6

Q7

Q8

Q9

0:00:43/03:00:00

A 42-year-old woman presents to the Emergency Department with a red eye and mild
discomfort only.
Examination reveals normal visual acuity and obvious reddening, localised to the medial side
of the eye. There is no discharge.
Which of the following is the most likely diagnosis?

A Acute closed-angle glaucoma

B Anterior uveitis

C Conjunctivitis

D Episcleritis

E Scleritis

Explanation 

D Episcleritis

A red eye where there are localised changes with mild discomfort and no loss of visual acuity
fits with a diagnosis of episcleritis. Artificial tears are usually sufficient to manage symptoms
in most cases. Topical corticosteroids may be required for prolonged symptoms.

A Acute closed-angle glaucoma

Acute closed-angle glaucoma is associated with severe pain, haloes around light sources,
photophobia and eye watering. The patient may be systemically unwell. Visual acuity is
significantly reduced. It is an ophthalmic emergency, and all cases require urgent referral to
ophthalmology.

B Anterior uveitis

Anterior uveitis is associated with blurred vision and headache, which would not fit with the
clinical features seen here. There may be pupil constriction and visible keratic precipitates.

C Conjunctivitis

Conjunctivitis is associated with eye pain, grittiness and a mucopurulent discharge.


Treatment with chloramphenicol eye drops is the first-line treatment for this condition.
E Scleritis

Scleritis is associated with more severe pain than that seen here. There is progressive
photophobia and visual impairment, rather than the picture seen here with no impact on
visual acuity.
51653

Rate this question:

Next Question

Previous Question Tag Question

Feedback End Review

Difficulty: Average

Peer Responses %

Q. Answered Flagged

Q1

Q2

Q3

Q4

Q5

Q6

Q7

Q8

Q9

0:00:43/03:00:00

A 72-year-old man is brought to the Emergency Department after becoming unwell at home
with nausea, vomiting, vertigo and double vision. He is unable to get out of his chair. He has
type 2 diabetes and hypertension and smokes ten cigarettes per day.
Examination reveals a blood pressure of 189/100 mmHg, and his pulse is 94 bpm and regular.
You note nystagmus to the left and left-sided Horner syndrome, and loss of pain and
temperature sensation affecting the right arm and leg and the right-hand side of the trunk.
Where is the most likely site of arterial occlusion?

A Anterior cerebral artery

B Anterior choroidal artery

C Posterior inferior cerebellar artery

D Ophthalmic artery

E Superior cerebellar artery

Explanation 

C Posterior inferior cerebellar artery

This clinical scenario fits well with lateral medullary syndrome, which can occur due to
occlusion of the posterior inferior cerebellar artery or of one of the vertebral arteries or the
basilar artery. Other disease of the cerebral arterial tree is usually apparent, with the circle of
Willis only intact in around 50% of patients with lateral medullary syndrome. The syndrome is
produced by a wedge-shaped infarction of the lateral medulla and the inferior surface of the
cerebellum.

A Anterior cerebral artery

Occlusion of the anterior cerebral artery leads to infarction of the medial aspects of the
frontal and parietal lobes, basal ganglia, anterior fornix and anterior corpus callosum. In this
situation, unilateral motor and sensory loss would be expected, with some evidence of
speech disturbance.

B Anterior choroidal artery

Interruption of blood flow through the anterior choroidal artery can result in hemiplegia on
the contralateral side of the body, contralateral loss of sensation and homonymous
hemianopia.
D Ophthalmic artery

Ophthalmic artery occlusion leads to unilateral visual loss, rather than the scenario here with
posterior territory symptoms.

E Superior cerebellar artery

Superior cerebellar artery infarction is more likely to present with cerebellar signs without the
other features, including Horner syndrome, seen here. Vertigo, in particular, is more likely to
be seen in posterior inferior cerebellar artery infarction.
51642

Rate this question:

Next Question

Previous Question Tag Question

Feedback End Review

Difficulty: Average

Peer Responses %

Q. Answered Flagged

Q1

Q2

Q3

Q4

Q5

Q6

Q7

Q8
Q. Answered Flagged 
0:00:43/03:00:00

A 32-year-old woman presents with a third episode of small joint polyarthritis in the last year
to the Rheumatology Clinic. On each occasion, her joint pain has responded to a course of
non-steroidals, and she has been well between attacks, with no long-lasting joint damage.
Examination reveals no skin rashes, normal nails with no pitting and small joint polyarthritis,
predominantly affecting the proximal interphalangeal and metacarpophalangeal joints.
Which of the following is the most appropriate disease-modifying agent for her?

A Adalimumab

B Hydroxychloroquine

C Infliximab

D Low-dose prednisolone

E Methotrexate

Explanation 

B Hydroxychloroquine

This patient has palindromic rheumatism, with recovery of joint function between attacks.
Anti-malarials are generally considered the intervention of choice for palindromic rheumatism
because of a relatively benign side-effect profile vs other disease-modifying anti-rheumatic
drugs (DMARDs). Patients may progress to rheumatoid arthritis or other connective tissue
disease; continued follow-up is therefore advised.

A Adalimumab

This is an anti-tumour necrosis factor (TNF) agent used in the treatment of rheumatoid
arthritis, psoriatic arthritis and inflammatory bowel disease. It is not considered a first-choice
option for palindromic rheumatism.

C Infliximab

This, like adalimumab, is an anti-TNF agent. It does not have a role in the initial treatment of
palindromic rheumatism.

D Low-dose prednisolone
Even low-dose prednisolone has an unfavourable adverse event profile, with respect to
weight gain, impaired glucose tolerance and osteoporosis, compared to other options, for the
treatment of palindromic rheumatism.

E Methotrexate

Methotrexate is an inhibitor of deoxyribonucleic acid synthesis. It has an unfavourable side-


effect profile, compared to hydroxychloroquine, in the treatment of palindromic rheumatism.
It does, however, remain a first-choice DMARD for the treatment of rheumatoid arthritis.
51675

Rate this question:

Next Question

Previous Question Tag Question

Feedback End Review

Difficulty: Average

Peer Responses %

Q. Answered Flagged

Q1

Q2

Q3

Q4

Q5

Q6

Q7

Q8

Q9

0:00:43/03:00:00

A 50-year-old man presents to the Dermatology Clinic with an intensely itchy rash affecting
the flexor surfaces of both wrists. It comprises several red-violet papules, which are shiny on
top, and you can see white streaks over the rash. He tells you that the spots heal and they
leave brown areas of discoloured skin.
Which of the following is the most appropriate way to treat the itching?

A Chlorphenamine

B Isotretinoin

C Levamisole

D Oral prednisolone

E Topical corticosteroids

Explanation 

E Topical corticosteroids

This patient has a typical rash associated with lichen planus where shiny red papules appear
on flexural skin surfaces. It often affects the wrists, forearms and legs, and lesions may
appear in the mouth. For limited disease, topical corticosteroids are an appropriate
intervention.

A Chlorphenamine

Although antihistamines may have a minor effect on itching, they do not resolve the
underlying rash and, as such, are not the appropriate initial option here.

B Isotretinoin

Isotretinoin has been used for the treatment of oral lichen planus; however, in this case, with
limited disease, topical steroids are more appropriate.

C Levamisole

Levamisole, an anti-helminthic known to have immunomodulatory properties, is used as a


treatment for oral lichen planus resistant to other interventions.
D Oral prednisolone

In this situation, with only local skin irritation affecting the flexural surfaces of both wrists,
systemic corticosteroid therapy is not required.
51612

Rate this question:

Next Question

Previous Question Tag Question

Feedback End Review

Difficulty: Average

Peer Responses %

Q. Answered Flagged

Q1

Q2

Q3

Q4

Q5

Q6

Q7

Q8

Q9

0:00:43/03:00:00

You are interested in trials of a new agent which is thought to reduce free water excretion.
Which part of the nephron is normally impermeable to water?

A Ascending limb of the loop of Henle

B Collecting duct

C Descending limb of the loop of Henle

D Distal convoluted tubule

E Proximal convoluted tubule

Explanation 

A Ascending limb of the loop of Henle

In the ascending limb, there is active reabsorption of ions, but it is relatively impermeable to
water. As a result, interstitial fluid can become hyperosmotic, relative to the fluid inside the
tubule.

B Collecting duct

The collecting duct is permeable to water, via aquaporin-2 channels, the expression of which
is dependent upon levels of antidiuretic hormone, (vasopressin).

C Descending limb of the loop of Henle

The descending limb of the loop of Henle is impermeable to reabsorption of ions, but water is
freely absorbed.

D Distal convoluted tubule

The distal convoluted tubule is responsible for regulation of sodium, potassium, calcium and
pH, with a number of transporters, including the thiazide-sensitive sodium–chloride
cotransporter, the adenosine triphosphate (ATP)-dependent sodium/potassium antiporter,
and the TRPV5 channels which govern the transport of calcium.

E Proximal convoluted tubule


The proximal convoluted tubule is responsible for reabsorption of water and a number of
ions, including sodium, potassium, urea, phosphate and citrate.
51602

Rate this question:

Next Question

Previous Question Tag Question

Feedback End Review

Difficulty: Average

Peer Responses %

Q. Answered Flagged

Q1

Q2

Q3

Q4

Q5

Q6

Q7

Q8

Q9

0:00:43/03:00:00

A 40-year-old man who has suffered from chronic constipation for the past few years
presents to the clinic for review. He was diagnosed with an anal fissure three months earlier
and is concerned that symptoms, including severe pain on defecation, have not improved
after increasing fluids and fibre in his diet and cutting out caffeine. Examination reveals that
his fissure has not healed.
Which of the following is the most appropriate next step?

A Botulinum toxin

B Glyceryl trinitrate (GTN) ointment

C Lactulose

D Lidocaine ointment

E Oral amlodipine

Explanation 

B Glyceryl trinitrate (GTN) ointment

GTN ointment is a next logical step in patients with an anal fissure who still have symptoms
despite increasing fibre and fluid intake. Patients should be counselled about the risk of
chronic headache when using this medication. A Cochrane review has suggested it is
marginally more successful than placebo in promoting healing of a lesion.

A Botulinum toxin

Botulinum toxin is approximately as effective as GTN ointment, invasive and considerably


more expensive. It is not therefore an appropriate choice here. Patients may also suffer
temporary faecal incontinence post-therapy.

C Lactulose

Lactulose is a stool-softening agent which is most effective in children for the treatment of
anal fissure. Given this patient has already increased his fibre and fluid intake, it is unlikely to
be effective.

D Lidocaine ointment
Lidocaine ointment is used as a second-line medical therapy for patients with anal fissure
who have severe pain despite other measures to control their symptoms. It is usually
administered before defecation.

E Oral amlodipine

Calcium channel antagonists do relax gastrointestinal smooth muscle tone. In patients with
symptoms resistant to local measures to treat anal fissure, they may achieve additional anal
relaxation and relieve pain.
51623

Rate this question:

Next Question

Previous Question Tag Question

Feedback End Review

Difficulty: Average

Peer Responses %

Q. Answered Flagged

Q1

Q2

Q3

Q4

Q5

Q6

Q7

Q8

Q9

 External Links

Nelson, R L, Thomas, K, Morgan, J, Jones, A. (2012). Non-surgical therapy for anal fissure.
cochrane.org/CD003431/COLOCA_non-surgical-therapy-for-anal-fissure.
(https://www.cochrane.org/CD003431/COLOCA_non-surgical-therapy-for-anal-
fissure.)
0:00:43/03:00:00

A 29-year-old man comes to the Emergency Department concerned that his new wife is
trying to kill him. He is hearing voices telling him to ‘finish her off’ before she stabs him. They
returned from their honeymoon in Kenya one week earlier.
He is taking mefloquine for malaria prophylaxis. His wife has moved out of the house.
Which of the following is the most likely diagnosis?

A Adjustment disorder

B Depression

C Drug-induced psychosis

D Schizophrenia

E Viral encephalitis

Explanation 

C Drug-induced psychosis

Mefloquine is known to be associated with a range of neuropsychiatric adverse effects. These


may include hallucinations, delusions, suicidal ideation, mania and schizophreniform disorder.
For this reason, mefloquine is not recommended for patients with a history of psychiatric
illness, and its alternatives should be considered in patients with a family history of
psychiatric disorder.

A Adjustment disorder

An adjustment disorder could account for any mood changes and physical symptoms which
appear after getting married (considered a stressful life event) but it would not account for
the psychotic behaviour seen here.

B Depression

Depression could account for mood changes after his recent honeymoon, but it could in no
way account for the hallucinations and delusions reported here.

D Schizophrenia
The symptoms here could fit with a diagnosis of acute schizophrenia, although the history of
mefloquine use preceding the presentation means that drug induced psychosis is much more
likely to be the underlying diagnosis.

E Viral encephalitis

Viral encephalitis is more likely to present with short term memory loss, drowsiness and
confusion, than the features of acute psychosis seen here.
51655

Rate this question:

Next Question

Previous Question Tag Question

Feedback End Review

Difficulty: Average

Peer Responses %

Q. Answered Flagged

Q1

Q2

Q3

Q4

Q5

Q6

Q7

Q8

Q9

 External Links

Electronic Medicines Compendium. (2018). Mefloquine.


medicines.org.uk/emc/product/9670
(https://www.medicines.org.uk/emc/product/9670)
0:00:43/03:00:00

A 19-year-old woman who suffers from an eating disorder is admitted to the Emergency
Department following a collapse. She is known to abuse both laxatives and diuretics, which
she buys on the Internet.
Her blood pressure is 90/60 mmHg, and her pulse is 90 bpm and regular. Her body mass
index is 17.5 kg/m 2.
Investigations:

Haemoglobin (Hb) 10.0 g/dl (11.5–15.5 g/dl)

White cell count (WCC) 6.1 × 10 9/l (4–11 × 10 9/l)

Platelets (PLT) 209 × 10 9/l (150–400 × 10 9/l)

Sodium (Na +) 139 mmol/l (135–145 mmol/l)

Potassium (K +) 2.8 mmol/l (3.5–5.0 mmol/l)

Bicarbonate (HCO 3) 32 mmol/l (24–30 mmol/l)

Creatinine (Cr) 95 µmol/l (5–120 mmol/l)

Which of the following would you expect to see on the 12-lead electrocardiogram?

A Broad P waves

B J waves

C ST elevation

D T wave inversion

E U waves

Explanation 

E U waves

U waves are seen in marked hypokalaemia. They are a small deflection classically seen in
leads V2 and V3, usually in the same direction as the T wave and seen directly after it. They
are thought to represent the electrical change seen with repolarisation of the Purkinje fibres
and are also seen in hypocalcaemia and hypomagnesaemia.

A Broad P waves
Broad P waves, usually notched, are classically related to left atrial enlargement due to mitral
stenosis.

B J waves

J waves are positive deflections occurring at the junction between the QRS complex and the
ST segment. They are related to hypothermia, not to hypokalaemia. They resolve, as the body
temperature is increased.

C ST elevation

ST elevation is characteristically associated with acute myocardial ischaemia or, in the case of
saddle-shaped ST elevation, pericarditis.

D T wave inversion

T wave inversion is a feature of myocardial ischaemia; it is not a characteristic feature of


hypokalaemia.
51575

Rate this question:

Next Question

Previous Question Tag Question

Feedback End Review

Difficulty: Average

Peer Responses %

Q. Answered Flagged

Q1

Q2

Q3
Q. Answered Flagged

Q4

Q5

Q6

Q7

Q8 
0:00:43/03:00:00

A 23-year-old woman presents to the Emergency Department with symptoms of acute


hepatitis around five days after returning from a holiday to Morocco. She ate local food
during her stay and did not seek any vaccinations from the general practitioner (GP) before
travelling. Examination reveals jaundice and she is tender in the right upper quadrant on
abdominal palpation.
Investigations:

Haemoglobin (Hb) 14.4 g/dl (11.5–15.5 g/dl)

White cell count (WCC) 10.2 × 10 9/l (4–11 × 10 9/l)

Platelets (PLT) 103 × 10 9/l (150–400 × 10 9/l)

Sodium (Na +) 145 mmol/l (135–145 mmol/l)

Potassium (K +) 4.3 mmol/l (3.5 – 5.0 mmol/l)

Creatinine (Cr) 123 µmol/l (50 – 120 µmol/l)

Alanine aminotransferase (ALT) 2345 iu/l (5–30 iu/l)

Gamma glutamyl transpeptidase (GGT) 432 iu/l (5–30 iu/l)

Alkaline phosphatase (ALP) 162 iu/l (30–130 iu/l)

Bilirubin 12 µmol/l (2–17 µmol/l)

Which of the following is the most likely cause of the raised ALT seen here?

A Common bile duct obstruction

B Hepatocyte apoptosis

C Hepatocyte necrosis

D Hepatocyte regeneration

E Intrahepatic bile duct obstruction

Explanation 

C Hepatocyte necrosis

This patient has increased transaminases because of hepatocyte necrosis, potentially due to
hepatitis A or hepatitis E infection. Cell necrosis is injury which results in premature cell death
by autolysis.
A Common bile duct obstruction

Common bile duct obstruction may result in a rise in transaminases, although the elevation is
usually lower than that seen here. A greater relative increase in alkaline phosphatase and GGT
is classically seen.

B Hepatocyte apoptosis

Apoptosis is programmed cell death, as opposed to injury and autolysis which are seen in
cellular necrosis.

D Hepatocyte regeneration

Hepatocyte regeneration does occur in the recovery period after acute hepatitis A infection,
although it is not associated with a rise in transaminases.

E Intrahepatic bile duct obstruction

Intrahepatic bile duct obstruction is associated with a greater elevation in alkaline


phosphatase, coupled with a rise in transaminases, rather than the predominant rise in ALT
seen here.
51596

Rate this question:

Next Question

Previous Question Tag Question

Feedback End Review

Difficulty: Average

Peer Responses %

Q. Answered Flagged

Q1
Q. Answered Flagged

Q2

Q3

Q4

Q5

Q6

Q7

Q8 
0:00:43/03:00:00

A 27-year-old man who works as a dog trainer comes to the Emergency Department for
review of a dog bite which he sustained some three days earlier. Although he cleaned the
bite at the time, there is now surrounding erythema and oedema where the tooth marks are
on the dorsum of his hand, and you note axillary lymphadenopathy on palpation. He is
pyrexial at 38.8 °C.
Which of the following is the most appropriate intervention?

A Intravenous (IV) ceftriaxone

B IV co-amoxiclav

C IV flucloxacillin

D Oral doxycycline and metronidazole

E Oral flucloxacillin

Explanation 

B IV co-amoxiclav

Dog bites normally introduce a mixed population of anaerobes and aerobes, including
possibly Pasteurella. This drives the choice of an antibiotic agent which has wide coverage,
such as co-amoxiclav, as the first-line agent for treating bite wounds. It is given orally for
prophylaxis and for less severe infections. In this case, the fever and axillary
lymphadenopathy prompts IV administration at least initially.

A Intravenous (IV) ceftriaxone

A cephalosporin is not an appropriate option here because of lack of adequate coverage


against anaerobic bacteria.

C IV flucloxacillin

Although this would provide good coverage against staphylococcal infection, the type of
mixed infections seen with dog bites are not appropriately managed with flucloxacillin alone.

D Oral doxycycline and metronidazole

This is an appropriate broad-spectrum option for prophylaxis against dog bite infections in
patients who are allergic to penicillin.
E Oral flucloxacillin

Like IV flucloxacillin, oral flucloxacillin does not provide a broad enough spectrum of
coverage against the mixed population of aerobes and anaerobes causing infection here.
51638

Rate this question:

Next Question

Previous Question Tag Question

Feedback End Review

Difficulty: Average

Peer Responses %

Q. Answered Flagged

Q1

Q2

Q3

Q4

Q5

Q6

Q7

Q8

Q9

0:00:43/03:00:00

A 59-year-old woman who has suffered from bronchiectasis since childhood, because of
whooping cough infection, comes to the Renal Clinic for review. She has developed
progressive pitting oedema of both lower limbs over the past few months.
Examination reveals a blood pressure of 123/82 mmHg, and her pulse is 70 bpm and regular.
Urine analysis reveals protein +++ and blood negative. Creatinine level is stable at 123 µmol/l.
Which of the following is the most likely cause of her proteinuria?

A Focal segmental glomerulosclerosis (FSGS)

B Immune complex-mediated membranoproliferative glomerulonephritis (MPGN)

C Membranous nephropathy

D Multiple myeloma

E Systemic amyloidosis

Explanation 

E Systemic amyloidosis

The most likely diagnosis here is systemic amyloidosis related to chronic inflammation as a
result of bronchiectasis. Amyloid deposition in the kidney leads to a progressively worsening
protein leak and drives presentation with symptoms of peripheral oedema. For cardiac
amyloid infiltration, magnetic resonance imaging (MRI) is the initial investigation of choice.

A Focal segmental glomerulosclerosis (FSGS)

FSGS is a common cause of renal impairment in younger adults. It is associated with


haematuria, proteinuria and hypertension, and may progress to frank proteinuria over time.

B Immune complex-mediated membranoproliferative glomerulonephritis (MPGN)

Immune-complex mediated MPGN is most commonly seen in patients with rheumatoid


arthritis, systemic lupus erythematosus or Sjögren syndrome. Monoclonal gammopathies may
be associated with both bronchiectasis and MPGN, although that does not appear to have
been the cause of bronchiectasis seen here.

C Membranous nephropathy
Idiopathic membranous nephropathy may present in patients with bronchiectasis, although it
is more likely that chronic inflammation leading to amyloidosis is the cause of the proteinuria
seen here.

D Multiple myeloma

Multiple myeloma can lead to proteinuria as a result of light chain deposition. There are,
however, no other features of myeloma outlined as part of the scenario here.
51659

Rate this question:

Next Question

Previous Question Tag Question

Feedback End Review

Difficulty: Average

Peer Responses %

Q. Answered Flagged

Q1

Q2

Q3

Q4

Q5

Q6

Q7

Q8

Q9

0:00:43/03:00:00

A 67-year-old man with idiopathic pulmonary fibrosis presents to the Emergency Department
with a rapid worsening in his shortness of breath, pleuritic chest pain and fever. His
temperature is 38.5 °C, and his pulse is 95 bpm and regular. His blood pressure is 112/82
mmHg. There are coarse crackles throughout both lung fields.
On his chest X-ray, you note that the right heart border is obscured.
Which of the following is the most likely reason for the chest X-ray appearance?

A Left upper lobe consolidation

B Left lower lobe consolidation

C Right lower lobe consolidation

D Right middle lobe consolidation

E Right upper lobe consolidation

Explanation 

D Right middle lobe consolidation

Loss of the right heart border is consistent with right middle lobe consolidation, which
potentially accounts for the acute deterioration in this patient’s shortness of breath and
pyrexia. Intravenous (IV) antibiotic therapy, with a combination of IV co-amoxiclav and
clarithromycin, is indicated.

A Left upper lobe consolidation

The left heart border is adjacent to the lingula of the left upper lobe, meaning consolidation
here can obscure the left heart border.

B Left lower lobe consolidation

Obscuration of the border between the lung and the diaphragm is suggestive of left lower
lobe consolidation.

C Right lower lobe consolidation

Like left lower lobe consolidation, right lower lobe consolidation obscures the border with the
diaphragm.
E Right upper lobe consolidation

Right upper lobe consolidation is associated with increased opacification above the
horizontal fissure. It may obscure the border between the lung and the clavicle.
51661

Rate this question:

Next Question

Previous Question Tag Question

Feedback End Review

Difficulty: Average

Peer Responses %

Q. Answered Flagged

Q1

Q2

Q3

Q4

Q5

Q6

Q7

Q8

Q9

0:00:43/03:00:00

A 45-year-old man is reviewed in the Rheumatology Clinic. He has been referred by his
general practitioner (GP) with poorly controlled asthma, sinusitis and an episode of left
common peroneal nerve palsy.
Examination reveals a blood pressure of 145/82 mmHg, and his pulse is 78 bpm and regular.
Auscultation of the chest reveals bilateral wheeze.
Investigations:

Haemoglobin (Hb) 11.9 g/dl (13.5–17.5 g/dl)

White cell count (WCC) 11.4 × 10 9/l (elevated eosinophils) (4–11 × 10 9/l)

Platelets (PLT) 203 × 10 9/l (150–400 × 10 9/l)

Sodium (Na +) 142 mmol/l (135–145 mmol/l)

Potassium (K +) 4.9 mmol/l (3.5–5.0 mmol/l)

Creatinine (Cr) 122 µmol/l (50–120 µmol/l)

Erythrocyte sedimentation rate (ESR) 72 mm/hour (0–10 mm/hour)

Which of the following is the most likely diagnosis?

A Eosinophilic granulomatosis with polyangiitis (EGPA)

B Goodpasture syndrome

C Granulomatosis with polyangiitis

D Polyarteritis nodosa

E Primary eosinophilic syndrome

Explanation 

A Eosinophilic granulomatosis with polyangiitis (EGPA)

This patient has several pointers to the diagnosis of EGPA, including poorly controlled
asthma, sinusitis, mononeuritis and peripheral blood eosinophilia. Thirty to 40% of patients
with EGPA are pANCA-positive, and biopsy of a lung lesion characteristically demonstrates
necrotising granulomas. High-dose corticosteroids are the initial intervention of choice.

B Goodpasture syndrome
Goodpasture syndrome is associated with anti-glomerular basement membrane antibodies,
which lead to pulmonary and renal vasculitis, rather than the picture of asthma and sinusitis
seen here. It is also not associated with eosinophilia.

C Granulomatosis with polyangiitis

Granulomatosis with polyangiitis is more suggested by pulmonary infiltration, rather than a


picture of poorly controlled asthma, and is not associated with peripheral blood eosinophilia.

D Polyarteritis nodosa

Asthma and eosinophilia are not prominent features of polyarteritis nodosa, which is a
vasculitis primarily affecting small and medium-sized arteries.

E Primary eosinophilic syndrome

Primary eosinophilic syndrome is a diagnosis of exclusion, once other causes such as


infection and vasculitis have been excluded first.
51671

Rate this question:

Next Question

Previous Question Tag Question

Feedback End Review

Difficulty: Average

Peer Responses %

Q. Answered Flagged

Q1

Q2

Q3
Q. Answered Flagged

Q4

Q5

Q6

Q7

Q8 
0:00:43/03:00:00

A 54-year-old man is reviewed on the ward some six hours after admission with central
crushing chest pain radiating to his left arm. Initial ST depression has resolved following
treatment with 300 mg of aspirin, ticagrelor and fondaparinux. He has now developed atrial
fibrillation (AF) with a ventricular rate of 64 bpm. Examination reveals a blood pressure of
122/84 mmHg. There are no signs of cardiac failure. Troponin is elevated at 5.2 ng/ml.
Which of the following is the most appropriate way to manage his anticoagulation?

A Continue aspirin, clopidogrel and fondaparinux

B Switch fondaparinux to apixaban

C Switch fondaparinux to dalteparin

D Switch fondaparinux to rivaroxaban

E Switch fondaparinux to warfarin

Explanation 

A Continue aspirin, clopidogrel and fondaparinux

This patient has acute AF during a period of ischaemia. At this point, AF may be temporary
and resolve spontaneously with management of the ischaemia. Aspirin, clopidogrel and
fondaparinux have proven outcome benefits in this situation and represent the intervention of
choice. Oral factor X inhibitors and warfarin are potentially less effective and difficult to
reverse in an emergency situation. Over the longer term, if the patient remains in persistent
AF, then a factor Xa inhibitor should be considered if the CHA 2DS 2-VASc score is > 1.

B Switch fondaparinux to apixaban

There is no easy way to reverse oral factor X inhibitors in an acute situation, meaning that
switching to apixaban here is not appropriate, given urgent intervention such as
percutaneous coronary intervention (PCI) or later coronary artery bypass grafting may be
required.

C Switch fondaparinux to dalteparin

Low-molecular-weight heparin is less effective in improving cardiovascular outcomes in acute


coronary syndrome (ACS), compared to fondaparinux. The switch here is therefore not an
appropriate one.
D Switch fondaparinux to rivaroxaban

Like apixaban, rivaroxaban cannot be easily reversed, and there is no evidence in the context
of ACS that it is any more effective than fondaparinux.

E Switch fondaparinux to warfarin

Warfarin requires loading, meaning it is not appropriate here. However, it can be considered
as a long-term option for anticoagulation in the presence of permanent AF.
51593

Rate this question:

Next Question

Previous Question Tag Question

Feedback End Review

Difficulty: Average

Peer Responses %

Q. Answered Flagged

Q1

Q2

Q3

Q4

Q5

Q6

Q7

Q8

Q9

0:00:43/03:00:00

A 43-year-old woman, who is a known alcoholic, comes to the Dermatology Clinic for review.
She has developed an erythematous skin rash which worsens on sun exposure. The rash has
been known to blister and heals, with hyperpigmentation and hypertrichosis. Her general
practitioner (GP) has noted elevated transaminase and ferritin levels.
Which of the following is the most likely diagnosis?

A Acute intermittent porphyria (AIP)

B Congenital erythropoietic porphyria (CEP)

C Hereditary coproporphyria (HCP)

D Porphyria cutanea tarda (PCT)

E Variegate porphyria (VP)

Explanation 

D Porphyria cutanea tarda (PCT)

PCT is a cutaneous porphyria which fits well with the clinical picture seen here. It is
associated with alcohol abuse, oestrogen exposure and hepatitis C infection. Iron overload is
also frequently found to coexist with PCT. Diagnosis is made by measurement of porphyrin
levels in blood, urine and stool. Avoidance of alcohol is the key initial intervention.
Venesection may be required for symptomatic iron overload.

A Acute intermittent porphyria (AIP)

AIP presents acutely with hypertension, hyponatraemia and neuropsychiatric features. A


range of drugs, including the oral contraceptive pill, may precipitate episodes of AIP, and
intravenous (IV) haem arginate is the intervention of choice.

B Congenital erythropoietic porphyria (CEP)

CEP presents in childhood with red urine which fluoresces in diapers, making it a relatively
straightforward diagnosis. Patients suffer from severe photosensitivity. Splenectomy,
chloroquine and bone marrow transplantation are all potential interventions.

C Hereditary coproporphyria (HCP)


HCP is a mixed porphyria where patients may present with both a photosensitive rash and
neuropsychiatric features. It is not as strongly associated with alcohol exposure as PCT and
does not therefore fit with the clinical scenario seen here.

E Variegate porphyria (VP)

Like HCP, VP is a mixed porphyria with patients presenting with both skin rashes and acute
neuropsychiatric episodes. It is less common than PCT.
51610

Rate this question:

Next Question

Previous Question Tag Question

Feedback End Review

Difficulty: Average

Peer Responses %

Q. Answered Flagged

Q1

Q2

Q3

Q4

Q5

Q6

Q7

Q8

Q9

0:00:43/03:00:00

A 74-year-old man is admitted to the Emergency Department after a collapse. On arrival, his
blood pressure is 90/60 mmHg; his pulse is 180 bpm and regular. He is complaining of central
crushing chest pain. The cardiac monitor shows a broad complex tachycardia.
Which of the following features on the electrocardiogram would be most supportive of a
diagnosis of ventricular tachycardia (VT) vs supraventricular tachycardia with aberrant
conduction?

A Negative concordance across the chest leads

B Normal axis

C Presence of typical right bundle branch block morphology

D QRS 140 ms in diameter

E RSR complexes with a taller second R wave

Explanation 

A Negative concordance across the chest leads

When leads V1 to V6 show entirely positive (R) or entirely negative (QS) complexes, with no
RS complexes seen, this is consistent with VT as the underlying diagnosis.

B Normal axis

Extreme axis deviation, the so-called ‘northwest axis’, where the QRS is positive in lead aVR
and negative in leads I and aVF, is suggestive of VT.

C Presence of typical right bundle branch block morphology

Absence of either a typical right bundle or left bundle branch block morphology is consistent
with VT.

D QRS 140 ms in diameter

A broad QRS complex, typically > 160 ms, is consistent with a diagnosis of VT.

E RSR complexes with a taller second R wave


RSR complexes with a taller second R wave are consistent with normal right bundle branch
block. Where the left R wave is taller, this is consistent with VT.
51576

Rate this question:

Next Question

Previous Question Tag Question

Feedback End Review

Difficulty: Average

Peer Responses %

Q. Answered Flagged

Q1

Q2

Q3

Q4

Q5

Q6

Q7

Q8

Q9

0:00:43/03:00:00

A 45-year-old woman with rheumatoid arthritis (RA) has failed to gain control of her
symptoms with adalimumab and methotrexate, and a plan is made to switch her monoclonal
antibody treatment to rituximab.
Which of the following is the mode of action of rituximab?

A Anti-CD19

B Anti-CD20

C Anti-interleukin (IL)-2

D Anti-IL-6

E Anti-tumour necrosis factor (TNF)-alpha

Explanation 

B Anti-CD20

CD20 is a receptor found on B-lymphocytes. Anti-CD20 monoclonal antibodies lead to B-cell


depletion and a reduction in B-cell T-cell cross-talk and overall antibody formation in patients
with autoimmune disease such as RA. They also have a role in the treatment of
haematological malignancy.

A Anti-CD19

CD19 is a target for B-cell lymphoma, and genetically engineered T-cells which target the
CD19 receptor and drug antibody conjugates which target the receptor are both promising as
treatment modalities.

C Anti-interleukin (IL)-2

Recombinant IL-2 is used in the treatment of metastatic renal cell carcinoma. It may
exacerbate pre-existing autoimmune disease when used at high dose. Low doses of IL-2 are
under investigation as a treatment for a range of autoimmune diseases, as at low doses, IL-2
has a differential effect on T-effector and T-regulatory cells.

D Anti-IL-6
Toclizumab is an anti-IL-6 monoclonal antibody which has efficacy in the treatment of RA. It
is recommended by the National Institute for Health and Care Excellence (NICE) in patients
who have failed to gain control of symptoms with a conventional disease-modifying anti-
rheumatic drug (DMARD), although the guidance does not differentiate between a number of
monoclonal antibodies, suggesting that acquisition cost should play a role in the decision
process.

E Anti-tumour necrosis factor (TNF)-alpha

Adalimumab itself is an example of an agent that works through the TNF pathway. Other
monoclonal antibodies with anti-TNF activity include etanercept and golimumab.
51590

Rate this question:

Next Question

Previous Question Tag Question

Feedback End Review

Difficulty: Average

Peer Responses %

Q. Answered Flagged

Q1

Q2

Q3

Q4

Q5

Q6

Q7

Q8
Q. Answered Flagged 

 External Links

National Institute for Health and Care Excellence. 2016. Adalimumab, etanercept, infliximab, certolizumab …
nice.org.uk/guidance/ta375/chapter/1-Recommendations
(https://www.nice.org.uk/guidance/ta375/chapter/1-Recommendations)
0:00:43/03:00:00

A 23-year-old woman is reviewed on the ward some 24 hours after admission with diabetic
ketoacidosis (DKA). She has improved and begun to eat normal food, and her pH has
returned to normal. The nurses are concerned because her urine is still positive for ketones.
Which of the following is the most likely cause of the positive result?

A Acetoacetate

B Beta-hydroxybutyrate

C Lactate

D Acetone

E Succinate

Explanation 

A Acetoacetate

The two main ketones produced by the liver are acetoacetate and beta-hydroxybutyrate,
while smaller amounts of acetone are also formed. In response to treatment, a much more
rapid fall in beta-hydroxybutyrate is usually observed, compared to acetoacetate, meaning
that acetoacetate is the most likely culprit for the positive result seen here.

B Beta-hydroxybutyrate

Beta-hydroxybutyrate is a large contributor to the ketone load seen in DKA, with the ratio of
beta-hydroxybutyrate to acetoacetate rising from the usual 1 : 1 to as high as 10 : 1 at
presentation, although beta-hydroxybutyrate levels rapidly return to normal in response to
insulin therapy.

C Lactate

Lactate is increased in poor tissue perfusion, and normally any increase occurs in DKA in
response to a period of hypotension and hypovolaemia.

D Acetone

Acetone is produced in DKA but in smaller amounts, compared to the other two ketones. It is
detected by many healthcare professionals and is reported as a smell of ‘pear drops’.
E Succinate

Succinate is the substrate for the production of fumarate by the enzyme succinate
dehydrogenase. It is generated by mitochondria, as part of the tricarboxylic acid (TCA) cycle,
and is not a ketoacid.
51600

Rate this question:

Next Question

Previous Question Tag Question

Feedback End Review

Difficulty: Average

Peer Responses %

Q. Answered Flagged

Q1

Q2

Q3

Q4

Q5

Q6

Q7

Q8

Q9

0:00:43/03:00:00

A 22-year-old student is admitted straight from the airport with severe fever, the ‘worst
muscle and bone pain he has ever suffered’, frontal headache, nausea and vomiting. He was
returning from a bike tour in Cambodia and tells you he has taken malaria prophylaxis and
was feeling completely well until just after beginning the plane journey.
His blood pressure is 95/65 mmHg, and his pulse is 95 bpm and regular. He looks
dehydrated. He is mildly tender on abdominal palpation. You note a morbilliform rash. His
temperature is 39.6 °C.
Investigations:

Haemoglobin (Hb) 14.5 g/dl (13.5–17.5 g/dl)

White cell count (WCC) 4.3 × 10 9/l (4–11 × 10 9/l)

Platelets (PLT) 231 × 10 9/l (150–400 × 10 9/l)

Sodium (Na +) 145 mmol/l (135–145 mmol/l)

Potassium (K +) 5.0 mmol/l (3.5–5.0 mmol/l)

Creatinine (Cr) 131 µmol/l (50–120 µmol/l)

Glucose 5.4 mmol/l (3.5–5.5 mmol/l)

Thick and thin film Negative for malaria

What is the most likely diagnosis?

A Dengue fever

B Infectious mononucleosis

C Malaria

D Tick typhus

E Yellow fever

Explanation 

A Dengue fever

The most likely diagnosis here, given the very abrupt onset of fever, rash, bone and muscle
pain, is dengue fever, which is spread by the Aedes mosquito and is endemic to South East
Asia. Serology with positive immunoglobulin M antibodies can confirm the diagnosis, and
polymerase chain reaction techniques to detect the virus are increasingly available.
Treatment is supportive with analgesia and rehydration. It is important to assess for signs of
severe dengue or dengue haemorrhagic fever. If found, this can be life-threatening. In these
cases, patients often require blood product transfusion and more intensive monitoring. Those
patients already exposed to dengue fever are thought to be most at risk for dengue
haemorrhagic fever; therefore, it is very rare for travellers to get it.

B Infectious mononucleosis

This is more likely to be associated with a gradual onset of symptoms and evidence of
pharyngitis, as opposed to the scenario here with abrupt onset of symptoms, including
myalgia and rash.

C Malaria

The thick and thin film is negative, which does not absolutely rule out the condition, but given
the severity of symptoms, other evidence to support a malaria diagnosis, such as haemolysis,
would be expected. He also confirms he has been compliant with malaria prophylaxis.

D Tick typhus

Tick typhus does present with fevers and myalgia, although a clear history of a tick bite
would be expected. The rash is also usually seen later in the course of the disease, a few days
after other symptoms have begun.

E Yellow fever

Yellow fever occurs in sub-Saharan Africa and tropical South America. Although the initial
presentation may be similar, it is effectively ruled out here because of the geography.
51636

Rate this question:

Next Question

Previous Question Tag Question

Feedback End Review

Difficulty: Average

Peer Responses %
Q. Answered Flagged

Q1

Q2

Q3

Q4

Q5

Q6

Q7

Q8

Q9

0:00:43/03:00:00

A 49-year-old woman comes to the clinic some six weeks after a diagnosis of type 2
diabetes. She has been monitoring her fasting blood glucose levels, having made changes to
her diet, and her fasting blood sugar levels are still in the range of 8–10 mmol/l. Her blood
pressure is 149/90 mmHg and pulse 80 bpm and regular; her chest is clear. Abdomen is soft
and non-tender, and her body mass index (BMI) is 33 kg/m 2.
Which of the following is the most appropriate initial treatment for her diabetes?

A Acarbose

B Gliclazide

C Metformin

D Pioglitazone

E Sitagliptin

Explanation 

C Metformin

Metformin is the initial treatment of choice in this obese patient with type 2 diabetes. It will
have modest positive effects on her weight and lead to a reduction in HbA1c of between 0.7
and 1% (up to 11 mmol/mol). It has been proven in the UKPDS trial to impact on major adverse
cardiovascular events (MACE).

A Acarbose

Acarbose has a more modest effect on HbA1c than metformin, and many patients find
adverse effects of therapy, such as diarrhoea, flatulence and abdominal bloating, difficult to
deal with.

B Gliclazide

Gliclazide has more potent short-term effects in reducing HbA1c, but benefit is often lost with
respect to glucose control over as little as 18 months. Gliclazide can also drive increased
appetite and lead to weight gain, which is unhelpful in this situation.

D Pioglitazone
Pioglitazone is a peroxisome proliferator-activated receptor (PPAR)-gamma agonist, meaning
that it drives accumulation of subcutaneous fat. While it is highly effective in lowering blood
glucose levels, it is not an optimal choice here because it drives weight gain and has negative
effects on bone mineral density. It does have modest positive effects on MACE, at the
expense of an increase in heart failure cases.

E Sitagliptin

Sitagliptin is a dipeptidyl peptidase 4 (DPP IV) inhibitor. While it does not have positive
effects on cardiovascular outcomes, it has not been shown to have a negative impact on
MACE endpoints. As such, it is a reasonable choice in patients for whom metformin is
inappropriate or not tolerated.
51616

Rate this question:

Next Question

Previous Question Tag Question

Feedback End Review

Difficulty: Average

Peer Responses %

Q. Answered Flagged

Q1

Q2

Q3

Q4

Q5

Q6

Q7
Q. Answered Flagged

Q8
0:00:43/03:00:00

A 48-year-old man is reviewed in the Gastroenterology Clinic some two weeks after
admission with an upper gastrointestinal haemorrhage.
At endoscopy, he was found to have a suspicious area in the gastric antrum which bled on
contact with the endoscopy probe. A biopsy taken from near to the area of contact bleeding
has revealed a gastric mucosa-associated lymphoid tissue lymphoma (MALToma). He is
Helicobacter pylori-positive.
Which of the following is the most appropriate initial intervention?

A H. pylori eradication therapy

B Imatinib

C Local radiotherapy

D Rituximab

E Surgical resection

Explanation 

A H. pylori eradication therapy

Most gastric MALTomas are very slow-growing, remaining localised for many years, and a
significant percentage regress with H. pylori eradication – up to 80% may enter remission.

B Imatinib

Platelet-derived growth factor receptor antagonists, such as imatinib, may be effective in


treating patients with gastric MALToma which is resistant to other interventions.

C Local radiotherapy

Local radiotherapy is usually deployed in conjunction with an anti-CD20 agent, such as


rituximab, for patients who have not responded to medical therapy alone.

D Rituximab

Rituximab is considered in combination with radiotherapy for locally advanced disease, and in
combination with chemotherapy for disseminated gastric MALToma.
E Surgical resection

Surgical resection is not associated with better outcomes, compared to radiotherapy or


medical therapy, and can lead to significant morbidity.
51630

Rate this question:

Next Question

Previous Question Tag Question

Feedback End Review

Difficulty: Average

Peer Responses %

Q. Answered Flagged

Q1

Q2

Q3

Q4

Q5

Q6

Q7

Q8

Q9

0:00:43/03:00:00

A 23-year-old man comes to the clinic with lower back pain for the last four months. He tells
you the pain is improved by exercise and not improved by rest. It was initially improved by
ibuprofen, although this is no longer effective. He also suffers from intermittent episodes of
diarrhoea, which his general practitioner (GP) tells him is irritable bowel syndrome.
Physical examination reveals limited forward and lateral flexion of the spine.
Which of the features seen here is most suggestive of ankylosing spondylitis (AS)?

A Age 23

B Male sex

C Back pain improved by exercise, and not by rest

D Episodes of diarrhoea

E Improvement due to ibuprofen

Explanation 

C Back pain improved by exercise, and not by rest

It is recommended that the modified New York criteria are used for the diagnosis of AS.

Clinical criteria:

low back pain, for more than three months; improved by exercise, not relieved by rest
limitation of lumbar spine motion in both the sagittal and frontal planes
limitation of chest expansion relative to normal values for age and sex.

Radiological criterion: sacroiliitis on X-ray.

Definite AS is diagnosed if the radiological criterion is present plus at least one clinical
criterion, and probable AS if three clinical criteria are present or if the radiological criterion is
present but no clinical criteria are present. There is a strong association with HLA-B27, with 1–
2% of patients who are HLA-B27-positive developing the condition. This rises by a factor of 10
in individuals who also have one affected relative. Other important features of AS include
aortic regurgitation, anterior uveitis and pulmonary fibrosis. Peripheral enthesitis is also seen.
Initial treatment is with non-steroidal anti-inflammatory drugs (NSAIDs). If there is treatment
failure after two NSAIDs, a move to tumour necrosis factor (TNF) modulator therapy is
recommended.

A Age 23
Although AS is more likely to present at a younger age, it does not form part of the New York
criteria for diagnosis.

B Male sex

The male : female ratio of symptomatic AS is 3 : 1. Women usually have milder or subclinical
disease. Gender does not form part of the diagnostic criteria.

D Episodes of diarrhoea

Although AS may be associated with inflammatory bowel disease, symptoms of inflammatory


bowel disease do not contribute to confirming the diagnosis.

E Improvement due to ibuprofen

Response to NSAIDs is used to stage therapy, rather than to make the diagnosis. Failure to
control symptoms with two different NSAIDs prompts progression to an anti-TNF agent.
51669

Rate this question:

Next Question

Previous Question Tag Question

Feedback End Review

Difficulty: Average

Peer Responses %

Q. Answered Flagged

Q1

Q2

Q3
Q. Answered Flagged

Q4

Q5

Q6

Q7

Q8
0:00:43/03:00:00

A 28-year-old woman presents to the Rheumatology Clinic with a 6-week history of joint
pain, particularly affecting her hands, wrists and elbows.
Examination reveals no skin rashes, normal nails with no pitting and small joint polyarthritis,
predominantly affecting the proximal interphalangeal and metacarpophalangeal joints. The
distal interphalangeal joints are spared.
Which of the following is the most likely diagnosis?

A Osteoarthritis

B Psoriatic arthritis

C Reactive arthritis

D Rheumatoid arthritis

E Systemic lupus erythematosus

Explanation 

D Rheumatoid arthritis

The picture here with small joint polyarthritis, with sparing of the distal interphalangeal joints,
fits with a diagnosis of rheumatoid arthritis. Anti-cyclic citrullinated peptide, rheumatoid
factor testing and X-ray of the hands and feet are indicated as initial investigations. Initial
therapy is with a disease-modifying anti-rheumatic drug such as methotrexate; biologicals
used in the management of the condition can include anti-CD20 monoclonal antibodies,
tumour necrosis factor-modulating monoclonal antibodies and agents targeting interleukin-6,
among others.

A Osteoarthritis

The joint distribution is unusual for osteoarthritis, given the age of the patient. Osteoarthritis
of the hands is more likely to be seen in elderly patients who have held occupations which led
to repetitive use of the hands and fingers, such as working in a typing pool, which may have
driven the development of wear-and-tear arthritis.

B Psoriatic arthritis

Features suggesting psoriatic arthritis, such as the typical skin rash of psoriasis or nail pitting,
are absent here. However, it is important to remember that psoriatic arthritis can masquerade
as rheumatoid arthritis.
C Reactive arthritis

There are no triggers here to suggest reactive arthritis such as a recent gastrointestinal or
sexually transmitted infection.

E Systemic lupus erythematosus

Systemic lupus erythematosus is associated with a photosensitive skin rash, which is absent
here.
51674

Rate this question:

Next Question

Previous Question Tag Question

Feedback End Review

Difficulty: Average

Peer Responses %

Q. Answered Flagged

Q1

Q2

Q3

Q4

Q5

Q6

Q7

Q8

Q9

0:00:43/03:00:00

You are reviewing the results of a new drug to normalise gastrointestinal (GI) motility in
patients suffering from irritable bowel syndrome (IBS). You have collected transit times in
both the treatment and control groups, which have been measured using a smart pill.
Which of the following tests is most appropriate to determine differences in mean transit
time between the groups?

A Analysis of variance (ANOVA) test

B Kaplan–Meier estimator

C Kruskal–Wallis test

D t-test

E Wilcoxon rank sum test

Explanation 

D t-test

This test is designed to test if there is a difference between the means of two data sets where
the data are normally distributed. Although the new drug may have shortened or lengthened
GI motility in the active group, the transit times should still be normally distributed, meaning
the t-test is appropriate.

A Analysis of variance (ANOVA) test

The ANOVA test is commonly used in animal experiments to determine whether there are
differences between the means of a variable across three or more independent groups.

B Kaplan–Meier estimator

This is used for estimating differences in survival curves, eg those generated for time to first
cardiovascular event in an outcome study.

C Kruskal–Wallis test

The Kruskal–Wallis test is used commonly as a method of analysis for data where there is one
nominal variable and one measurement variable, and the data are not normally distributed, so
the one-way ANOVA cannot be used.
E Wilcoxon rank sum test

This is also known as the Mann–Whitney U test and is used to test if the means of two
independent samples of data are similar where the data are not normally distributed.
51608

Rate this question:

Next Question

Previous Question Tag Question

Feedback End Review

Difficulty: Average

Peer Responses %

Q. Answered Flagged

Q1

Q2

Q3

Q4

Q5

Q6

Q7

Q8

Q9

0:00:43/03:00:00

A 29-year-old builder, who has been ill over a few days with flu-like symptoms, comes to the
Emergency Department with dull central chest pain which is aggravated by inspiration,
coughing and lying flat and is relieved by sitting forward.
He has a non-productive cough. His blood pressure is 112/82 mmHg; his pulse is 82 bpm and
regular. You can hear a rub at the left sternal edge on auscultation.
Which of the following is most likely to be found on an electrocardiogram (ECG)?

A Bifid P waves

B Concave ST elevation across leads I, II, III, aVL, aVF and V2–6

C Concave ST elevation in leads aVR and V1

D Convex ST elevation in leads II, III and aVF

E T wave inversion in leads V1–V3

Explanation 

B Concave ST elevation across leads I, II, III, aVL, aVF and V2–6

This patient’s symptoms are consistent with acute pericarditis, which may be related to a
number of viral infections, including Coxsackie virus, echovirus, Epstein–Barr virus and
influenza. The classical ECG appearance is concave ST elevation across all leads, apart from
leads V1 and aVR where ST depression is seen. Troponin may be increased but is not an
indicator of myocardial ischaemia in this context.

A Bifid P waves

Widened, bifid P waves are seen in patients with left atrial enlargement related to mitral
stenosis, not with this clinical scenario.

C Concave ST elevation in leads aVR and V1

In leads aVR and V1, reciprocal ST depression, rather than ST elevation, is seen in patients
with pericarditis.

D Convex ST elevation in leads II, III and aVF


This is an indicator of myocardial ischaemia affecting the inferior territory and should prompt
initiation of treatments for acute coronary syndrome.

E T wave inversion in leads V1–V3

The limited territory affected by the T wave changes is more suggestive of antero-septal
myocardial ischaemia, rather than pericarditis related to a viral infection.
51578

Rate this question:

Next Question

Previous Question Tag Question

Feedback End Review

Difficulty: Average

Peer Responses %

Q. Answered Flagged

Q1

Q2

Q3

Q4

Q5

Q6

Q7

Q8

Q9

0:00:43/03:00:00

A 54-year-old man with known ischaemic heart disease, type 2 diabetes and psoriasis comes
to his 6-week appointment, having suffered an inferior myocardial infarction, for which he
underwent two stents. He is now taking a range of medications, including ramipril, aspirin,
ticagrelor, bisoprolol and atorvastatin.
His blood pressure is 115/75 mmHg and his pulse is 55 bpm and regular. His heart sounds are
normal. His chest is clear, and there is no significant ankle swelling. You note that he has
suffered a significant flare of his psoriasis.
Which of the following is the most likely cause of the worsening psoriasis?

A Aspirin

B Atorvastatin

C Bisoprolol

D Ticagrelor

E Ramipril

Explanation 

C Bisoprolol

Beta-blockers are recognised to worsen psoriasis in those with pre-existing disease, and it is
most likely that this patient was started on bisoprolol after his recent myocardial infarction. If
his skin disease is so severe that he cannot tolerate bisoprolol, substitution for another
medication known to slow the heart rate which has positive outcomes data, such as
ivabradine, is likely to be the most appropriate option.

A Aspirin

Aspirin is associated with urticarial rashes. There is no proven association with aspirin use
and the development of psoriasis.

B Atorvastatin

Atorvastatin may reduce inflammation and have a positive effect on psoriasis. It is associated
with allergic reactions which may result in urticaria, erythema multiforme and Stevens–
Johnson syndrome.
D Ticagrelor

Like aspirin and atorvastatin, ticagrelor is associated with drug-induced hypersensitivity-


related skin reactions, although it is not associated with worsening of psoriasis.

E Ramipril

Ramipril is associated with angio-oedema, exfoliative dermatitis, Stevens–Johnson syndrome


and erythema multiforme, but not with worsening of psoriasis.
51591

Rate this question:

Next Question

Previous Question Tag Question

Feedback End Review

Difficulty: Average

Peer Responses %

Q. Answered Flagged

Q1

Q2

Q3

Q4

Q5

Q6

Q7

Q8

Q9

 External Links

Electronic Medicines Compendium. 2017. Bisoprolol fumarate 10 mg film-coated tablets: undesirable effec…
medicines.org.uk/emc/product/8247/smpc#UNDESIRABLE_EFFECTS
(https://www.medicines.org.uk/emc/product/8247/smpc#UNDESIRABLE_EFFECTS)
0:00:43/03:00:00

A 54-year-old woman, who had Hashimoto’s thyroiditis diagnosed some ten years ago,
comes to the clinic for review. She has noticed that the right side of her goitre has begun
increasing in size over the past few months.
Examination reveals a palpable 5 cm mass affecting the right-hand side of the thyroid. It is
difficult to determine the margins vs the surrounding goitre. Thyroxine (T4), triiodothyronine
(T3) and thyroid-stimulating hormone (TSH) levels are in the normal range, as is serum
calcium level.
Which of the following is the most likely diagnosis?

A Anaplastic thyroid carcinoma

B Medullary cell carcinoma of the thyroid

C Multinodular goitre

D Thyroid adenoma

E Thyroid lymphoma

Explanation 

E Thyroid lymphoma

Among patients with Hashimoto’s thyroiditis, the risk of developing thyroid lymphoma is
some 60 times that of the general population, and around half of patients presenting with
thyroid lymphoma have a background of Hashimoto’s disease. In this case, a rapidly enlarging
mass with blurred margins to the surrounding goitre is entirely consistent with a lymphoma.
Ultrasound of the gland followed by needle or excision biopsy are the most appropriate next
steps.

A Anaplastic thyroid carcinoma

Anaplastic carcinoma presents with a very rapidly enlarging, hard thyroid mass, which is
locally infiltrating. It occurs in the elderly population and is not responsive to chemo- or
radiotherapy.

B Medullary cell carcinoma of the thyroid

This is a carcinoma of thyroid C cells, responsible for the production of calcitonin. It is most
often familial or occurs in patients with multiple endocrine neoplasia (MEN)-2 syndrome.
C Multinodular goitre

Unilateral thyroid enlargement against a background of Hashimoto’s thyroiditis is much more


likely to be due to an underlying lymphoma than multinodular goitre.

D Thyroid adenoma

Thyroid adenomas are associated with a discrete thyroid swelling, which is well demarcated
from the surrounding thyroid tissue. They are not more likely to occur against a background
of Hashimoto’s disease.
51615

Rate this question:

Next Question

Previous Question Tag Question

Feedback End Review

Difficulty: Average

Peer Responses %

Q. Answered Flagged

Q1

Q2

Q3

Q4

Q5

Q6

Q7

Q8



Q. Answered Flagged
0:00:43/03:00:00

A 20-year-old student returns from a trip to Africa with problems passing urine. He tells you
that he suffers from dull suprapubic pain and has pain on passing urine.
He has haematuria present at the end of passing urine, and you note blood and proteinuria
on dipstick testing. Urine examination is positive for Schistosoma haematobium.
Which of the following is the most appropriate intervention?

A Doxycycline

B Mebendazole

C Metronidazole

D Praziquantel

E Tinidazole

Explanation 

D Praziquantel

Praziquantel is thought to increase membrane permeability of schistosomal cells to calcium.


This leads to the paralysis of worms, so that they are then destroyed by the host immune
response and eliminated. It is thought to be the most effective option for the treatment of
urinary schistosomiasis. It is the terminal haematuria here which drives us towards urinary
schistosomiasis as the diagnosis. Hepatic schistosomiasis is associated with right upper
quadrant pain and progressive hepatic dysfunction. Some evidence now suggests that
albendazole may be an effective option in patients who have disease which is not responsive
to praziquantel.

A Doxycycline

Doxycycline is used for malaria prophylaxis, particularly when Plasmodium falciparum, which
is resistant to chloroquine, is present.

B Mebendazole

Mebendazole is a commonly used over-the-counter treatment for threadworms. It may be


used for prophylaxis against schistosomiasis infection but is not usually deployed as a
therapy.
C Metronidazole

A course of metronidazole is usually given for amoebic dysentery and may also be given for
the treatment of giardiasis.

E Tinidazole

Tinidazole used as a single dose is the intervention of choice for giardiasis, a protozoal
infection which presents with symptoms similar to irritable bowel syndrome.
51634

Rate this question:

Next Question

Previous Question Tag Question

Feedback End Review

Difficulty: Average

Peer Responses %

Q. Answered Flagged

Q1

Q2

Q3

Q4

Q5

Q6

Q7

Q8

Q9

 External Links

Ben, S A, Useh, M F. (2017). comparative study on the efficacy of praziquantel and albendazole in the trea…
ncbi.nlm.nih.gov/pubmed/28911129
(https://www.ncbi.nlm.nih.gov/pubmed/28911129)
0:00:43/03:00:00

A 54-year-old man is brought to the Emergency Department by the police. He has been
found wondering the streets in an unkempt state. According to a neighbour, his wife recently
passed away, and he has been unable to return to work, meaning that he has lost his job. He
tells you that his mind no longer exists and he is slowly dying, with parts of his body shutting
down each day. He has not eaten for days and sleeps only two hours per night.
Which of the following is the most likely diagnosis?

A Alcohol withdrawal

B Depression

C Drug-induced psychosis

D Frontal lobe dementia

E Schizophrenia

Explanation 

B Depression

This patient has suffered two significant life events and has several features of depression,
including disturbed sleep, poor appetite and nihilistic delusions. He does not express suicidal
ideation. There is potentially an opportunity to begin antidepressant treatment, in
conjunction with high-intensity psychological therapy, although if the situation worsens, he
may require electroconvulsive therapy.

A Alcohol withdrawal

Alcohol withdrawal is associated with agitation, hallucinations, including tactile ones which
often take the form of insects crawling over the skin, and seizures. Treatment is primarily with
chlordiazepoxide or other benzodiazepines.

C Drug-induced psychosis

This can result in hallucinations, delusions and features of depression, although in a man with
the profile described here, depression is more likely to be the underlying cause.

D Frontal lobe dementia


Frontal lobe dementia is associated with chronic behavioural changes, often manifest in
situations where their actions would be deemed inappropriate, such as laughing at bad news
or sexually inappropriate behaviour towards colleagues.

E Schizophrenia

A first presentation with schizophrenia would be unusual at the age of 54, and the life event
triggers here are more in keeping with depression as the likely underlying diagnosis.
51654

Rate this question:

Next Question

Previous Question Tag Question

Feedback End Review

Difficulty: Average

Peer Responses %

Q. Answered Flagged

Q1

Q2

Q3

Q4

Q5

Q6

Q7

Q8

Q9

0:00:43/03:00:00

A 64-year-old man who has type 2 diabetes, obesity and osteoarthritis of both knees
presents to the Emergency Department with sudden onset of severe pain in his right upper
calf. You note that the skin is tight over the upper calf and there is a fluctuant cyst-like
swelling behind the right knee on the medial side.
Which of the following is the most likely diagnosis?

A Deep vein thrombosis (DVT)

B Gastrocnemius muscle tear

C Lipoma

D Ruptured popliteal cyst

E Superficial thrombophlebitis

Explanation 

D Ruptured popliteal cyst

The swelling is the result of synovial fluid distending the gastrocnemio-semimembranosus


bursa. The fluid drives local inflammation, which accounts for the significant pain reported
here. A DVT must be excluded, and physiotherapy and analgesia are the mainstay of therapy.

A Deep vein thrombosis (DVT)

The sudden onset of pain and the fact the main area of swelling is behind the knee suggest a
ruptured popliteal cyst is the more likely diagnosis. An ultrasound should be performed,
however, to rule out a DVT.

B Gastrocnemius muscle tear

A calf muscle tear would be more likely in a patient complaining of sudden-onset pain more
distally, particularly on plantar flexion and dorsiflexion of the foot.

C Lipoma

A lipoma may present as a posterior swelling, although it would not usually be associated
with the sudden onset of severe pain, as seen here.
E Superficial thrombophlebitis

Superficial thrombophlebitis presents with a palpable thrombus within superficial veins in the
lower limb, rather than the picture seen here with a cyst-like swelling.
51673

Rate this question:

Next Question

Previous Question Tag Question

Feedback End Review

Difficulty: Average

Peer Responses %

Q. Answered Flagged

Q1

Q2

Q3

Q4

Q5

Q6

Q7

Q8

Q9

0:00:43/03:00:00

A 38-year-old woman is reviewed in the Rheumatology Clinic. She has systemic lupus
erythematosus (SLE) which is currently managed with oral hydroxychloroquine.
Her joint pains have significantly improved, although there has been a gradual worsening in
her serum creatinine levels over the past few months, from 115 µmol/l to 175 µmol/l.
Which of the following is the most appropriate way to slow renal progression in this
patient?

A Azathioprine

B Cyclophosphamide

C Increased dose of hydroxychloroquine

D Methotrexate

E Mycophenolate mofetil

Explanation 

E Mycophenolate mofetil

Mycophenolate mofetil is as effective as cyclophosphamide in achieving remission in lupus


nephritis (the most likely cause of the worsening creatinine seen here) and is safer with a
lower risk of ovarian failure.

A Azathioprine

Azathioprine is less effective than both azathioprine and cyclophosphamide in achieving


remission of lupus nephritis. It is therefore considered a second- or third-line agent for
patients where the other options are unsuitable.

B Cyclophosphamide

Cyclophosphamide is usually reserved for life-threatening renal vasculitis, rather than the
gradual deterioration in renal function seen here.

C Increased dose of hydroxychloroquine

Hydroxychloroquine is not effective vs other options, such as cyclophosphamide or


mycophenolate, in slowing renal progression.
D Methotrexate

Methotrexate is primarily an option for managing joint disease in rheumatoid arthritis and
psoriatic arthritis, rather than renal progression in SLE.
51663

Rate this question:

Next Question

Previous Question Tag Question

Feedback End Review

Difficulty: Average

Peer Responses %

Q. Answered Flagged

Q1

Q2

Q3

Q4

Q5

Q6

Q7

Q8

Q9

0:00:43/03:00:00

A 42-year-old woman presents to the clinic with an erythematous rash, which worsens on sun
exposure. When it occurs on the scalp, there is scarring hair loss, and areas of skin that have
previously been affected heal with areas of atrophy. Atrophic skin is most prominent over her
elbows. The lesions are only rarely itchy.
Routine bloods, including complement and inflammatory markers, are unremarkable.
Which of the following is the most likely diagnosis?

A Actinic keratosis

B Discoid lupus

C Lichen planus

D Porphyria cutanea tarda

E Systemic lupus erythematosus

Explanation 

B Discoid lupus

The scenario here of a scarring rash affecting sun-exposed areas of skin fits with cutaneous
lupus. The presence of normal complement and inflammatory markers fits with the absence
of systemic lupus. Sun exposure should be minimised, with topical steroids an appropriate
option for limited skin disease. For more extensive discoid lupus, hydroxychloroquine is the
next step.

A Actinic keratosis

Actinic keratosis usually begins as a small erythematous, scaly plaque on the face but can be
seen on any sun-exposed area. It is related to chronic sun exposure and usually occurs in
patients older than 42 years.

C Lichen planus

The onset of lichen planus is usually acute, and it is typically associated with intensely itchy
2–5 mm, red or violet, shiny, flat-topped papule with white streaks (known as Wickham’s
striae).

D Porphyria cutanea tarda


This is a blistering, erythematous rash which is associated with sun exposure. It is most
commonly seen in patients who have a history of alcohol abuse.

E Systemic lupus erythematosus

Although systemic lupus erythematosus fits with the skin rash seen here, the normal
complement and inflammatory markers makes systemic disease much less likely. Other
clinical features of systemic disease, such as joint pain and renal involvement, are also absent
here.
51611

Rate this question:

Next Question

Previous Question Tag Question

Feedback End Review

Difficulty: Average

Peer Responses %

Q. Answered Flagged

Q1

Q2

Q3

Q4

Q5

Q6

Q7

Q8

Q9

0:00:43/03:00:00

A 29-year-old man comes to the Respiratory Clinic for review. He has suffered from night
sweats and weight loss over the past three months and has now developed dull right-sided
chest pain. He attended the Emergency Department where a diagnostic tap revealed
lymphocytosis, with no growth. Polymerase chain reaction (PCR) is negative for tuberculosis.
A computed tomography (CT) scan of the thorax reveals hilar lymphadenopathy, an area of
right-sided pleural thickening and a right-sided pleural effusion.
Investigations:

Haemoglobin (Hb) 11.5 g/dl (13.5–17.5 g/dl)

White cell count (WCC) 8.9 × 10 9/l (4–11 × 10 9/l)

Platelets (PLT) 231 × 10 9/l (150–400 × 10 9/l)

Sodium (Na +) 140 mmol/l (135–145 mmol/l)

Potassium (K +) 4.5 mmol/l (3.5–5.0 mmol/l)

Creatinine (Cr) 95 µmol/l (5–120 mmol/l)

Erythrocyte sedimentation rate (ESR) 75 mm (0–10 mm in first hour)

Which of the following is the most useful next investigation?

A Bone marrow aspiration

B Gallium scanning

C Immunophenotyping of peripheral blood

D Mediastinoscopy and lymph node biopsy

E Pleural biopsy

Explanation 

E Pleural biopsy

The suspicion here is of an underlying lymphoma, given the patient’s age, symptoms over the
past few months and changes seen on CT thorax. Pleural biopsy has the advantage of
yielding information on the architecture of lymphoid tissue associated with abnormal cells
and also of providing cells which can be used for immunophenotyping. Both of these pieces
of information can aid targeting of therapy.
A Bone marrow aspiration

Apart from mild anaemia and elevated ESR, there is little evidence of changes on the
peripheral blood picture. Bone marrow aspiration is invasive and may miss the primary
diagnosis. It is therefore not the investigation of choice here.

B Gallium scanning

Gallium scanning is considered for evaluation of lymph node involvement for staging of
disease. At this stage, making the diagnosis with a pleural biopsy of the affected area is the
most appropriate next step.

C Immunophenotyping of peripheral blood

Pleural biopsy is more appropriate here because it will not only yield cells for
immunophenotyping, but it may also give information as to how the lymphocytes are
organised in tissue, both of which stratify intervention and prognosis.

D Mediastinoscopy and lymph node biopsy

This is much more invasive, compared to CT-guided pleural biopsy, and will not yield much
more with respect to diagnostic information at this stage.
51587

Rate this question:

Next Question

Previous Question Tag Question

Feedback End Review

Difficulty: Average

Peer Responses %

Q. Answered Flagged
Q. Answered Flagged

Q1

Q2

Q3

Q4

Q5

Q6

Q7

Q8
0:00:43/03:00:00

A 32-year-old woman is admitted to the Emergency Department, having suffered a left-sided


stroke. She returned from a long-haul flight to Australia the previous day and had been
complaining of pain in her left lower limb. She has no past medical history of note, and her
only medication is the combined oral contraceptive pill.
She is distressed with obvious weakness of the right side of the face and right arm, coupled
with speech disturbance. You note a swollen left lower limb, consistent with a deep vein
thrombosis (DVT). Her blood pressure is 155/95 mmHg; her pulse is 90 bpm and regular. Her
oxygen (O 2) saturation is only 91% on air. A computed tomography (CT) scan of the head
reveals no signs of haemorrhage.
Which of the following is the investigation of choice to confirm the cause of her stroke?

A Anti-phospholipid antibody screening

B Carotid ultrasound scan

C Bubble contrast echocardiography (echo)

D Factor V Leiden screening

E 12-lead electrocardiogram (ECG)

Explanation 

C Bubble contrast echocardiography (echo)

In a patient with a likely DVT, low O 2 saturation, with a possible pulmonary embolism (PE),
and symptoms of an acute stroke, paradoxical embolism as a cause of the stroke should be
considered. This occurs due to an undiagnosed patent foramen ovale (PFO), which can be
identified on contrast echo. PFOs are found in up to 35% of an unselected series of post-
mortems.

A Anti-phospholipid antibody screening

Anti-phospholipid antibody syndrome is a cause of venous thromboembolism but would not


explain the coexistence of a DVT, PE and an acute stroke in this patient.

B Carotid ultrasound scan

This would be an investigation of choice if there was no evidence of venous


thromboembolism seen in this scenario. In this situation, a contrast Echo is more useful.
D Factor V Leiden screening

It is possible for a factor V Leiden mutation to coexist in this patient and increase the risk of
thromboembolism, although it would not explain the coexistence of disease on both the
venous and arterial sides of the circulation.

E 12-lead electrocardiogram (ECG)

A 12-lead ECG may be normal in PFOs, and hence it is not a reliable screening test here.
Abnormalities which may be seen on the ECG include right axis deviation and T wave
inversion in leads V1 to V3.
51577

Rate this question:

Next Question

Previous Question Tag Question

Feedback End Review

Difficulty: Average

Peer Responses %

Q. Answered Flagged

Q1

Q2

Q3

Q4

Q5

Q6

Q7

Q8
Q. Answered Flagged

0:00:43/03:00:00

A 45-year-old South-Asian man, who is being treated for tuberculosis, comes to the
Emergency Department. He is in the fourth week of quadruple therapy and complains that he
has become colour-blind. His tuberculosis treatment consists of ethambutol, rifampicin,
isoniazid and pyrazinamide. He also takes sildenafil for erectile dysfunction.
Which of the following is the likely cause of his visual disturbance?

A Ethambutol

B Isoniazid

C Pyrazinamide

D Rifampicin

E Sildenafil

Explanation 

A Ethambutol

Ethambutol is the most commonly implicated anti-tuberculous drug, with respect to ocular
toxicity. It is known to cause neuritis, which is most commonly associated with a central
scotoma, and red-green colour blindness may also result. Visual changes should prompt
discontinuation of therapy, and they often completely resolve over time.

B Isoniazid

It is associated with optic neuritis, but this is much rarer, compared with ethambutol.

C Pyrazinamide

Abnormal liver function tests or joint pain are the most likely side-effect of pyrazinamide
therapy.

D Rifampicin

Rifampicin leads to yellow/orange/red discoloration of body secretions. For this reason,


contact lens wearers should be warned at initiation of therapy that this is a possibility.

E Sildenafil
Sildenafil at high dose is known to cause blue discoloration of vision because of activation of
phosphodiesterase type 6 (PDE-6) receptors, located in rod and cone cells on the retina.
51594

Rate this question:

Next Question

Previous Question Tag Question

Feedback End Review

Difficulty: Average

Peer Responses %

Q. Answered Flagged

Q1

Q2

Q3

Q4

Q5

Q6

Q7

Q8

Q9

 External Links

Electronic Medicines Compendium. 2017. Ethambutol 400 mg tablets.


medicines.org.uk/emc/product/5311/smpc
(https://www.medicines.org.uk/emc/product/5311/smpc)
0:00:43/03:00:00

A 22-year-old nurse is reviewed in the Allergy Clinic, following an emergency admission for
anaphylaxis after a dental procedure. She has had intermittent skin rashes related to medical
gloves and switched to nitrile gloves a few years earlier. Examination in the clinic is
unremarkable. You suspect latex allergy.
Which of the following is the most appropriate way to make the diagnosis?

A Cross-reactivity testing with trigger foods

B Mast cell tryptase during an attack

C Provocation testing

D Radioallergosorbent (RAST) testing

E Skin-prick testing

Explanation 

E Skin-prick testing

Skin-prick testing with extracts of Hevea latex B and C serum protein is a safe activity, as long
as the extracts are standardised for concentration and stability. The results of skin-prick
testing are composed of a positive control (histamine) and a negative control (saline), and
the test is given in escalating concentrations, beginning at 0.001 mg/ml.

A Cross-reactivity testing with trigger foods

Reaction to food in latex fruit syndrome is unpredictable and could result in a severe
anaphylactic reaction. It is not advised in this situation.

B Mast cell tryptase during an attack

Mast cell tryptase may be elevated during an attack, although it would not specifically
indicate a relationship to latex exposure.

C Provocation testing

Inhalational challenges, exposure to gloves, and nasal provocation have all been trialled. It is,
however, extremely difficult to predict antigen exposure, and provocation testing risks
provoking anaphylaxis.
D Radioallergosorbent (RAST) testing

Serology testing is an alternative to skin-prick testing when it is unavailable or patients


decline skin-prick testing as an initial option. It may help identify patients who are sensitised
to latex but are asymptomatic.
51609

Rate this question:

Next Question

Previous Question Tag Question

Feedback End Review

Difficulty: Average

Peer Responses %

Q. Answered Flagged

Q1

Q2

Q3

Q4

Q5

Q6

Q7

Q8

Q9

0:00:43/03:00:00

A 40-year-old man is referred to the Gastroenterology Clinic for review. He presented to his
general practitioner (GP) with increasing tiredness a few weeks earlier and was found to have
elevated transaminase and alkaline phosphatase levels.
Alpha-1 antitrypsin deficiency (AATD) is suspected. He is a non-smoker, who drinks ten units
of alcohol per week. Peak flow is only mildly reduced.
Which genotype is he most likely to have?

A MM

B SZ

C MZ

D II

E ZZ

Explanation 

C MZ

The M gene is the normal one with respect to alpha-1 antitrypsin production. Patients who are
heterozygotes carrying one Z allele are thought to be more prone to AATD-related chronic
liver disease, especially against a background of mild alcohol consumption. Clinical lung
disease associated with AATD only usually becomes apparent in smokers.

A MM

This represents two normal genes associated with alpha-1 antitrypsin production, meaning
that levels of alpha-1 antitrypsin in these patients are normal and they are not prone to liver
or lung disease.

B SZ

The SZ genotype is a compound heterozygote for AATD and is therefore more likely to
develop both emphysema (even in the absence of smoking) and cirrhotic liver disease than
the MZ genotype.

D II
This is one of the homozygotic forms of AATD, which is associated with an increased risk of
clinical lung disease. It does not fit with the clinical picture we see here.

E ZZ

Like II, the ZZ genotype puts patients at significant risk of lung disease because of more
significant AATD.
51624

Rate this question:

Next Question

Previous Question Tag Question

Feedback End Review

Difficulty: Average

Peer Responses %

Q. Answered Flagged

Q1

Q2

Q3

Q4

Q5

Q6

Q7

Q8

Q9

 External Links

Alpha One Registry


alphaoneregistry.org/faqs/faqs_genetic_counseling
(https://alphaoneregistry.org/faqs/faqs_genetic_counseling)
0:00:43/03:00:00

A 28-year-old man, who has been working in a bar in Spain, comes to the Sexually
Transmitted Diseases Clinic on his return to the United Kingdom. He was treated there for
gonorrhoea with a single dose of azithromycin. Unfortunately, nucleic acid amplification
testing has suggested he is still infected.
Which of the following antibiotics should he have also received?

A Ceftriaxone

B Ciprofloxacin

C Doxycycline

D Penicillin G

E Rifampicin

Explanation 

A Ceftriaxone

Standard treatment for uncomplicated gonococcal infection is ceftriaxone 500 mg


intramuscularly (IM), with 1 g azithromycin orally, given as a single dose. Only in patients with
severe penicillin allergy, or who are allergic to cephalosporin, should azithromycin alone be
considered, when a higher dose of 2 g is usually given.

B Ciprofloxacin

Quinolones are no longer recommended as first-line therapy for gonococcal infection


because of the risk of resistance. Ciprofloxacin or ofloxacin may be an option for patients
where sensitivities are established, and the patient is penicillin- or cephalosporin-allergic.

C Doxycycline

Doxycycline is usually given as a 14-day course for patients who have gonococcal epididymo-
orchitis. This is following a single dose of ceftriaxone IM.

D Penicillin G

A single IM injection of penicillin G is an option for the treatment of syphilis; it is not usually
recommended in combination for the treatment of gonorrhoea.
E Rifampicin

Rifampicin has been trialled as a single-dose treatment for gonorrhoea as early as 1968, when
a treatment failure rate of 10% was already identified then. Therefore, it is not recommended
here.
51632

Rate this question:

Next Question

Previous Question Tag Question

Feedback End Review

Difficulty: Average

Peer Responses %

Q. Answered Flagged

Q1

Q2

Q3

Q4

Q5

Q6

Q7

Q8

Q9

0:00:43/03:00:00

A 35-year-old man is admitted to the Emergency Department with rapidly worsening nausea
and vomiting over the past few days, accompanied by haemoptysis. He smokes 20 cigarettes
per day and uses a salbutamol inhaler but is otherwise well.
Examination reveals a blood pressure of 178/100 mmHg, and his pulse is 78 bpm and regular.
There are scattered crackles on auscultation of the chest.
Investigations:

Haemoglobin (Hb) 9.7 g/dl (13.5–17.5 g/dl)

White cell count (WCC) 12.2 × 10 9/l (4–11 × 10 9/l)

Platelets (PLT) 321 × 10 9/l (150–400 × 10 9/l)

Sodium (Na +) 143 mmol/l (135–145 mmol/l)

Potassium (K +) 5.8 mmol/l (3.5–5.0 mmol/l)

Creatinine (Cr) 720 µmol/l (50–120 µmol/l)

Anti-glomerular basement
Positive
membrane (GBM) antibody

Chest X-ray Patchy infiltration

She is treated with intravenous (IV) methylprednisolone.

Which immunosuppressant agent should be used alongside this?

A Adalimumab

B Azathioprine

C Cyclophosphamide

D Golimumab

E Methotrexate

Explanation 

C Cyclophosphamide
Cyclophosphamide, alongside methylprednisolone, is the initial immunosuppressant of choice
for the treatment of anti-GBM disease (Goodpasture syndrome). There is not thought to be a
difference in efficacy between oral and IV delivery of cyclophosphamide, although IV therapy
may be required, particularly where patients are vomiting. An initial oral dose of 2 mg/kg/day
of cyclophosphamide is recommended.

A Adalimumab

The development of anti-GBM disease has been reported in patients treated with anti-tumour
necrosis factor (TNF) therapies. Across the class of anti-TNF monoclonal antibodies (Mabs),
trials have been conflicting, with some smaller trials suggesting a benefit, whereas others
have shown no positive impact. Adalimumab is given as a once every two weeks
subcutaneous injection.

B Azathioprine

Azathioprine is a steroid-sparing therapy used over the longer term in patients with anti-GBM
disease, not as an induction agent for immunosuppression. Of some concern in patients
presenting with gout who are taking azathioprine is the interaction with allopurinol, which
can result in accumulation of 6-mercaptopurine, leading to marked immunosuppression.

D Golimumab

Golimumab is an anti-TNF-based therapy usually deployed in the treatment of psoriasis and


psoriatic arthritis. As such, it is not an appropriate first-line immunosuppressant here. It is
given as a once-monthly subcutaneous injection.

E Methotrexate

Methotrexate is the immunosuppressant of choice in rheumatoid arthritis and psoriatic


arthritis; it is not a first-line agent in the treatment of anti-GBM disease.
51649

Rate this question:

Next Question

Previous Question Tag Question

Feedback End Review

Difficulty: Average

Peer Responses %
Q. Answered Flagged

Q1

Q2

Q3

Q4

Q5

Q6

Q7

Q8

Q9

0:00:43/03:00:00

A 72-year-old man is admitted with confusion. He smokes 40 cigarettes per day and is
awaiting an appointment in the next few days for a hilar mass on his chest X-ray. According
to his relatives, over the past 72 hours, he has become muddled and forgetful and has now
taken to his bed.
On examination, his blood pressure is 135/82 mmHg and his pulse is 73 bpm and regular. He
has wheeze, consistent with chronic obstructive pulmonary disease (COPD), on auscultation
of his chest.
Investigations:

Haemoglobin (Hb) 11.0 g/dl (13.5–17.5 g/dl)

White cell count (WCC) 7.0 × 10 9/l (4–11 × 10 9/l)

Platelets (PLT) 203 × 10 9/l (150–400 × 10 9/l)

Sodium (Na +) 124 mmol/l (135–145 mmol/l)

Potassium (K +) 4.5 mmol/l (3.5–5.0 mmol/l)

Creatinine (Cr) 95 µmol/l (50–120 µmol/l)

Which of the following is the most likely cause of his lung mass?

A Biopsy of lung lesions

B Bronchial carcinoid

C Large cell carcinoma of the bronchus

D Small cell carcinoma of the bronchus

E Squamous cell carcinoma of the bronchus

Explanation 

D Small cell carcinoma of the bronchus

This patient is most likely to have syndrome of inappropriate antidiuretic hormone (SIADH)
related to an underlying bronchial carcinoma. Small cell carcinomas make up around 10–15%
of all lung cancers, and around 70% of all cases of SIADH related to malignancy is due to
small cell lung cancer. Fluid restriction is the initial intervention of choice.

A Biopsy of lung lesions


Adenocarcinomas of the bronchus are not normally hormone-producing. Several older case
reports attribute hormone production to adenocarcinoma, although these may represent
misclassifications, based on histology.

B Bronchial carcinoid

Bronchial carcinoid occurs in younger patients and is not associated with smoking. Tumours
are slow-growing, presenting often with obstructive symptoms, leading to pneumonia or
haemoptysis. They may produce antidiuretic hormone, as well as 5-hydroxyindoleacetic acid.

C Large cell carcinoma of the bronchus

Large cell carcinoma of the bronchus is not a hormone-secreting tumour.

E Squamous cell carcinoma of the bronchus

Squamous cell carcinoma is most commonly implicated in hypercalcaemia associated with


malignancy, because of the production of parathyroid-related hormone-related peptide.
51667

Rate this question:

Next Question

Previous Question Tag Question

Feedback End Review

Difficulty: Average

Peer Responses %

Q. Answered Flagged

Q1

Q2

Q3
Q. Answered Flagged

Q4

Q5

Q6

Q7

Q8 
0:00:43/03:00:00

A 24-year-old man comes to the clinic for review. He has suffered from severe lower back
pain and stiffness for the past five months, which has not responded to regular naproxen. He
is also suffering from bilateral knee pain and intermittent episodes of uveitis. He tells you he
has a brother who has psoriasis and inflammatory bowel disease. There is evidence of
reduced lateral and forward flexion of the spine, and you cannot see any evidence of skin
rash or nail pitting.
Which of the following is the best way to confirm the underlying diagnosis?

A Anti-cyclic citrullinated peptide antibody testing

B HLA-B27 testing

C Magnetic resonance imaging (MRI) of lumbosacral spine and pelvis

D Plain X-ray of lumbar spine

E Rheumatoid factor antibody testing

Explanation 

C Magnetic resonance imaging (MRI) of lumbosacral spine and pelvis

In early ankylosing spondylitis, the likely diagnosis here, plain X-ray is relatively insensitive. As
such, MRI is preferred as the best way to confirm the diagnosis and can guide early
progression to anti-tumour necrosis factor biological therapy.

A Anti-cyclic citrullinated peptide antibody testing

This is of most value in predicting rapid progression in patients with rheumatoid arthritis. It
does not have value in making the diagnosis of ankylosing spondylitis.

B HLA-B27 testing

Although HLA-B27 positivity is strongly associated with the development of ankylosing


spondylitis, negativity does not absolutely rule it out. HLA-B60 and B39 are also associated
with development of the disease.

D Plain X-ray of lumbar spine

Although a plain X-ray can identify sacroiliitis, in early disease, it is potentially not sensitive
enough, and MRI is preferred.
E Rheumatoid factor antibody testing

This may be a relevant negative to screen for, but it is not useful in making a diagnosis of
ankylosing spondylitis. The pattern of joint disease seen here does not suggest rheumatoid
arthritis.
51672

Rate this question:

Next Question

Previous Question Tag Question

Feedback End Review

Difficulty: Average

Peer Responses %

Q. Answered Flagged

Q1

Q2

Q3

Q4

Q5

Q6

Q7

Q8

Q9

0:00:43/03:00:00

A 64-year-old man is referred to the Cardiology Clinic because of repeated syncopal


episodes, the last of which occurred while he was in the local supermarket. He has ischaemic
heart disease and has suffered an inferior myocardial infarction some five years earlier.
Examination in the clinic is unremarkable. His blood pressure is 138/84 mmHg; his pulse is 70
bpm and regular. Heart sounds are normal; the chest is clear, and there is mild pitting
oedema of both ankles. A 24-hour tape has been performed.
Which of the following findings on the electrocardiogram (ECG) is the strongest indication
for permanent pacemaker insertion?

A Left bundle branch block

B 5 × pauses of 2 seconds at night on 24-hour tape

C Periods of Mobitz type 1 heart block at night on 24-hour tape

D Periods of 38 bpm at night on 24-hour tape

E Right bundle branch block and left anterior fascicular block

Explanation 

E Right bundle branch block and left anterior fascicular block

The correct answer represents a bifascicular block, which predisposes to a complete heart
block and, in this situation with repeated syncope, is an indication for progression to a
permanent pacemaker.

A Left bundle branch block

Although a left bundle branch block confirms defective conduction, most likely because of
ischaemic heart disease, it does not necessarily predispose to a complete heart block.
Significant prolongation of the QRS, particularly in the presence of heart failure, may
predispose to ventricular tachycardia (VT) and is an indication for implantable cardioverter–
defibrillator (ICD) insertion.

B 5 × pauses of 2 seconds at night on 24-hour tape

Pauses of up to 3 seconds are considered unremarkable and do not require permanent


pacing. In the presence of symptoms, such as those in this case, had more prolonged pauses
been identified, permanent pacing would have been considered.
C Periods of Mobitz type 1 heart block at night on 24-hour tape

Second-degree heart block with a Mobitz type 1 pattern is considered low risk for
progression to complete heart block. Therefore, pacing is not indicated.

D Periods of 38 bpm at night on 24-hour tape

Significant bradycardia in the absence of complete heart block is seen at night and is not a
strong indication for permanent pacemaker insertion.
51579

Rate this question:

Next Question

Previous Question Tag Question

Feedback End Review

Difficulty: Average

Peer Responses %

Q. Answered Flagged

Q1

Q2

Q3

Q4

Q5

Q6

Q7

Q8

Q9

 External Links

Dalia, T, Amr, B S. 2018. Pacemaker insertions.


ncbi.nlm.nih.gov/books/NBK507823/
(https://www.ncbi.nlm.nih.gov/books/NBK507823/)
0:00:43/03:00:00

A 72-year-old man is admitted for insertion of a pacemaker for cardiac resynchronisation


therapy. The left ventricular lead is inserted via the coronary sinus.
Where is the ostium of the coronary sinus?

A Left atrium

B Left ventricle

C Right atrium

D Right ventricle

E Superior vena cava

Explanation 

C Right atrium

The ostium of the coronary sinus drains into the right atrium in the posteroseptal region near
to the tricuspid valve. In 60% of patients, it is partially covered by the Thebesian valve.

A Left atrium

The left atrium receives the pulmonary veins. The oblique vein of the left atrium is partially
responsible for drainage of blood from the left atrial tissue; it descends obliquely on the back
of the left atrium and ends in the coronary sinus at its left extremity.

B Left ventricle

Most of the drainage from the left ventricle happens via the great cardiac vein, which begins
at the cardiac apex and ascends along the anterior longitudinal sulcus to the base of the
ventricles. It later merges with the oblique vein of the left atrium. It is fed by the lateral and
inferior cardiac veins.

D Right ventricle

The small cardiac vein originates at the base of the right ventricle and parallels the course of
the right coronary artery; it may empty into the coronary sinus, directly into the right atrium
or into the middle cardiac vein.
E Superior vena cava

The superior vena cava is formed by the right and left brachiocephalic and azygos veins. It
enters at the upper and posterior portions of the right atrium.
51597

Rate this question:

Next Question

Previous Question Tag Question

Feedback End Review

Difficulty: Average

Peer Responses %

Q. Answered Flagged

Q1

Q2

Q3

Q4

Q5

Q6

Q7

Q8

Q9

0:00:43/03:00:00

A 46-year-old man who has chronic kidney disease due to autosomal dominant polycystic
kidney disease (ADPKD) is admitted to the Emergency Department with a subarachnoid
haemorrhage thought to be due to a berry aneurysm. He has one brother who is 52 years old
and has normal renal function, with a normal abdominal examination. The patient has three
children who are aged 26, 24 and 22 years.
Which of the following is appropriate with respect to initial screening for ADPKD?

A Screen all first-degree relatives by magnetic resonance angiogram (MRA) of the


brain

B Screen this patient’s children with an abdominal ultrasound scan

C Screen this patient’s children using genetic testing

D Screen all first-degree relatives with an abdominal ultrasound scan

E Screen all male relatives with an abdominal ultrasound scan

Explanation 

B Screen this patient’s children with an abdominal ultrasound scan

This patient has ADPKD. The fact that his brother is aged 52 and has normal renal function
and a normal abdominal examination effectively means he is excluded from having the
disease. The children are above the age of 20, meaning that the sensitivity for diagnosing
ADPKD on ultrasound is effectively 100%. False negatives are seen below this age.

A Screen all first-degree relatives by magnetic resonance angiogram (MRA) of the


brain

The risk here is of discovering an incidentaloma in relatives who do not have ADPKD, which
may drive significant anxiety without benefit to the patient. MRA may be appropriate once
the diagnosis of ADPKD has been established.

C Screen this patient’s children using genetic testing

Given all the children are above the age of 20, ultrasound of the abdomen is sensitive enough
as an initial test to detect ADPKD.

D Screen all first-degree relatives with an abdominal ultrasound scan


This is not required for the patient’s brother, given that he is aged 52, has no symptoms of
renal disease and a normal abdominal examination.

E Screen all male relatives with an abdominal ultrasound scan

ADPKD carries an autosomal dominant inheritance; not screening any female children for the
disease therefore puts them at significant risk.
51603

Rate this question:

Next Question

Previous Question Tag Question

Feedback End Review

Difficulty: Average

Peer Responses %

Q. Answered Flagged

Q1

Q2

Q3

Q4

Q5

Q6

Q7

Q8

Q9

0:00:43/03:00:00

A 48-year-old man comes to the Endocrine Clinic for review. He has gained 10 kg in weight
over the past six months and developed striae over his lower abdomen. His general
practitioner (GP) is concerned because he has also developed type 2 diabetes, now managed
with oral metformin, and has worsening hypertension, now managed with ramipril and
indapamide. His blood pressure in the clinic is 155/95 mmHg and his pulse is 72 bpm and
regular. His 24-hour urinary free cortisol level is markedly elevated on two separate urine
collections over different days.
Which of the following is most likely to reveal the underlying cause of his Cushing
syndrome?

A Chest X-ray

B Low-dose dexamethasone test

C Midnight cortisol

D Plasma adrenocorticotropic hormone (ACTH)

E Random cortisol

Explanation 

D Plasma adrenocorticotropic hormone (ACTH)

Undetectable plasma ACTH against an elevated serum cortisol level is diagnostic of ACTH-
independent Cushing syndrome. This is either due to a primary cortisol-producing adrenal
adenoma or carcinoma or due to exogenous glucocorticoid use.

A Chest X-ray

Although a chest X-ray may discover an underlying pulmonary tumour, only a very small
proportion of these are ACTH-producing, meaning that any finding is most likely to be
unconnected to the presentation.

B Low-dose dexamethasone test

Low-dose dexamethasone testing is most useful in patients who are unable to reliably collect
a 24-hour urine sample. It does not distinguish between pituitary-dependent and pituitary-
independent Cushing syndrome. A high-dose test can distinguish between the two, with a >
90% reduction in cortisol secretion suggesting pituitary-dependent Cushing syndrome.
C Midnight cortisol

Midnight cortisol is used as a screening test for Cushing syndrome, with elevated levels
suggesting the syndrome is present, not differentiating the cause.

E Random cortisol

Cortisol is subject to pulsatile secretion, and analysing random samples is therefore not
useful in diagnosing Cushing syndrome.
51619

Rate this question:

Next Question

Previous Question Tag Question

Feedback End Review

Difficulty: Average

Peer Responses %

Q. Answered Flagged

Q1

Q2

Q3

Q4

Q5

Q6

Q7

Q8

Q9

0:00:43/03:00:00

A 56-year-old man who has gout and type 2 diabetes comes to the clinic with an acutely
swollen right knee. He is unable to flex the knee more than 30 degrees or bear any weight on
the limb. There is obvious erythema and an effusion surrounding the joint.
Which of the following is needed to confirm the underlying diagnosis?

A Blood cultures

B C-reactive protein

C Joint aspiration

D Response to allopurinol

E Serum urate

Explanation 

C Joint aspiration

There are several potential causes for this patient’s knee pain and swelling, including septic
arthritis, gout and pseudogout. For this reason, treating blind with non-steroidals, assuming it
is an acute episode of gout, is not advised. Microscopy with crystallography will differentiate
between gout and pseudogout (positive birefringent crystals in gout); white count will be
informative with respect to the presence of inflammation or sepsis, and growth will reveal any
underlying organisms.

A Blood cultures

Blood cultures are unlikely to be informative, unless there are clues to disseminated sepsis as
a cause of the knee swelling, eg gonococcal arthritis.

B C-reactive protein

C-reactive protein will be raised across all three main potential diagnoses, including gout,
pseudogout and septic arthritis, and is therefore not informative here.

D Response to allopurinol

Allopurinol is not an appropriate intervention for acute gout because it can actually worsen
symptoms during a flare.
E Serum urate

Serum urate may be elevated or near the upper limit of normal in the context of a gout flare.
It is not useful in diagnosing an acute attack of gout and would not rule out other causes of
acute knee inflammation.
51670

Rate this question:

Next Question

Previous Question Tag Question

Feedback End Review

Difficulty: Average

Peer Responses %

Q. Answered Flagged

Q1

Q2

Q3

Q4

Q5

Q6

Q7

Q8

Q9

0:00:43/03:00:00

A 34-year-old woman comes to the Asthma Clinic for review. She is currently taking
fluticasone 500 mg inhaled twice daily but is still symptomatic and finding she is using a
whole salbutamol inhaler, as required (prn), every ten days. Her peak flow is 380 litres/min
(505 predicted).
Which is the most appropriate next step in asthma management?

A Montelukast

B Prednisolone

C Salmeterol

D Theophylline

E Tiotropium

Explanation 

A Montelukast

The National Institute for Health and Care Excellence (NICE) guidelines recommend a trial of
leukotriene receptor antagonist (LTRA) in patients who fail to respond to inhaled
corticosteroids alone. She is already taking the equivalent of 1 mg of beclometasone twice
daily, and there is little scope to further increase the inhaled steroid dose. If the trial of LTRA
fails to control symptoms, addition of a long-acting beta agonist is recommended.

B Prednisolone

Oral corticosteroids are an intervention of last resort for the treatment of asthma because of
risks associated with long-term systemic steroid exposure, including weight gain, impaired
glucose tolerance and osteoporosis.

C Salmeterol

A long-acting beta agonist, such as salmeterol, is the default choice in patients where
symptoms are not controlled on inhaled corticosteroids and an LTRA.

D Theophylline

Oral theophylline is a beta agonist which has limited value in patients who already have an
adequate inhaler technique. It carries risks of atrial arrhythmia and, as such, is best avoided.
E Tiotropium

Tiotropium is a long-acting cholinergic antagonist. It has most value in the treatment of


patients with uncontrolled symptoms of chronic obstructive pulmonary disease, not asthma.
51662

Rate this question:

Next Question

Previous Question Tag Question

Feedback End Review

Difficulty: Average

Peer Responses %

Q. Answered Flagged

Q1

Q2

Q3

Q4

Q5

Q6

Q7

Q8

Q9

 External Links

National Institute for Health and Care Excellence. (2017). Asthma: diagnosis, monitoring and chronic asth…
nice.org.uk/guidance/ng80
(https://www.nice.org.uk/guidance/ng80)
0:00:43/03:00:00

A 64-year-old man, a painter and decorator, comes to the clinic for review. He has been
suffering from multiple fainting episodes while at work over the past few months. He has
hypertension and smokes 25 cigarettes per day. He is left-handed.
His blood pressure in the clinic is measured at 175/85 mmHg in the right arm and 148/75
mmHg in the left. He has no carotid bruits.
Which of the following is the most likely diagnosis?

A Absence seizures

B Coarctation of the aorta

C Subclavian steal syndrome

D Transient ischaemic attacks

E Vasovagal syncope

Explanation 

C Subclavian steal syndrome

This patient’s left-handedness, differential in blood pressures between the right and left arms
and multiple fainting episodes when he uses his left arm for vigorous exercise, holding it
above his head, fits with subclavian steal syndrome. This is due to retrograde blood flow
through the vertebral artery to supply blood to the left arm.

A Absence seizures

The trigger here is clearly one of exercise involving the left arm. It would not fit with a
diagnosis of epilepsy for this patient’s episodes only to be related to physical work.

B Coarctation of the aorta

Coarctation of the aorta would present with symptoms involving the lower limbs, as well as
the left arm, at an earlier age, with difficult-to-control hypertension. Presentation at the age
of 64 would be highly unlikely.

D Transient ischaemic attacks


The fact the attacks always happen at work and after a period of exercise makes transient
ischaemic attacks an unlikely diagnosis. Focal signs, rather than transient loss of
consciousness, would also be expected.

E Vasovagal syncope

This could be a possible diagnosis once subclavian steal syndrome has been excluded, but
given he is left-handed and there is a differential in blood pressures, subclavian steal
syndrome should be ruled out first.
51643

Rate this question:

Next Question

Previous Question Tag Question

Feedback End Review

Difficulty: Average

Peer Responses %

Q. Answered Flagged

Q1

Q2

Q3

Q4

Q5

Q6

Q7

Q8

Q9

0:00:43/03:00:00

A 78-year-old woman with atrial fibrillation (AF) and ischaemic heart disease presents to the
Emergency Department with acute-onset abdominal pain and vomiting. She has been
suffering from increasing abdominal pain after meals over the last few months.
Her blood pressure is 95/60 mmHg, and her pulse is 105 bpm and irregular. Her abdomen is
generally tender, with signs of peritonism.
Investigations:

Haemoglobin (Hb) 10.9 g/dl (11.5–15.5 g/dl)

White cell count (WCC) 14.8 × 10 9/l (4–11 × 10 9/l)

Platelets (PLT) 209 × 10 9/l (150–400 × 10 9/l)

Sodium (Na +) 143 mmol/l (135–145 mmol/l)

Potassium (K +) 5.9 mmol/l (3.5–5.0 mmol/l)

Creatinine (Cr) 195 µmol/l (50–120 µmol/l)

Bicarbonate (HCO 3) 15 mmol/l (24–30 mmol/l)

Lactate 5.9 mmol/l (0.5–2.2 mmol/l)

Where is the most likely location of this patient’s abdominal problem?

A Caecum

B Gall bladder

C Ileum

D Splenic flexure

E Stomach

Explanation 

D Splenic flexure

The presentation of abdominal pain, vomiting, metabolic acidosis and an elevated lactate
level fits with a diagnosis of ischaemic colitis. Marginal branches of the middle colic artery
(part of the superior mesenteric territory) and left colic arteries (part of the inferior
mesenteric territory) supply the transverse and descending segments of the colon. There is
an arterial and lymphatic watershed existing near to the splenic flexure, making this part of
the colon at risk of ischaemic colitis. Further investigation of this patient is required with an
urgent abdominal computed tomography (CT) scan with contrast. Following that,
consideration of urgent surgical intervention is required.

A Caecum

The caecum may be affected because of ileitis in patients with Crohn’s disease, though the
patient’s age and co-morbidities make Crohn’s disease unlikely in this case. As a site of colon
cancer, there is a late discovery due to stool being predominantly liquid at this stage. It is not
an area of the bowel at risk of ischaemia, the most likely diagnosis here.

B Gall bladder

The marked elevation in lactate level in an arteriopath who has AF makes ischaemic colitis
much more likely than cholecystitis or a gall bladder perforation.

C Ileum

The ileum is only supplied by the superior mesenteric artery; because there is no watershed
area, with a crossover of arterial supply, it is usually not at risk from ischaemia.

E Stomach

The stomach has an excellent arterial supply and is not usually subject to ischaemia. Gastric
perforations are seen with untreated ulcer disease, although that does not fit with the history
reported here.
51627

Rate this question:

Next Question

Previous Question Tag Question

Feedback End Review

Difficulty: Average

Peer Responses %
Q. Answered Flagged

Q1

Q2

Q3

Q4

Q5

Q6

Q7

Q8

Q9

0:00:43/03:00:00

A 54-year-old woman who has asthma comes to the Emergency Department with signs of
left lower lobe consolidation and a cold sore on her left upper lip. You suspect that she has
streptococcal pneumonia and request a streptococcal antigen test.
The sensitivity of this test is 80%, and the specificity 95%. The prevalence of streptococcal
pharyngitis in adults with pneumonia is 40%.
What is the likelihood that the patient has streptococcal pharyngitis if the rapid
streptococcal antigen test is positive?

A 95%

B 91%

C 88%

D 80%

E 5%

Explanation 

B 91%

This is the positive predictive value (PPV).

To calculate the answer, you need to imagine how the figures around prevalence, sensitivity
and specificity would look in a hypothetical group of 1 000 patients.

Streptococcal pneumonia No streptococcal pneumonia Total

Positive streptococcal,
320 30 350
antigen test

Negative streptococcal
80 570 650
antigen test

Total 400 600 1 000

The PPV is the proportion of subjects who tested positive who have the disease.

A 95%

This is the specificity, which, as stated in the question, is 95%.


C 88%

This is the negative predictive value, ie the proportion of patients who test negative who do
not have the disease.

D 80%

This is the sensitivity, ie the ability of the test to detect everyone with pneumonia. It detects
80% of the cases.

E 5%

This does not fit with a calculation for the sensitivity, specificity, or negative or positive
predictive value.
51606

Rate this question:

Next Question

Previous Question Tag Question

Feedback End Review

Difficulty: Average

Peer Responses %

Q. Answered Flagged

Q1

Q2

Q3

Q4

Q5

Q6
Q. Answered Flagged

Q7

Q8

 External Links

Talluri, S K. (2009). Positive predictive value.


bmj.com/content/339/bmj.b3835
(https://www.bmj.com/content/339/bmj.b3835)
0:00:43/03:00:00

A 67-year-old man, who has received palliative radiotherapy for a right-sided squamous cell
carcinoma of the bronchus months earlier, comes to the Emergency Department complaining
of an egg cup full of haemoptysis. He also suffered from dull right-sided chest pain over the
past 2–3 weeks, which has not responded to his usual regimen of morphine slow-release and
paracetamol in combination.
Examination reveals decreased breath sounds at the right base, but nothing else of note. His
blood pressure is 145/82 mmHg; his pulse is 73 bpm and regular.
Investigations:

(13.5–17.5
Haemoglobin (Hb) 10.2 g/dl
g/dl)

(4–11 ×
White cell count (WCC) 11.1 × 10 9/l
10 9/l)

(150–400
Platelets (PLT) 187 × 10 9/l
× 10 9/l)

(135–145
Sodium (Na +) 144 mmol/l
mmol/l)

(3.5–5.0
Potassium (K +) 4.9 mmol/l
mmol/l)

(5–120
Creatinine (Cr) 103 µmol/l
mmol/l)

(10.6–14.9
Prothrombin time (PT) 13.1 s
s)

Activated partial (23.0–


33.2 s
thromboplastin time (APTT) 35.0 s)

Right-sided X-ray appearance unchanged from


Chest X-ray
the last post-radiotherapy film

Which of the following is the most appropriate next step?

A Computed tomography pulmonary angiogram (CTPA)

B Fresh frozen plasma (FFP)

C Tranexamic acid

D Vitamin K
E Ventilation/perfusion (V/Q) scan

Explanation 

A Computed tomography pulmonary angiogram (CTPA)

In this case, haemoptysis may be related to the erosion of a pulmonary vessel because of an
underlying tumour or may be due to another cause such as a pulmonary embolism (PE). As
such, a CTPA is the most appropriate next step. In the event that a large vessel at risk of
erosion or bleeding is identified, embolisation may be one potential intervention. If a PE is
identified, anticoagulation should be considered.

B Fresh frozen plasma (FFP)

This patient’s platelets, PT and APTT are in the normal range. FFP has a role in the treatment
of massive bleeding, which is not apparent here, and a PE should be ruled out first.

C Tranexamic acid

Tranexamic acid is appropriate for massive haemoptysis due to erosion of a vessel when
other options, such as embolisation, have been ruled out. In this case, the patient has stable
observations and there is time to assess for a PE or pulmonary vessel erosion with a CTPA,
before considering tranexamic acid.

D Vitamin K

The PT is in the normal range, and there is no reason to suspect that vitamin K is deficient
here. It is unlikely to reduce any risk of bleeding and will not impact on the risk of further PE
if that is the underlying cause.

E Ventilation/perfusion (V/Q) scan

Although a V/Q scan will identify a potential PE, it is less useful, compared to CTPA, with
respect to identifying encroachment of any tumour on pulmonary vessels and the potential
for intervention.
51583

Rate this question:

Next Question

Previous Question Tag Question

Feedback End Review

Difficulty: Average
Peer Responses %

Q. Answered Flagged

Q1

Q2

Q3

Q4

Q5

Q6

Q7

Q8

Q9

0:00:43/03:00:00

A 24-year-old man is reviewed in the Nephrology Clinic. He has been identified as having
microscopic haematuria by his general practitioner (GP) and has already undergone an
ultrasound scan of his abdomen and pelvis which has been reported as normal.
His blood pressure is 122/82 mmHg, and his pulse is 72 bpm and regular. Abdominal
palpation is unremarkable.
Investigations:

Haemoglobin (Hb) 13.2 g/dl (13.5–17.5 g/dl)

White cell count (WCC) 7.2 × 10 9/l (4–11 × 10 9/l)

Platelets (PLT) 203 × 10 9/l (150–400 × 10 9/l)

Sodium (Na +) 145 mmol/l (135–145 mmol/l)

Potassium (K +) 4.5 mmol/l (3.5–5.0 mmol/l)

Creatinine (Cr) 105 µmol/l (50–120 µmol/l)

Urine Blood +, protein +

Which of the following is the most likely diagnosis?

A Goodpasture syndrome

B Immunoglobulin A (IgA) nephropathy

C Membranous nephropathy

D Polycystic kidney disease

E Transitional cell carcinoma (TCC) of the bladder

Explanation 

B Immunoglobulin A (IgA) nephropathy

Microscopic haematuria with or without proteinuria is a presenting feature of IgA


nephropathy in up to half of patients with the condition. Confirmation of the diagnosis
requires a renal biopsy, although in patients with stable renal function who are not
hypertensive, a watch-and-wait approach can be taken.

A Goodpasture syndrome
Goodpasture syndrome leads to frank haematuria and is due to anti-glomerular basement
membrane antibodies. There is a rapidly progressive decline in renal function, in contrast to
the chronic haematuria and stable creatinine seen here.

C Membranous nephropathy

Membranous nephropathy is associated with marked proteinuria, which is often in the


nephrotic range. It would not fit with this clinical scenario.

D Polycystic kidney disease

Polycystic kidney disease is associated with a gradual worsening of renal function over time
and bilateral enlarged kidneys with cysts. Ultrasound has a sensitivity for diagnosing
polycystic kidney disease approaching 100% above 20 years of age.

E Transitional cell carcinoma (TCC) of the bladder

Although TCC of the bladder is a possible diagnosis, the age of the patient makes this much
less likely. Other factors such as smoking which can accelerate development of the disease
are also absent here.
51647

Rate this question:

Next Question

Previous Question Tag Question

Feedback End Review

Difficulty: Average

Peer Responses %

Q. Answered Flagged

Q1

Q2
Q. Answered Flagged

Q3

Q4

Q5

Q6

Q7

Q8 
0:00:43/03:00:00

A 42-year-old man is admitted to the Emergency Department from the local prison. He
complains of central crushing chest pain radiating down his left arm. This is his fourth
admission in the last three months, and he has had a normal coronary angiogram after his
second admission. There is apparently some bullying in his prison, and he has been asked to
be moved.
Examination, electrocardiogram and routine bloods are normal.
Which of the following is the most likely diagnosis?

A Conversion disorder

B Coronary artery vasospasm

C Factitious disorder

D Malingering

E Somatisation disorder

Explanation 

D Malingering

Malingering is described as an intentionally produced deception syndrome which has an


external incentive. In this case, the prisoner gains time out of jail each time he complains of
chest pain.

A Conversion disorder

A conversion disorder is an unconsciously produced somatoform disorder. Most conversion


disorders are manifest as neurological complaints such as loss of power or sensation in a
limb.

B Coronary artery vasospasm

Although this is not impossible, the normal investigations on each occasion make it unlikely
as a cause of his presentation. He has significant incentives for malingering.

C Factitious disorder
A factitious disorder is an intentionally produced deception syndrome which does not have
an external incentive, which is how it differs from malingering.

E Somatisation disorder

A somatisation disorder is unconsciously produced and leads usually to multiple complaints


across multiple organ systems.
51657

Rate this question:

Next Question

Previous Question Tag Question

Feedback End Review

Difficulty: Average

Peer Responses %

Q. Answered Flagged

Q1

Q2

Q3

Q4

Q5

Q6

Q7

Q8

Q9

 External Links

BMJ Best Practice. (2019). Factitious disorders


bestpractice.bmj.com/topics/en-gb/695
(https://bestpractice.bmj.com/topics/en-gb/695)
0:00:43/03:00:00

A 74-year-old man with chronic obstructive pulmonary disease (COPD) comes to the
Emergency Department with an exacerbation. Despite one hour of continuous nebulisation
with salbutamol and ipratropium, and intravenous (IV) hydrocortisone, he is still markedly
symptomatic. He is currently managed with 28% inspired oxygen. His pH is 7.30, p(O 2) 7.1 kPa
and p(CO 2) 7.6 kPa.
Which of the following is the most appropriate next intervention?

A Continue nebulisation

B Intubation and ventilation

C IV aminophylline

D IV magnesium

E Non-invasive positive pressure ventilation (NIPPV)

Explanation 

E Non-invasive positive pressure ventilation (NIPPV)

This patient is acidotic, with carbon dioxide (CO 2) retention and hypoxia. In this situation,
when the pH is above 7.26 and acidotic, despite one hour of conventional therapy, NIPPV is
said to be the most effective. This will help reverse acidosis, maintaining respiratory muscle
strength and reducing the risk of progression to the Intensive Therapy Unit.

A Continue nebulisation

Continued nebulisation will not reverse the acidosis seen here. The risk is that there will be a
gradual further fall in pH, worsening respiratory muscle weakness, hypoxia and acidosis.

B Intubation and ventilation

Intubation and ventilation are the obvious next step in the event that NIPPV fails to control
his symptoms. It is important to assess the pros and cons of intubation, taking into account
his pre-morbid state and any wishes that he may have previously held. Discussion with family
members in the case of initiating level 3 care is always important.

C IV aminophylline
If nebulisation is adequately delivered, there is little value in systemic beta agonist therapy. In
this situation, it is only likely to worsen the risk of atrial arrhythmia.

D IV magnesium

IV magnesium is indicated in the treatment of acute severe asthma. Its efficacy has not been
proven in the treatment of COPD.
51665

Rate this question:

Next Question

Previous Question Tag Question

Feedback End Review

Difficulty: Average

Peer Responses %

Q. Answered Flagged

Q1

Q2

Q3

Q4

Q5

Q6

Q7

Q8

Q9

 External Links

Guideline Development Group


brit-thoracic.org.uk/document-library/clinical-information/niv/niv-guidelines/btsrcpics-guideline-on-niv-in-copd/
(https://www.brit-thoracic.org.uk/document-library/clinical-information/niv/niv-
guidelines/btsrcpics-guideline-on-niv-in-copd/)
0:00:43/03:00:00

A 67-year-old woman attends the Gastroenterology Clinic for follow-up of iron deficiency
anaemia. She has undergone two colonoscopies and one upper gastrointestinal (GI)
endoscopy, none of which revealed the source of bleeding. There have also been two
episodes of sudden collapse without warning.
A capsule endoscopy has revealed evidence of small bowel angiodysplasia. Her general
practitioner (GP) has noted a systolic murmur.
Which of the following valve disorders is most likely to be present in this patient?

A Aortic regurgitation

B Aortic stenosis

C Mitral regurgitation

D Mitral stenosis

E Pulmonary regurgitation

Explanation 

B Aortic stenosis

This patient has Heyde syndrome, characterised by aortic stenosis leading to episodes of
sudden collapse and iron deficiency anaemia as a result of small bowel angiodysplasia. The
angiodysplasia is thought to develop in relation to von Willebrand’s disease 2A because of
blood flowing across a stenotic aortic valve. GI bleeding usually significantly improves once
the valve is replaced.

A Aortic regurgitation

Aortic regurgitation results in cardiac failure and ventricular dilatation but does not carry the
same association with angiodysplasia. It is also associated with a diastolic, rather than a
systolic, murmur.

C Mitral regurgitation

Mitral regurgitation occurs as a pan systolic murmur. It is seen both as a result of valve
disease and as functional MR related to cardiac failure. It is not associated with the
development of angiodysplasia.
D Mitral stenosis

Mitral stenosis is associated with increased intensity of the first heart sound and a diastolic
murmur. It is associated with the development of right ventricular failure and atrial fibrillation,
rather than GI bleeding.

E Pulmonary regurgitation

Although pulmonary regurgitation is associated with a systolic murmur, it is usually seen in


association with pulmonary hypertension, rather than with GI bleeding.
51580

Rate this question:

Next Question

Previous Question Tag Question

Feedback End Review

Difficulty: Average

Peer Responses %

Q. Answered Flagged

Q1

Q2

Q3

Q4

Q5

Q6

Q7

Q8
Q. Answered Flagged 
0:00:43/03:00:00

A 74-year-old woman has recently started treatment with fludarabine for chronic
lymphocytic leukaemia (CLL). She has become short of breath over the past few days, with a
significant reduction in her exercise tolerance, and has developed anaemia with a
haemoglobin (Hb) level of 6.2 g/dl (11.5–15.5 g/dl).
Which of the following is the most appropriate transfusion for her?

A Filtered red cells

B Irradiated red cells

C Packed red cells

D Washed red cells

E Whole blood

Explanation 

B Irradiated red cells

Irradiated red cells are indicated for patients with haematological malignancies requiring
blood transfusion who are at risk of transfusion-related graft-versus-host disease (TA-GvHD).
Irradiation damages white cell deoxyribonucleic acid (DNA), rendering the cells non-
functioning. Examples of patients at risk of TA-GvHD include those treated with fludarabine,
those with congenital immune system disorders and patients with lymphoma who have
undergone bone marrow transplant.

A Filtered red cells

Although filtered red cells reduce the risk of TA-GvHD, they do not abolish it completely. For
this reason, irradiated red cells are preferred.

C Packed red cells

Packed red cells contain functioning white blood cells. For this reason, they must not be used
in patients at risk of TA-GvHD.

D Washed red cells

Like filtering red cells, washing red cells to remove any remaining plasma is less effective than
irradiating them, with respect to reducing white cell burden.
E Whole blood

Whole blood contains a significant number of white blood cells and should not be used in this
patient because of the high risk of driving TA-GvHD.
51585

Rate this question:

Next Question

Previous Question Tag Question

Feedback End Review

Difficulty: Average

Peer Responses %

Q. Answered Flagged

Q1

Q2

Q3

Q4

Q5

Q6

Q7

Q8

Q9

0:00:43/03:00:00

A 72-year-old man is admitted to the Emergency Department with acute neurological


symptoms. He has weakness of his left leg and foot, with associated sensory loss. He looks
apathetic and is non-communicative. You suspect he has suffered an acute stroke.
Where is the lesion most likely to have occurred?

A Proximal segment of the anterior cerebral artery

B Distal segment of the anterior cerebral artery

C Anterior choroidal artery

D Posterior communicating artery

E Posterior inferior cerebellar artery

Explanation 

B Distal segment of the anterior cerebral artery

This patient’s symptoms are consistent with a stroke affecting the parietal lobe, which can
result from lesions of the distal anterior cerebral artery. Other symptoms which may be seen
include incontinence, particularly in bilateral parietal lesions, and development of a
contralateral grasp reflex.

A Proximal segment of the anterior cerebral artery

Proximal segment occlusion of the anterior cerebral artery is less likely to result in
neurological sequelae because of the existence of collateral blood supply.

C Anterior choroidal artery

The anterior choroidal artery supplies a major portion of the midbrain, the lateral geniculate
body, choroid plexus of the lateral ventricles and third ventricle, globus pallidus, caudate
nucleus, amygdala, hypothalamus, red nucleus, substantia nigra, posterior limb of the internal
capsule, optic tract, hippocampus and the fimbria of the fornix.

D Posterior communicating artery

Each posterior communicating artery supplies the three cerebral arteries of the same side. It
is a common site for aneurysm formation, rather than for occlusive stroke.
E Posterior inferior cerebellar artery

Occlusion of the posterior inferior cerebellar artery results in lateral medullary syndrome,
leading to symptoms such as nausea, vomiting, vertigo and nystagmus, and impairment of
glossopharyngeal and vagus nerve function. There are ipsilateral cerebellar signs and
contralateral loss of pain and temperature sensation.
51598

Rate this question:

Next Question

Previous Question Tag Question

Feedback End Review

Difficulty: Average

Peer Responses %

Q. Answered Flagged

Q1

Q2

Q3

Q4

Q5

Q6

Q7

Q8

Q9

0:00:43/03:00:00

You are reviewing the results of a new liquid biopsy technique for the detection of lung
cancer recurrence. The company are hoping the technique may replace repeat computed
tomography (CT) and positron emission tomography (PET) scanning in the future. The trial of
the technique was conducted against traditional methods of detecting recurrence considered
as the gold standard.
The results are shown below.

Recurrence of No recurrence of
Total
cancer cancer

Positive liquid biopsy for recurrence 95 5 100

Negative liquid biopsy for


15 85 100
recurrence

Total 110 90 200

Which of the following is the sensitivity of liquid biopsy?

A 15/110 = 14%

B 85/100 = 85%

C 95/110 = 86%

D 85/90 = 94%

E 95/100 = 95%

Explanation 

C 95/110 = 86%

This is the sensitivity, ie the ability of the test to detect everyone with a recurrence of cancer.
In this case, it is 95/(95 + 15), which is 86%.

A 15/110 = 14%

This is the reciprocal of the sensitivity and does not represent a performance parameter used
to evaluate screening tests.

B 85/100 = 85%
This is the negative predictive value, ie the ability of the test to correctly identify people who
do not have a cancer recurrence, which equals 85/(85 + 15).

D 85/90 = 94%

This is the specificity of the test, which equals 85/(85 + 5).

E 95/100 = 95%

This is the positive predictive value, ie the ability of the test to correctly identify all patients
who have a cancer recurrence, which equals 95/(95 + 5).
51607

Rate this question:

Next Question

Previous Question Tag Question

Feedback End Review

Difficulty: Average

Peer Responses %

Q. Answered Flagged

Q1

Q2

Q3

Q4

Q5

Q6

Q7
Q. Answered Flagged

Q8
0:00:43/03:00:00

A 72-year-old woman is admitted from her nursing home with projectile vomiting and
explosive diarrhoea. This started with no warning, and several other residents in her facility
have been ill for 24–48 hours with a gastrointestinal illness. She opens her bowels once on
arrival in the Emergency Department with profuse watery diarrhoea.
Her blood pressure is 125/90 mmHg, and her pulse is 85 bpm and regular. There is a 20-
mmHg drop in her blood pressure on standing.
Which of the following is the most likely underlying cause of her gastrointestinal illness?

A Clostridium difficile

B Enterovirus

C Norovirus

D Salmonella enterica

E Shigella sonnei

Explanation 

C Norovirus

Norovirus causes vomiting in > 50% of those affected, as well as abdominal pain, diarrhoea
and anorexia. It has an incubation period of between 24 and 48 hours, and a duration of
symptoms of 12–60 hours. It is highly infectious and can be spread by person-to-person
contact. Along with rotavirus, it is a very common cause of nursing home gastroenteritis.

A Clostridium difficile

C. difficile leads to symptoms of colitis, including abdominal pain and diarrhoea with blood
and mucus. Infection control measures are required to reduce person-to-person spread,
including environmental decontamination. Limitation of cephalosporins and quinolones
should also be considered to limit cases of C. difficile.

B Enterovirus

Enteroviruses, such as Coxsackie B, are more likely to be associated with milder


gastrointestinal symptoms and a respiratory illness picture, rather than the profuse diarrhoea
and vomiting seen here.
D Salmonella enterica

Salmonella infections are invasive and are associated with diarrhoea and vomiting, but
invasive symptoms such as blood in the stool are more likely to be seen.

E Shigella sonnei

The main feature of Shigella infection is bloody diarrhoea, which would not fit with the
vomiting seen here and the characteristics of the diarrhoea as described. It can, however,
been seen in outbreaks, most often in children’s day-care centres.
51621

Rate this question:

Next Question

Previous Question Tag Question

Feedback End Review

Difficulty: Average

Peer Responses %

Q. Answered Flagged

Q1

Q2

Q3

Q4

Q5

Q6

Q7

Q8
Q. Answered Flagged 

 External Links

Dirk, M D, Veitch, M G, Hall, G V. (2010). Gastroenteritis and food-borne disease in elderly people living in l…
academic.oup.com/cid/article/50/3/397/395320
(https://academic.oup.com/cid/article/50/3/397/395320)
0:00:43/03:00:00

A 35-year-old climber presents with a left lower motor neurone facial nerve palsy. He has
returned from the Austrian Alps a few weeks earlier, after which he tells you he noticed an
odd round patch of erythema on his left leg, with a blackened area at its centre.
Examination in the clinic, apart from the facial nerve palsy, is unremarkable.
Which of the following is the most useful investigation?

A Ancylostoma duodenale serology

B Borrelia burgdorferi serology

C Orientia tsutsugamushi serology

D Rickettsia felis serology

E Rickettsia rickettsii serology

Explanation 

B Borrelia burgdorferi serology

This man is likely to be suffering from Lyme disease as a result of B. burgdorferi infection
acquired from a tick bite while climbing in Austria. The rash on his leg is consistent with an
eschar (the blackened area) and erythema chronicum migrans (the round patch of
erythema). The National Institute for Health and Care Excellence (NICE) recommendation is
actually to begin treatment with doxycycline, while awaiting serology, in patients who give a
history of likely tick bite. Complications which can occur in patients with Lyme disease
include mononeuritis and complete heart block.

A Ancylostoma duodenale serology

It is a species of roundworm known to infect humans, cats and dogs. It may cause iron
deficiency anaemia with heavy infection and can be treated with mebendazole or
albendazole.

C Orientia tsutsugamushi serology

This is the cause of scrub typhus which occurs in South and South East Asian Pacific lands. It
is spread by small mites known as chiggers.

D Rickettsia felis serology


R. felis is found to infect cats and is a cause of endemic typhus infection.

E Rickettsia rickettsii serology

This is the cause of Rocky Mountain spotted fever, which is spread by tick bites, but is
associated with an erythematous, non-itchy rash. It also does not fit with this patient’s area of
travel to Austria.
51635

Rate this question:

Next Question

Previous Question Tag Question

Feedback End Review

Difficulty: Average

Peer Responses %

Q. Answered Flagged

Q1

Q2

Q3

Q4

Q5

Q6

Q7

Q8

Q9

0:00:43/03:00:00

A 45-year-old man comes to the clinic for review. He has been feeling increasingly fearful of
failure at work and tells you that, for at least three days out of every five working days, he
almost feels too anxious to go to work and has taken to drinking a vodka each morning to
calm his nerves. Over the past month, he has refused three work engagements because of
stress and constantly has palpitations, sweating and a dry mouth. He also worries about
caring for his child because he feels he cannot do a good enough job. He is irritable and has
been visiting a chiropractor for muscle tension and wakes around 0530 h each morning.
Physical examination is unremarkable, and he takes no regular medication.

Which of the following is the most likely diagnosis?

A Alcoholism

B Agoraphobia

C Depression

D Generalised anxiety disorder (GAD)

E Obsessive–compulsive disorder

Explanation 

D Generalised anxiety disorder (GAD)

Several features here point towards GAD, including the fact he has symptoms of anxiety for
more days than not, uses alcohol to self-medicate through his anxiety, has other symptoms
such as muscle tension, palpitations and sweating and suffers from early morning wakening
with anxiety. Cognitive behavioural therapy and assistance to withdraw from alcohol are both
important first steps in his management.

A Alcoholism

Although he is using alcohol to self-medicate for his anxiety, this is unlikely to be the primary
problem. With appropriate intervention for GAD, a reduction in alcohol consumption should
be achievable.

B Agoraphobia

Agoraphobia is specifically a fear of being in situations where escape might be difficult or


that help would not be available if things go wrong. It does not fit with the particular worries
seen here around work or the inability to perform childcare duties.
C Depression

Anxiety is much more prominent than low mood in this scenario. The only feature which
could fit with both depression and GAD is early morning wakening.

E Obsessive–compulsive disorder

Obsessions are unwanted, intrusive thoughts, images or urges that repeatedly enter the
person's mind. Compulsions are repetitive behaviours or mental acts that the person feels
driven to perform. It does not therefore fit with the situation described here.
51656

Rate this question:

Next Question

Previous Question Tag Question

Feedback End Review

Difficulty: Average

Peer Responses %

Q. Answered Flagged

Q1

Q2

Q3

Q4

Q5

Q6

Q7

Q8
Q. Answered Flagged 
0:00:43/03:00:00

A 76-year-old woman suffering from emphysema comes to the Emergency Department with
an exacerbation of chronic obstructive pulmonary disease (COPD). She currently takes high-
dose inhaled corticosteroids, long-acting beta agonist and long-acting anticholinergic
therapy. Her respiratory rate is 35 breaths/minute on admission, and she is anxious and
distressed. Oxygen (O 2) saturation on air is 86%. pH on venous blood gas is 7.40.
Which of the following is the most appropriate O 2 therapy regimen?

A 100% O 2 via non-rebreathe mask

B 60% O 2 via Venturi mask

C 35% O 2 via Venturi mask

D 28% O 2 via Venturi mask

E Air only

Explanation 

D 28% O 2 via Venturi mask

It is likely that this patient chronically runs an O 2 saturation of between 88% and 91%. Hence,
only a small increase in inspired O 2 is likely to be needed to achieve the target O 2 saturation.
Given this patient has significant shortness of breath and is distressed, choosing 28% O 2,
which can be turned down if needed, is preferable to a small step-up from air to 24%. In this
situation, it is also imperative that the patient receives regular monitoring of her blood gases
to ensure that she is not developing decompensated type 2 respiratory failure. Regular gases
will also be useful to aid the titration of inspired O 2 through the Venturi mask.

A 100% O 2 via non-rebreathe mask

This is not appropriate for a patient with a long history of COPD; it is likely to drive carbon
dioxide (CO 2) retention and worsening acidosis, which, in turn, will drive respiratory muscle
weakness and subsequent decompensated type 2 respiratory failure.

B 60% O 2 via Venturi mask

Like 100% O 2, 60% O 2 via a Venturi mask is likely to drive significant CO 2 retention, with
decompensated acidosis and decreased conscious level.

C 35% O 2 via Venturi mask


35% O 2 may drive excess CO 2 retention. It should be avoided as an initial O 2 replacement
dose but could be used as a step-up if hypoxia is still present on 28% O 2.

E Air only

This patient has significant hypoxia, and maintaining inspired O 2 of 21% only increases the risk
of confusion, tissue acidosis and cardiac arrhythmia.
51664

Rate this question:

Next Question

Previous Question Tag Question

Feedback End Review

Difficulty: Average

Peer Responses %

Q. Answered Flagged

Q1

Q2

Q3

Q4

Q5

Q6

Q7

Q8

Q9

0:00:43/03:00:00

A 48-year-old woman with a 7-year history of rheumatoid arthritis presents to the Neurology
Clinic with a left foot drop. Her rheumatoid arthritis (RA) is currently under control, although
she has required methotrexate and adalimumab in combination to gain control of her
symptoms and is awaiting a left knee replacement.
Examination reveals evidence of RA affecting multiple joints, including fingers, toes, hands,
feet, wrists, knees and elbows. You confirm the left foot drop.
Which of the following is the most likely cause of her foot drop?

A Anterior tarsal tunnel syndrome

B Entrapment of the peroneal nerve at the fibular neck

C Inflammatory neuritis

D L5/S1 disc prolapse

E Posterior tarsal tunnel syndrome

Explanation 

B Entrapment of the peroneal nerve at the fibular neck

Common peroneal nerve palsy is relatively common in patients who have RA. Nerve
entrapment is seen with both severe knee deformity due to RA and post-knee replacement. A
magnetic resonance imaging (MRI) scan is the obvious next step to determine the degree of
entrapment.

A Anterior tarsal tunnel syndrome

Anterior tarsal tunnel syndrome is a rarely reported entrapment neuropathy of the deep
peroneal nerve under the extensor retinaculum at the ankle. In contrast to posterior tarsal
tunnel syndrome, it is associated with burning and pain over the anterior surface of the ankle
and the dorsum of the foot.

C Inflammatory neuritis

An inflammatory neuritis could lead to a foot drop in this situation but is less likely than
physical entrapment because of long-standing rheumatoid disease.

D L5/S1 disc prolapse


L5/S1 disc prolapse is a cause of foot drop, although the absence of back pain makes it very
unlikely to be a cause of this patient’s symptoms.

E Posterior tarsal tunnel syndrome

Posterior tarsal tunnel syndrome involves damage to the tibial nerve where it runs under the
flexor retinaculum on the medial side of the ankle. The initial symptom is usually burning pain
or sensory loss affecting the medial and lateral plantar nerves.
51640

Rate this question:

Next Question

Previous Question Tag Question

Feedback End Review

Difficulty: Average

Peer Responses %

Q. Answered Flagged

Q1

Q2

Q3

Q4

Q5

Q6

Q7

Q8

Q9

0:00:43/03:00:00

A 45-year-old man, who is on a long-term attachment in West Africa, is flown home by his
company after becoming unwell. He believes that he is being poisoned and people are trying
to steal his money, and that he has powers so that he can see into other people’s minds to
determine their intentions.
He is currently taking mefloquine for malarial prophylaxis and ramipril 5 mg daily for control
of blood pressure. His brother takes medication for bipolar disorder.
Investigations:

Haemoglobin (Hb) 13.3 g/dl (13.5–17.5 g/dl)

White cell count (WCC) 7.1 × 10 9/l (4–11 × 10 9/l)

Platelets (PLT) 203 × 10 9/l (150–400 × 10 9/l)

Sodium (Na +) 143 mmol/l (135–145 mmol/l)

Potassium (K +) 4.5 mmol/l (3.5–5.0 mmol/l)

Creatinine (Cr) 102 µmol/l (50–120 µmol/l)

Glucose (HbA 1c) 6.1 mmol/mol (< 53 mmol/mol)

C-reactive protein (CRP) 8 mg/l (0–10 mg/l)

Which of the following is the most likely diagnosis?

A Bipolar disorder

B Cannabis-induced psychosis

C Mefloquine-induced psychosis

D Primary brain tumour

E Viral encephalitis

Explanation 

C Mefloquine-induced psychosis

Psychiatric disturbance has been well described in patients who take mefloquine.
Manifestations, including generalised anxiety disorder, psychosis, schizophrenia, suicide
attempts, suicidal thoughts and self-endangering behaviour, have all been documented. It is
recommended that, in patients where a first-degree relative has a history of psychiatric
disorder, the risks of using mefloquine for malaria prophylaxis should be carefully considered
as they may outweigh the benefits.

The recommended prophylactic regimen is 250 mg weekly, starting 2–3 weeks before
entering a malaria area to assess tolerability, continuing throughout the time in the area and
for four weeks after leaving the area. Doxycycline or chloroquine may be appropriate for
patients who are unable to take mefloquine.

A Bipolar disorder

Although it is not impossible for a first presentation of bipolar disorder to occur at the age of
45, it is less likely than mefloquine psychosis, given the history of recently starting malaria
prophylaxis in this case.

B Cannabis-induced psychosis

We are given no indication that this patient has used cannabis. However, the scenario of
psychosis seen here may be similar to that seen with cannabis abuse.

D Primary brain tumour

A frontal lobe tumour may present with mood changes +/- psychotic features, although the
short-time course would be very unusual for a tumour to be the cause of his symptoms.

E Viral encephalitis

Viral encephalitis is more usually associated with decreased consciousness and memory loss,
rather than the psychotic features seen here.
51589

Rate this question:

Next Question

Previous Question Tag Question

Feedback End Review

Difficulty: Average

Peer Responses %
Q. Answered Flagged

Q1

Q2

Q3

Q4

Q5

Q6

Q7

Q8

Q9

 External Links

Public Health England. 2014. Malaria prevention guidelines for travellers from the UK.
gov.uk/government/publications/malaria-prevention-guidelines-for-travellers-from-the-uk
(https://www.gov.uk/government/publications/malaria-prevention-guidelines-for-
travellers-from-the-uk)
0:00:43/03:00:00

An 18-year-old man with learning difficulties is admitted to the Emergency Department. He


has hearing loss, cerebellar ataxia, dysarthria and dysphagia that has led to difficulties with
recurrent pneumonia. You read that he has Niemann–Pick disease type C (NPC).
Which of the following is the primary defect in NPC?

A Abnormal processing and transport of lipids

B Abnormal processing and transport of amino acids

C Failure of glucose sensing

D Hepatic nuclear factor (HNF)-1 alpha deficiency

E Succinate dehydrogenase deficiency

Explanation 

A Abnormal processing and transport of lipids

NPC is highly variable in its presentation, with some affected individuals presenting in infancy
and suffering rapidly progressive neurological dysfunction, while others may have onset of
their disease in adulthood. It is caused by a defect in the NPC1 protein which is a key
component of lysosomes, which are responsible for storage, transport and processing of
lipids.

B Abnormal processing and transport of amino acids

A number of amino acid metabolism disorders exist, but NPC is not one of them. Examples
include phenylketonuria, which occurs due to a defect in phenylalanine hydroxylase.

C Failure of glucose sensing

MODY-2 (maturity-onset diabetes of the young type 2) occurs due to a mutation in the
glucokinase gene, leading to an inability to properly sense high blood glucose levels.

D Hepatic nuclear factor (HNF)-1 alpha deficiency

HNF-1 alpha deficiency also leads to the development of MODY (type 3). HNF-1 alpha is a
transcription factor, and normal function is required for pancreatic beta cells to adequately
express glucose transporter (GLUT)-1 and GLUT-2. Patients with MODY-3 often respond to
low-dose sulfonylureas.
E Succinate dehydrogenase deficiency

This is a key mitochondrial enzyme, deficiencies of which lead to difficulties with maintaining
energy production.
51599

Rate this question:

Next Question

Previous Question Tag Question

Feedback End Review

Difficulty: Average

Peer Responses %

Q. Answered Flagged

Q1

Q2

Q3

Q4

Q5

Q6

Q7

Q8

Q9

0:00:43/03:00:00

A 67-year-old retired shipyard worker comes to the Respiratory Clinic with gradually
worsening shortness of breath over the past six months despite using a salbutamol inhaler
prescribed by his general practitioner (GP). He continues to smoke 25 cigarettes per day.
Examination reveals a blood pressure of 152/90 mmHg, and his pulse is 76 bpm and regular.
There is wheeze on auscultation of the chest.
Investigations:

Haemoglobin(Hb) 13.8 g/dl (13.5–17.5 g/dl)

White cell count (WCC) 7.7 × 10 9/l (4–11 × 10 9/l)

Platelets (PLT) 299 × 10 9/l (150–400 × 10 9/l)

Sodium (Na +) 143 mmol/l (135–145 mmol/l)

Potassium (K +) 4.3 mmol/l (3.5–5.0 mmol/l)

Creatinine (Cr) 95 µmol/l (50–120 µmol/l)

Obstructive picture (decreased forced


expiratory volume in 1 second
Spirometry
(FEV 1) against a relatively normal
forced vital capacity (FVC))

Chest X-ray Bilateral pleural plaques

Which of the following is the most likely cause of his shortness of breath?

A Asthma

B Asbestosis

C Chronic obstructive pulmonary disease (COPD)

D Idiopathic pulmonary fibrosis (IPF)

E Sarcoidosis

Explanation 

C Chronic obstructive pulmonary disease (COPD)


This patient is a heavy smoker, and the obstructive picture described on spirometry fits best
with a diagnosis of COPD. Pleural plaques are present in between 20% and 60% of patients
who have been exposed to asbestos, which fits with his previous occupation as a shipyard
worker. He could benefit from a long-acting anticholinergic inhaler added to his therapy.

A Asthma

With the history of heavy smoking seen here, COPD is much more likely to be the cause of
obstructive lung disease seen here vs asthma. It would also be important to perform
reversibility studies during the spirometry to exclude this as a diagnosis.

B Asbestosis

Asbestosis implies interstitial lung disease, which would be manifest by a restrictive pattern
on spirometry, rather than the obstructive picture seen here. Also there would likely be
fibrotic lung changes seen on the chest X-ray, alongside the pleural plaques.

D Idiopathic pulmonary fibrosis (IPF)

IPF is associated with a restrictive picture on lung function tests, and the pleural plaques are
suggestive of exposure to asbestos, which would not support a diagnosis of IPF.

E Sarcoidosis

Sarcoid-related lung disease is associated with a picture of restrictive lung disease, rather
than the obstructive picture seen here. Other features such as bilateral hilar
lymphadenopathy are also absent here.
51658

Rate this question:

Next Question

Previous Question Tag Question

Feedback End Review

Difficulty: Average

Peer Responses %
Q. Answered Flagged

Q1

Q2

Q3

Q4

Q5

Q6

Q7

Q8

Q9

0:00:43/03:00:00

A 17-year-old woman comes to the Haematology Clinic with her mother. Her main complaint
is that of menorrhagia, although she has also suffered prolonged bleeding after a tooth
extraction. Her mother tells you that she has also suffered from menorrhagia. Physical
examination in the clinic is unremarkable.
Investigations:

Haemoglobin (Hb) 12.0 g/dl (11.5–15.5 g/dl)

White cell count (WCC) 7.1 × 10 9/l (4–11 × 10 9/l)

Platelets (PLT) 232 × 10 9/l (150–400 × 10 9/l)

Prothrombin time (PT) 13.1 s (10.6–14.9 s)

Activated partial thromboplastin time (APTT) 32.0 s (23.0–35.0 s)

Which of the following is the most likely diagnosis?

A Antithrombin III deficiency

B Factor VII deficiency

C Haemophilia A

D Haemophilia B

E von Willebrand’s disease (vWD)

Explanation 

E von Willebrand’s disease (vWD)

vWD is associated with a bleeding tendency in the face of normal PT and APTT and follows
an autosomal dominant inheritance pattern, accounting for why the mother and daughter are
both affected here. Type 1 vWD accounts for 60–80% of cases and is associated with a
reduction in enzyme levels to between 20% and 45% of normal levels. Desmopressin
increases factor VIII and von Willebrand factor levels and may be used before planned
surgery.

A Antithrombin III deficiency


Antithrombin III deficiency is associated with an increased risk of venous thromboembolism.
It is usually autosomal dominant in its inheritance pattern and may also be acquired in
patients with proteinuric renal disease.

B Factor VII deficiency

Factor VII deficiency follows an autosomal recessive inheritance pattern and is much rarer
than vWD, meaning it is less likely to be the diagnosis here. It would be possible to
investigate further for this by assessing the partial thromboplastin time or plasma factor VII
activity.

C Haemophilia A

Haemophilia A is an X-linked disease, resulting in the deficiency of factor VIII. It leads to an


increased bleeding tendency. Most women do not exhibit symptoms of the disease, although
their APTT may be slightly outside the normal range.

D Haemophilia B

Like haemophilia A, haemophilia B is an X-linked disorder. It results in deficiency of factor IX,


leading to increased bleeding. Like haemophilia A, symptoms are rarely seen in women with
haemophilia B, meaning that it does not fit with the presentation in this clinical scenario.
51582

Rate this question:

Next Question

Previous Question Tag Question

Feedback End Review

Difficulty: Average

Peer Responses %

Q. Answered Flagged

Q1
Q. Answered Flagged

Q2

Q3

Q4

Q5

Q6

Q7

Q8 
0:00:43/03:00:00

A 70-year-old man who has ischaemic heart disease comes to the Emergency Department
with constipation. He has been unable to open his bowels for the past four days and is
complaining of intense lower abdominal pain. He is still passing flatus.
Medications include verapamil, atorvastatin, omeprazole, ramipril, aspirin and fluconazole for
recurrent Candida infection. Examination reveals a blood pressure of 142/84 mmHg, and the
pulse rate is 45 bpm and regular. His abdomen is soft, although there is generalised
tenderness. Bowel sounds are sluggish. Rectal examination reveals a small amount of hard
stool.

Investigations:

Haemoglobin (Hb) 13.0 g/dl (13.5–17.5 g/dl)

White cell count (WCC) 8.4 × 10 9/l (4–11 × 10 9/l)

Platelets (PLT) 209 × 10 9/l (150–400 × 10 9/l)

Sodium (Na +) 144 mmol/l (135–145 mmol/l)

Potassium (K +) 4.5 mmol/l (3.5–5.0 mmol/l)

Creatinine (Cr) 103 µmol/l (50–120 µmol/l)

An abdominal X-ray reveals extensive stool within the colon.

Which of the following is the most likely cause of this patient’s constipation?

A Atorvastatin

B Fluconazole

C Hypothyroidism

D Ischaemic colitis

E Omeprazole

Explanation 

B Fluconazole

Verapamil is extensively metabolised by CYP450 enzymes, including 3A4, and is a well-


known cause of constipation. Fluconazole, an azole antifungal, is a potent inhibitor of 3A4
and, in this case, would have led to accumulation of verapamil and the constipation seen
here, coupled with relative bradycardia.
A Atorvastatin

Atorvastatin is metabolised by CYP3A4, although it is not a potent inhibitor of the enzyme. It


is therefore not likely to be responsible for the constipation seen here.

C Hypothyroidism

Hypothyroidism is a cause of constipation, although there are no specific features of


hypothyroidism suggested in this clinical scenario. The relative bradycardia is more likely to
be related to verapamil excess.

D Ischaemic colitis

Ischaemic colitis is seen in patients with a history of ischaemic heart disease, although it
usually results in diarrhoea, rather than the constipation seen here.

E Omeprazole

Omeprazole is a cause of microscopic colitis, which results in chronic watery diarrhoea, as


opposed to constipation. It has no significant interaction potential with verapamil.
51588

Rate this question:

Next Question

Previous Question Tag Question

Feedback End Review

Difficulty: Average

Peer Responses %

Q. Answered Flagged

Q1
Q. Answered Flagged

Q2

Q3

Q4

Q5

Q6

Q7

Q8
0:00:43/03:00:00

A 45-year-old man presents to the Emergency Department with severe diarrhoea and lower
abdominal pain some seven hours after eating a Chinese meal. He has no vomiting. You
understand that six other people in his party have similar symptoms.
Examination reveals a blood pressure of 95/60 mmHg, and a pulse of 95 bpm. He is very
tender over his lower abdomen, with active bowel sounds.
Which of the following is the most likely cause of his symptoms?

A Bacillus cereus

B Campylobacter jejuni

C Escherichia coli

D Norovirus

E Staphylococcus aureus

Explanation 

A Bacillus cereus

This patient has diarrhoea around seven hours after a Chinese meal. Bacillus cereus, which
can cause either vomiting or diarrhoea, is the most likely cause, with reheated rice a possible
cause of infection. Management is supportive, with oral rehydration adequate in most cases.

B Campylobacter jejuni

Spread by contaminated meat or dairy products, this is thought to be the most common
cause of community-acquired bacterial gastroenteritis. The incubation period is between 1–7
days, and severity of the disease may be related to the number of bacteria ingested.
Management is supportive, with antibiotics usually only considered for severe symptoms.

C Escherichia coli

Enterotoxin-producing E. coli is the most common cause of traveller’s diarrhoea. The


incubation period would be expected to be longer than that seen here.

D Norovirus
Norovirus is a cause of outbreaks of vomiting and diarrhoea in winter. Symptoms usually last
for 24–48 hours. It would be highly unusual for a group of people to present at the same
time, only with diarrhoea, making it unlikely as the diagnosis here.

E Staphylococcus aureus

S. aureus gastroenteritis is because of a toxin, usually formed in dairy products which have
been incorrectly stored. It leads to explosive vomiting, which appears 1–6 hours after
exposure to the contaminated foodstuff.
51631

Rate this question:

Next Question

Previous Question Tag Question

Feedback End Review

Difficulty: Average

Peer Responses %

Q. Answered Flagged

Q1

Q2

Q3

Q4

Q5

Q6

Q7

Q8

Q9

0:00:43/03:00:00

A 49-year-old woman comes to the clinic for review. She complains of diplopia which
worsens throughout the day, and progressive ptosis which is making it difficult for her to
drive, particularly in the afternoons. She is finding it difficult to hold down a job due to
increasing tiredness.
On examination in the clinic, there is fatigable ptosis affecting both eyes. After application of
cold compresses, she can maintain upward gaze for much longer.
Which of the following is the most likely diagnosis?

A Guillain–Barré syndrome

B Lower motor neurone facial nerve palsy

C Myasthenia gravis

D Oculomotor nerve palsy

E Abducens nerve palsy

Explanation 

C Myasthenia gravis

Ocular muscle weakness, including ptosis, is a common presenting feature of myasthenia


gravis. Of note is that muscle functioning in myasthenia gravis improves with cold, which has
led to the use of cold compresses around the face before repeating an assessment of ptosis.
Because the test is easy to deploy, it has replaced the edrophonium test in many cases for
making the diagnosis.

A Guillain–Barré syndrome

Although the Miller–Fischer variant of Guillain–Barré syndrome may present with central
muscle weakness, the more prolonged time course of weakness here, without an obvious
trigger such as an episode of gastroenteritis, makes Guillain–Barré syndrome a very unlikely
diagnosis.

B Lower motor neurone facial nerve palsy

Isolated ptosis affecting both eyes does not fit with a lower motor neurone facial nerve palsy.

D Oculomotor nerve palsy


A complete oculomotor nerve palsy results in movement of the eye to the down-and-out
position. A partial oculomotor nerve palsy may result in ptosis, but it would be highly unusual
for this to be bilateral.

E Abducens nerve palsy

The abducens nerve controls the lateral rectus muscle. Abducens nerve palsy therefore leads
to diplopia on lateral gaze, not the scenario here with fatigable ptosis.
51645

Rate this question:

Next Question

Previous Question Tag Question

Feedback End Review

Difficulty: Average

Peer Responses %

Q. Answered Flagged

Q1

Q2

Q3

Q4

Q5

Q6

Q7

Q8

Q9

0:00:43/03:00:00

A 25-year-old man with von Hippel–Lindau (VHL) disease comes to the clinic with decreased
vision affecting his left eye. He tells you this has developed gradually, but he is now finding it
very difficult to drive. His visual acuity is 6/6 in the right eye, and 6/36 in the left eye.
Which of the following is the most likely cause of his visual disturbance?

A Choroidal haemangioma

B Haemangioblastoma

C Ocular melanoma

D Optic nerve glioma

E Rentinoblastoma

Explanation 

B Haemangioblastoma

These are common in VHL disease, being found in 60% of patients. They can occur in children
aged under ten years, although the average age at presentation is 25 years. They are usually
asymptomatic until complications, such as subretinal oedema, retinal detachment or
glaucoma, develop, which lead to visual loss. Both laser photocoagulation and anti-vascular
endothelial growth factor are potential interventions. Other features outside the central
nervous system include phaeochromocytoma, the development of renal cysts and, in some
cases, renal carcinoma (renal abnormalities are seen in 60% of patients).

A Choroidal haemangioma

Choroidal haemangiomas are benign vascular tumours which occur with greater frequency in
conditions associated with vascular malformations such as Sturge–Weber syndrome.

C Ocular melanoma

Ocular melanomas do not occur with increased frequency in patients with VHL, and there is
no mention of other risk factors for the development of melanoma here.

D Optic nerve glioma

Optic nerve gliomas are rare benign tumours that can occur along the optic nerve or affect
the optic chiasm. They are associated with neurofibromatosis type 1.
E Rentinoblastoma

Retinoblastoma presents at an average age of 12 months for heritable disease, which is


usually bilateral. Non-heritable disease presents at an average age of 24 months, making it
very unlikely to be the cause of this patient’s visual loss.
51652

Rate this question:

Next Question

Previous Question Tag Question

Feedback End Review

Difficulty: Average

Peer Responses %

Q. Answered Flagged

Q1

Q2

Q3

Q4

Q5

Q6

Q7

Q8

Q9

0:00:43/03:00:00

A 28-year-old woman who has type 1 diabetes comes to the clinic for review. She has been
feeling tired and lethargic over the past few months and is increasingly off her food. She has
reduced her insulin recently because of frequent episodes of hypoglycaemia.
Examination reveals a blood pressure of 95/60 mmHg, and her pulse is 80 bpm and regular.
She is thin, with a body mass index (BMI) of 19.0 kg/m 2.
Investigations:

Haemoglobin (Hb) 10.4 g/dl (11.5–15.5 g/dl)

White cell count (WCC) 7.0 × 10 9/l (4–11 × 10 9/l)

Platelets (PLT) 171 × 10 9/l (150–400 × 10 9/l)

Sodium (Na +) 130 mmol/l (135–145 mmol/l)

Potassium (K +) 5.5 mmol/l (3.5–5.0 mmol/l)

Creatinine (Cr) 122 µmol/l (50–120 µmol/l)

Glucose 4.5 mmol/l (3.5–5.5 mmol/l)

Urine sodium 30 mmol/l (50–200 mmol/l)

Which of the following is the most likely diagnosis?

A Adrenal insufficiency

B Hypothyroidism

C Renal tubular acidosis type 1 (RTA1)

D Renal tubular acidosis type 4 (RTA4)

E Syndrome of inappropriate antidiuretic secretion (SIADH)

Explanation 

A Adrenal insufficiency

The scenario here with hypotension, hyponatraemia, borderline elevated potassium levels,
and decreased urinary sodium excretion fits with a diagnosis of adrenal insufficiency, most
likely autoimmune in aetiology. The hypoglycaemia is also consistent with the diagnosis.
Treatment of adrenal insufficiency is with intravenous (IV) hydrocortisone initially, converting
to oral twice-daily dosing, without waiting for a confirmatory cosyntropin test if the index of
suspicion is high.

B Hypothyroidism

Although hypothyroidism does lead to hyponatraemia, it does not fit with this patient’s
weight, hyperkalaemia, and decreased requirements for insulin, which is more in keeping with
a diagnosis of adrenal insufficiency.

C Renal tubular acidosis type 1 (RTA1)

RTA1 leads to metabolic acidosis and hypokalaemia, rather than the scenario here with
hyperkalaemia and decreased urinary sodium excretion.

D Renal tubular acidosis type 4 (RTA4)

RTA4 is associated with deficient aldosterone action, which does lead to hyperkalaemia and
decreased urinary sodium excretion, although this would not fit with the other features of
adrenal insufficiency, such as hypoglycaemia, which are seen here.

E Syndrome of inappropriate antidiuretic secretion (SIADH)

SIADH is not associated with hyperkalaemia and does not fit with the other features such as
tiredness, lethargy and hypoglycaemia seen here.
51648

Rate this question:

Next Question

Previous Question Tag Question

Feedback End Review

Difficulty: Average

Peer Responses %
Q. Answered Flagged

Q1

Q2

Q3

Q4

Q5

Q6

Q7

Q8

Q9 
0:00:43/03:00:00

A 32-year-old woman with inflammatory bowel disease is prescribed ciclosporin as


maintenance therapy for Crohn’s disease. She is interested in hearing about the mode of
action (MoA) of ciclosporin.
Which of the following correctly reflects the MoA of ciclosporin?

A CD20 receptor inhibition

B Inhibition of interleukin (IL)-2 production

C Inhibition of IL-6 production

D Integrin receptor antagonism

E Tumour necrosis factor (TNF)-alpha receptor inhibition

Explanation 

B Inhibition of interleukin (IL)-2 production

Ciclosporin (also known as ciclosporin A) is a cyclic polypeptide consisting of 11 amino acids.


At the cellular level, ciclosporin inhibits the production and release of lymphokines, including
IL-2. Ciclosporin appears to block resting lymphocytes in the G0 or G1 phase of the cell cycle
and inhibits the antigen-triggered release of lymphokines by activated T-cells.

A CD20 receptor inhibition

Anti-CD20 agents, such as rituximab, lead to depletion of mature B-cells and a reduction in
antibody production. They are used in the management of inflammatory disorders such as
rheumatoid arthritis (RA).

C Inhibition of IL-6 production

Toclizumab inhibits the binding of IL-6 to its receptor. Elevated IL-6 levels are recognised to
play a role in the pathogenesis of RA, and toclizumab is licensed for the treatment of RA
where other therapeutic interventions have not been successful.

D Integrin receptor antagonism

Integrin receptor antagonists are used in the treatment of Crohn’s disease; vedolizumab is
one example. They block the passage of white cells through the gut mucosa, breaking the
cycle of inflammation.
E Tumour necrosis factor (TNF)-alpha receptor inhibition

A range of agents such as adalimumab which inhibit the TNF-alpha axis are licensed for the
treatment of inflammatory bowel disease. Ciclosporin does not work via the TNF-alpha
pathway.
51595

Rate this question:

Next Question

Previous Question Tag Question

Feedback End Review

Difficulty: Average

Peer Responses %

Q. Answered Flagged

Q1

Q2

Q3

Q4

Q5

Q6

Q7

Q8

Q9

0:00:43/03:00:00

A 45-year-old man presents to the Endocrine Clinic with chronic tiredness and loss of libido.
His general practitioner (GP) suspects he may have a pituitary hormone deficiency.
Which of the following pituitary hormones is under constant inhibition?

A Adrenocorticotrophic hormone (ACTH)

B Antidiuretic hormone (ADH)

C Luteinising hormone (LH)

D Prolactin

E Thyroid-stimulating hormone (TSH)

Explanation 

D Prolactin

Prolactin secretion is under constant inhibition from dopamine. A progressive rise in prolactin
is seen during pregnancy, and the levels then fall back to the normal range around six weeks
after delivery, even if the woman is breastfeeding. Dopamine agonist therapy can be utilised
to suppress prolactin production in patients presenting with prolactinoma, which may be the
cause of the chronic tiredness and loss of libido seen here.

A Adrenocorticotrophic hormone (ACTH)

ACTH production is pulsatile, with the half-life being around ten minutes. Production of ACTH
is under control of corticotrophin-releasing hormone (CRH), and the production of CRH is
under negative feedback from any elevation in glucocorticoid levels.

B Antidiuretic hormone (ADH)

ADH is secreted via the posterior pituitary gland, and secretion increases in response to
hypovolaemia and/or increased osmolality. It drives an increased expression of aquaporin-2
channels in the collecting duct, which, in turn, leads to increased reabsorption of water by the
kidney and vasoconstriction.

C Luteinising hormone (LH)


Production of LH is cyclical, with an LH surge occurring 24–48 hours before ovulation. LH can
be measured in the urine with ovulation predictor kits, which are commercially available.
Persistently high LH levels are seen in gonadal dysgenesis, Turner syndrome, and the post-
menopausal state.

E Thyroid-stimulating hormone (TSH)

TSH is released in a circadian manner and is pulsatile. It is under negative feedback from T3
and T4, which regulate its release, rather than being under constant suppression.
51601

Rate this question:

Next Question

Previous Question Tag Question

Feedback End Review

Difficulty: Average

Peer Responses %

Q. Answered Flagged

Q1

Q2

Q3

Q4

Q5

Q6

Q7

Q8

Q9

0:00:43/03:00:00

A 22-year-old student is found unconscious in his university accommodation. He has only


recently moved into the flat, and the others in the accommodation know nothing about him.
On arrival in the Emergency Department, his Glasgow Coma Scale (GCS) score is 10. His
blood pressure is 90/50 mmHg and respiratory rate 35 breaths per minute, and you note that
he smells of alcohol. He moans when you are examining his abdomen. He looks of normal
weight. A glucose finger-prick test in the ambulance is measured at 37 mmol/l.
Which of the following is the most likely diagnosis?

A Glucagonoma

B Latent autoimmune diabetes of adulthood (LADA)

C Maturity-onset diabetes of the young (MODY)

D Type 1 diabetes

E Type 2 diabetes

Explanation 

D Type 1 diabetes

This patient’s glucose, coupled with hypotension and probable respiratory compensation for
metabolic acidosis, is suggestive of diabetic ketoacidosis (DKA). Intravenous fluid
replacement, followed by a fixed-rate insulin infusion, is the most appropriate intervention.
Alcohol consumption increases the risks for both hypoglycaemia and DKA in patients with
type 1 diabetes.

A Glucagonoma

Glucagonomas do lead to hyperglycaemia, but patients present with a picture of weight loss,
diarrhoea and a characteristic skin rash (necrolytic migratory erythema). They are also
vanishingly rare.

B Latent autoimmune diabetes of adulthood (LADA)

LADA, colloquially known as slow-burn type 1 diabetes, is unlikely to present with DKA as the
initial feature. Patients are more likely to present with chronic features of hyperglycaemia
such as recurrent balanitis.
C Maturity-onset diabetes of the young (MODY)

MODY does not usually present with DKA, and patients are most likely to present with
symptoms associated with chronic hyperglycaemia such as recurrent balanitis and/or
superficial skin infections.

E Type 2 diabetes

Type 2 diabetes is much less likely, given this patient’s normal weight, young age and
presentation with DKA, all of which count against type 2 diabetes as the underlying
diagnosis.
51613

Rate this question:

Next Question

Previous Question Tag Question

Feedback End Review

Difficulty: Average

Peer Responses %

Q. Answered Flagged

Q1

Q2

Q3

Q4

Q5

Q6

Q7
Q. Answered Flagged

Q8
0:00:43/03:00:00

A 78-year-old man comes to the clinic for review. Most recently, he has begun feeling
nauseated and lethargic, and there has been a slow increase in his lithium levels over the past
few months. He has lost weight for no specific reason. He takes lithium for bipolar disorder
and has done so for 35 years.
His blood pressure is 122/82 mmHg, and his pulse is 70 bpm and regular. His body mass index
(BMI) is 22 kg/m 2.
Investigations:

Haemoglobin (Hb) 12.9 g/dl (13.5–17.5 g/dl)

White cell count (WCC) 6.7 × 10 9/l (4–11 × 10 9/l)

Platelets (PLT) 209 × 10 9/l (150–400 × 10 9/l)

Sodium (Na +) 138 mmol/l (135–145 mmol/l)

Potassium (K +) 3.8 mmol/l (3.5–5.0 mmol/l)

105 µmol/l (up from 95 µmol/l


Creatinine (Cr) (50–120 µmol/l)
some six months earlier)

Which of the following is the most likely cause of his increase in lithium levels?

A Age-related decreased hepatic synthetic function

B Decreased lean body mass

C Recent use of clarithromycin

D Recent increase in oral fluid intake

E Reduction in glomerular filtration rate (GFR)

Explanation 

B Decreased lean body mass

Reduced volume of distribution, because of decreased lean body mass and decreased total
body water, can lead to a slow rise in lithium levels. This scenario fits with the gradual loss of
weight over the past few months.

A Age-related decreased hepatic synthetic function


Lithium is predominantly excreted by the kidneys, and not metabolised by the liver; hence,
hepatic synthetic function does not impact on lithium levels.

C Recent use of clarithromycin

Clarithromycin is a CYP3A4 inhibitor, which inhibits the metabolism of agents such as


ciclosporin. It does not impact on metabolism of lithium.

D Recent increase in oral fluid intake

A decrease in oral fluid intake would drive a rise in lithium levels; it would be expected that
increasing fluid intake would drive an increase in the volume of distribution and reduce
lithium levels.

E Reduction in glomerular filtration rate (GFR)

The change in creatinine levels seen here is minimal and is highly unlikely to impact on lithium
levels.
51622

Rate this question:

Next Question

Previous Question Tag Question

Feedback End Review

Difficulty: Average

Peer Responses %

Q. Answered Flagged

Q1

Q2

Q3
Q. Answered Flagged

Q4

Q5

Q6

Q7

Q8 
0:00:43/03:00:00

A 39-year-old woman presents to the Emergency Department as a general practitioner (GP)


referral with rapidly worsening renal function, nosebleeds and haemoptysis.
Her blood pressure is 152/88 mmHg, and her pulse is 72 bpm and regular. There are scattered
crackles on auscultation of the chest.
Investigations:

Haemoglobin (Hb) 10.9 g/dl (11.5–15.5 g/dl)

White cell count (WCC) 12.1 × 10 9/l (4–11 × 10 9/l)

Platelets (PLT) 342 × 109/l (150–400 × 10 9/l)

Sodium (Na +) 144 mmol/l (135–145 mmol/l)

Potassium (K +) 5.2 mmol/l (3.5–5.0 mmol/l)

Creatinine (Cr) 195 µmol/l (50–120 µmol/l)

Anti-PR3 antibody Positive

Chest X-ray Bilateral pulmonary infiltrates

Which of the following is the most likely diagnosis?

A Anti-glomerular basement membrane (GBM) disease

B Eosinophilic granulomatosis

C Granulomatosis with polyangiitis

D Idiopathic pulmonary fibrosis

E Microscopic polyangiitis

Explanation 

C Granulomatosis with polyangiitis

In patients with granulomatosis with polyangiitis (the likely diagnosis here, given the evidence
of sinusitis, pulmonary infiltration and renal impairment), around 95% of patients are ANCA-
positive. Around 65% of patients are also positive for anti-PR3. This condition, formerly
known as Wegener’s granulomatosis, is associated with both pulmonary and renal vasculitis.
When vasculitis is present in the lungs, it can lead to infiltration on the chest X-ray and
increased transfer factor, consistent with blood in the alveolar space.
A Anti-glomerular basement membrane (GBM) disease

This is not associated with sinusitis. Anti-GBM antibodies are characteristic of the condition,
rather than the anti-PR3 antibodies seen here.

B Eosinophilic granulomatosis

Eosinophilic granulomatosis is associated with an elevated eosinophil count (not seen here)
and symptoms consistent with poorly controlled asthma, rather than the haemoptysis and
pulmonary infiltrates seen here.

D Idiopathic pulmonary fibrosis

Idiopathic pulmonary fibrosis may be associated with the pulmonary infiltrates seen here on
chest X-ray, although it would not fit with the worsening renal function and the anti-PR3
antibodies.

E Microscopic polyangiitis

Patients with microscopic polyangiitis are more likely to present with anti-MPO antibodies, vs
the anti-PR3 antibodies seen here, although the condition is known to be associated with
pulmonary haemorrhage.
51660

Rate this question:

Next Question

Previous Question Tag Question

Feedback End Review

Difficulty: Average

Peer Responses %

Q. Answered Flagged
Q. Answered Flagged

Q1

Q2

Q3

Q4

Q5

Q6

Q7

Q8
0:00:43/03:00:00

A 54-year-old man who has been working on a water treatment project in Egypt comes to
the clinic on his return to the United Kingdom. He has been feeling tired and listless, with
intermittent fevers and abdominal pain, over the past few weeks. Infection with Schistosoma
mansoni is diagnosed.
Which of the following is the biggest risk to the patient over the longer term?

A Bladder carcinoma

B Chronic kidney disease

C Hepatic fibrosis

D Hydronephrosis

E Renal stones

Explanation 

C Hepatic fibrosis

S. mansoni is a human parasite. It is responsible for infection of the hepatic portal venous
plexus and veins around the small intestines. Eggs produced by the parasite are highly
antigenic; they produce a granulomatous response within hepatic cells which drives fibrosis.
Over the longer term, this fibrosis can lead to portal hypertension and associated
complications.

A Bladder carcinoma

Bladder carcinoma is known to be associated with Schistosoma haematobium, which infects


the venous plexus around the bladder. This is a particular problem in Egypt where high levels
of tobacco smoking are thought to be synergistic with S. haematobium infection in leading to
the development of cancers.

B Chronic kidney disease

Chronic kidney disease is a consequence of S. haematobium, not S. mansoni, infection. This


may be related to stone formation or due to ureteric fibrosis leading to obstructive kidney
disease.

D Hydronephrosis
Hydronephrosis is seen in association with S. haematobium, not S. mansoni, infection. It may
reverse on treatment of the disease, suggesting that oedema plays a role in ureteric
obstruction.

E Renal stones

Renal stones do occur in patients with S. haematobium infection. This may be due to chronic
urinary stasis and secondary bacterial infection.
51637

Rate this question:

Next Question

Previous Question Tag Question

Feedback End Review

Difficulty: Average

Peer Responses %

Q. Answered Flagged

Q1

Q2

Q3

Q4

Q5

Q6

Q7

Q8

Q9

0:00:43/03:00:00

A 58-year-old woman is reviewed on the ward some four hours after an endoscopic
retrograde cholangiopancreatography (ERCP) for biliary sepsis with ductal stones. She is
treated with intravenous (IV) ciprofloxacin and IV fluid replacement. Observations reveal a
temperature of 37.6 °C. Her blood pressure is 100/60 mmHg, and her pulse is 95 bpm and
regular.
Investigations:

Haemoglobin (Hb) 13.4 g/dl (11.5–15.5 g/dl)

White cell count (WCC) 11.9 × 10 9/l (4–11 × 10 9/l)

Platelets (PLT) 102 × 10 9/l (150–400 × 10 9/l)

Sodium (Na +) 142 mmol/l (135–145 mmol/l)

Potassium (K +) 4.5 mmol/l (3.5–5.0 mmol/l)

Creatinine (Cr) 134 µmol/l (5–120 mmol/l)

Prothrombin time (PT) 20.1 s (10.6–14.9 s)

Activated partial thromboplastin time (APTT) 42.4 s (23.0–35.0 s)

Fibrinogen 120 mg/dl (125–300 mg/dl)

D-dimer 781 µg/ml < 0.5 µg/ml

Which of the following is the most likely cause of the abnormal clotting results seen here?

A Anti-phospholipid antibody syndrome

B Disseminated intravascular coagulation (DIC)

C Factor V Leiden mutation

D Inappropriate fluid replacement

E Venous thromboembolism

Explanation 

B Disseminated intravascular coagulation (DIC)

This patient’s blood picture with low platelets, clotting values outside the normal range,
decreased fibrinogen levels and elevated D-dimer levels fits with a diagnosis of DIC as a
result of Gram-negative sepsis. It is expected, now that the biliary obstruction has been
removed and the patient is treated with antibiotics, that the clotting abnormality will steadily
resolve.

A Anti-phospholipid antibody syndrome

This fits well with a prolonged APTT, but it would not fit with the other clotting abnormalities
seen here, which are much more likely to be connected to the acute sepsis.

C Factor V Leiden mutation

The factor V Leiden mutation is associated with thrombophilia and does not fit with the
picture here of DIC associated with acute sepsis.

D Inappropriate fluid replacement

Inappropriate fluid replacement is more likely to drive abnormalities in electrolytes, rather


than the changes in platelets and clotting factors seen here.

E Venous thromboembolism

While venous thromboembolism results in elevated D-dimer levels, it would not fit with the
other features in this scenario, including features of acute sepsis and low platelets.
51581

Rate this question:

Next Question

Previous Question Tag Question

Feedback End Review

Difficulty: Average

Peer Responses %

Q. Answered Flagged
Q. Answered Flagged

Q1

Q2

Q3

Q4

Q5

Q6

Q7

Q8
0:00:43/03:00:00

A 42-year-old woman presents to the Rheumatology Clinic for review. She has progressively
worsening shortness of breath and painful proximal muscle weakness, which has developed
over the past six months. Examination reveals a blood pressure of 118/82 mmHg. Her pulse is
74 bpm and regular. There is obvious proximal muscle weakness on clinical examination, and
inspiratory crackles in both bases on auscultation. Pulmonary function testing reveals a
restrictive picture.
Which of the following autoantibodies is most likely to be positive in this patient?

A Anti-Jo1

B Anti-mitochondrial

C Anti-smooth muscle

D c-ANCA

E p-ANCA

Explanation 

A Anti-Jo1

Anti-Jo1 is seen both in patients with polymyositis and in those with pulmonary fibrosis, both
of which are potential diagnoses here. In total, they are found in 30% of those with myositis
and in 60–70% of those with idiopathic pulmonary fibrosis.

B Anti-mitochondrial

Over 95% of patients with primary biliary cirrhosis are found to have anti-mitochondrial
antibodies where symptoms of increasing tiredness, itching and progressive liver disease are
apparent.

C Anti-smooth muscle

Anti-smooth muscle antibodies are classically seen in patients with chronic autoimmune
hepatitis where they may be directed against actin, troponin or tropomyosin.

D c-ANCA
c-ANCA autoantibodies are classically seen in patients with granulomatosis with polyangiitis
where patients present with evidence of midline sinusitis, haemorrhagic pulmonary vasculitis
and renal vasculitis.

E p-ANCA

Positive p-ANCA is seen in up to half of patients with eosinophilic granulomatosis with


polyangiitis, in primary sclerosing cholangitis and in some patients with rheumatoid arthritis,
none of which are suggested by the diagnosis here.
51604

Rate this question:

Next Question

Previous Question Tag Question

Feedback End Review

Difficulty: Average

Peer Responses %

Q. Answered Flagged

Q1

Q2

Q3

Q4

Q5

Q6

Q7

Q8

Q9

0:00:43/03:00:00

A 52-year-old woman with Graves’ disease is referred for intervention to manage her
thyrotoxicosis. Her thyroid disease has been stabilised on a block–replace regimen with oral
thyroxine and carbimazole. She has significant Graves’ ophthalmopathy with proptosis and lid
lag.
Which of the following interventions to manage her disease is most likely to worsen her
ophthalmopathy?

A Azathioprine

B Prednisolone

C Radioiodine

D Rituximab

E Thyroidectomy

Explanation 

C Radioiodine

Radioiodine is recognised to worsen Graves’ ophthalmopathy when used as a treatment for


thyroid disease. This may be due to decompensation of thyroid disease with unrecognised
hypothyroidism post-procedure, or due to other factors such as release of thyroid antigens
post-treatment. The risk of worsening disease can be minimised by use of a block–replace
regimen.

A Azathioprine

Azathioprine is not effective as a treatment for Graves’ eye disease. It has no effect on any of
the clinical parameters associated with eye disease, although it does decrease thyroid
microsomal antibodies.

B Prednisolone

Prednisolone is used as an acute treatment for decompensated thyroid eye disease, while
second-line immunotherapy or other options, such as ocular radiotherapy or decompression
surgery, can be instigated.

D Rituximab
Rituximab has mixed results as a treatment for Graves’ eye disease. Although some trials
have shown positive benefit on symptoms of thyroid eye disease, others have demonstrated
equivocal results. It is not used as a treatment for thyrotoxicosis.

E Thyroidectomy

Thyroidectomy is not associated with the same risk of worsening thyroid eye disease as
radioiodine if regular monitoring of thyroid function is instituted and thyroxine titrated post-
operatively to avoid transient hypothyroidism.
51620

Rate this question:

Next Question

Previous Question Tag Question

Feedback End Review

Difficulty: Average

Peer Responses %

Q. Answered Flagged

Q1

Q2

Q3

Q4

Q5

Q6

Q7

Q8

Q9

0:00:43/03:00:00

You are asked to review a 25-year-old woman who has had 12 sutures to a scalp wound she
sustained, having fallen on a night out with friends. She has drunk three large glasses of wine.
The Emergency Department staff have noted that her left pupil is 1 mm larger in diameter
than the right. Both react normally to light and accommodation.
Which of the following is the most likely explanation for her pupil sizes?

A Argyll Robertson pupil

B Holmes Adie pupil

C Optic neuritis

D Skull base fracture

E Variant of normal

Explanation 

E Variant of normal

An uncomplicated scalp wound is highly unlikely in a well 25-year-old to be associated with


significant underlying pathology. A difference in pupil size of 1 mm or less in diameter is
considered a normal variant if both pupils are reactive to light and accommodation.

A Argyll Robertson pupil

These are usually bilateral (although they can be asymmetrical in diameter). It is


characterised by a tonically small pupil that reacts poorly, or not at all, to light, but briskly to
accommodation (light-near dissociation). The pupils do not dilate well with pharmacological
agents. This condition is strongly associated with tertiary syphilis.

B Holmes Adie pupil

This is a common finding in younger women. The pupil is dilated in the early stages and may
also be irregular. Like the Argyll Robertson pupil, it reacts slowly to light, but briskly to
accommodation. Eighty per cent are unilateral, and once the pupil has constricted, it remains
small for an abnormally long time. The Holmes Adie pupil is considered a variation of normal,
but in rare cases, it is the result of an abnormality affecting the efferent parasympathetic
pathway.

C Optic neuritis
Optic neuritis is associated with an afferent pupillary defect, which does not fit with the
examination findings seen here.

D Skull base fracture

Base of skull fracture seems unlikely, given the mode of injury described here. In addition, it
may be associated with bruising around the eyes or cerebrospinal fluid leakage from the ears,
neither of which are seen here. An efferent pupillary defect is usually found.
51639

Rate this question:

Next Question

Previous Question Tag Question

Feedback End Review

Difficulty: Average

Peer Responses %

Q. Answered Flagged

Q1

Q2

Q3

Q4

Q5

Q6

Q7

Q8

Q9

0:00:43/03:00:00

A 32-year-old woman is admitted to the Emergency Department from the Transplant Clinic
with a rise in her creatinine level despite optimised levels of ciclosporin. She is six weeks
post-renal transplantation, and rejection is suspected.
Which of the following factors is likely to have the biggest impact on rejection in this
patient?

A Cold ischaemia time > six hours in the donor kidney

B Human leukocyte antigen (HLA) matching

C Number of blood transfusions in the donor

D Recipient diabetic nephropathy

E Sex matching

Explanation 

B Human leukocyte antigen (HLA) matching

The collaborative transplant study has shown that matching for HLA DR and B antigens has a
significant impact on graft survival. The maximal negative effect of DR mismatching is seen
around six months post-transplantation. With respect to B mismatching, the maximal effect
on rejection appears to occur around two years post-transplantation.

A Cold ischaemia time > six hours in the donor kidney

Cold ischaemia time of > 24 hours is associated with a significant increase in graft failure.
Where possible, time to transplantation is usually minimised to within 12 hours.

C Number of blood transfusions in the donor

The number of blood transfusions in the recipient is a significant risk factor for the
development of rejection, because it increases the risk of graft-versus-host disease, although
it clearly does not impact as much as HLA mismatching which carries a pre-determined fixed
risk of rejection.

D Recipient diabetic nephropathy


Recipient diabetic nephropathy implies poor diabetes control, and if this is not corrected,
then there is an increased risk for transplant failure. Kidney/pancreas transplants are
associated with a lower risk of graft failure.

E Sex matching

Limited data suggest that sex mismatching may be associated with improved graft function
at one year, although the data are not definitive.
51650

Rate this question:

Next Question

Previous Question Tag Question

Feedback End Review

Difficulty: Average

Peer Responses %

Q. Answered Flagged

Q1

Q2

Q3

Q4

Q5

Q6

Q7

Q8

Q9

0:00:43/03:00:00

A 25-year-old rugby player, who has suffered multiple shoulder dislocations, comes to the
Neurology Clinic for review. He has an area of numbness over the regimental badge area.
Which nerve is most likely to have been damaged?

A Axillary nerve

B Second intercostal nerve

C Long thoracic nerve

D Radial nerve

E Suprascapular nerve

Explanation 

A Axillary nerve

The axillary nerve is often damaged by recurrent shoulder dislocation. Damage can lead to
loss of sensation over the regimental badge area and weakness of the deltoid muscle.

B Second intercostal nerve

Thoracic intercostal nerves supply the intercostal muscles and sensation over the skin over
the chest.

C Long thoracic nerve

The long thoracic nerve supplies the serratus anterior muscle; sensory symptoms are often
minimal, although patients may complain of burning pain over the posterior aspect of the
scapula if there is irritation of the nerve.

D Radial nerve

The radial nerve is responsible for sensory and motor supply distal to the regimental badge
area. It innervates the medial and lateral heads of the triceps, as well as several forearm
muscles.

E Suprascapular nerve
The suprascapular nerve supplies the supraspinatus and infraspinatus muscles, and sensory
innervation to the acromioclavicular and glenohumeral joints.
51646

Rate this question:

Next Question

Previous Question Tag Question

Feedback End Review

Difficulty: Average

Peer Responses %

Q. Answered Flagged

Q1

Q2

Q3

Q4

Q5

Q6

Q7

Q8

Q9

0:00:43/03:00:00

A 25-year-old woman comes to the Neurology Clinic some four weeks after suffering a rear
shunt car accident. After the accident, she has complained of posterior neck pain and says
when she bends her neck forward, she gets electric shock-like sensations in her limbs. She
has also noticed clumsiness in her hands and that her legs have become increasingly stiff and
weak.
Neurological examination reveals 4/5 power weakness affecting both lower limbs, more
marked on the left than on the right, with increased tone and an upgoing plantar on the left.
An original neck X-ray was normal.

Which of the following is the most likely diagnosis?

A Cervical disc prolapse

B Fibromyalgia

C Missed vertebral fracture

D Simple whiplash

E Syringomyelia

Explanation 

A Cervical disc prolapse

The upper motor neurone symptoms seen here affecting both lower limbs and the upgoing
plantar on the left are consistent with cervical myelopathy related to a disc prolapse.
Lhermitte’s sign with pain in the limbs on flexion of the neck is also an important finding. A
disc prolapse would not have been seen on the original plain film of the neck. A magnetic
resonance imaging (MRI) scan is indicated to confirm the diagnosis.

B Fibromyalgia

This is associated with multiple points of muscular pain, which do not correspond to any
likely physical injury. It is seen more commonly in patients with underlying psychiatric
disease, and investigations of affected joints or muscle groups are usually entirely normal.

C Missed vertebral fracture

Although it is not impossible that a fracture has been missed, it is more likely, given that the
original X-ray was normal, that the symptoms are due to a disc prolapse. An MRI scan will
help rule this out and is the next logical step anyway.
D Simple whiplash

Simple whiplash would not account for the neurological findings seen here but would be
consistent with the posterior neck pain.

E Syringomyelia

Syringomyelia develops gradually over a number of weeks but can occur after neck trauma.
Classically, syringomyelia is associated with weakness in the hands and sensation loss is
experienced in a shawl-like distribution over the arms, shoulders and upper body.
51641

Rate this question:

Next Question

Previous Question Tag Question

Feedback End Review

Difficulty: Average

Peer Responses %

Q. Answered Flagged

Q1

Q2

Q3

Q4

Q5

Q6

Q7

Q8
Q. Answered Flagged 
0:00:43/03:00:00

A 23-year-old woman is reviewed in the Cancer Genetics Clinic. She is concerned about her
risk of cancer, as both her father and uncle died of pancreatic cancer and her 37-year-old
older brother has recently been diagnosed with the condition. She is found to carry the
BRCA1 gene mutation.
Which of the following cancers is she most at risk of?

A Acute myeloid leukaemia

B Colon cancer

C Glioblastoma multiforme

D Osteosarcoma

E Ovarian cancer

Explanation 

E Ovarian cancer

The risk of breast and ovarian cancers is significantly increased in patients with the BRCA1
gene mutation. By the age of 70, breast cancer risk in women is estimated to be 72.8% (95%
confidence interval (CI) 67.9–77.7%) and ovarian cancer risk 40.7% (95% CI 35.7–45.6%). The
risk for a second primary breast cancer by the age of 70 is 40.5% (95% CI 34.1–47.0%) in
women with the BRCA1 mutation.

A Acute myeloid leukaemia

Acute myeloid leukaemia is seen with increased frequency in patients with the BRCA2 gene
mutation. It occurs as a consequence of Fanconi’s anaemia.

B Colon cancer

The risk of colon cancer is increased by a factor of two in patients with the BRCA1 mutation,
although this is still a lesser overall risk, compared to the risk of ovarian cancer.

C Glioblastoma multiforme

Glioblastoma multiforme is rare and is not related to BRCA mutation. It occurs with increased
frequency in older patients and may be associated with exposure to ionising radiation.
D Osteosarcoma

Osteosarcoma is associated with long-standing Paget’s disease, not with the BRCA gene
mutation.
51584

Rate this question:

Next Question

Previous Question Tag Question

Feedback End Review

Difficulty: Average

Peer Responses %

Q. Answered Flagged

Q1

Q2

Q3

Q4

Q5

Q6

Q7

Q8

Q9

 External Links

Brose, M S, Rebbeck, T R, Calzone, K A, et al. 2002. Cancer risk estimates for BRCA1 mutation carriers ide…
ncbi.nlm.nih.gov/pubmed/12237282
(https://www.ncbi.nlm.nih.gov/pubmed/12237282)
0:00:43/03:00:00

A 24-year-old woman, who is 16 weeks’ pregnant with her first child, comes to the
Emergency Department for review. She has been visiting her sister, whose youngest child has
been diagnosed the previous evening with chickenpox. Physical examination is normal, and
urgent antibody testing for varicella-zoster virus (VZV) is negative.
Which of the following is the most appropriate next step?

A Intravenous (IV) aciclovir

B Oral aciclovir

C Reassurance

D VZV immunoglobulin (VZIG)

E VZV vaccination

Explanation 

D VZV immunoglobulin (VZIG)

The Royal College of Obstetricians and Gynaecologists (RCOG) green-top guidelines are
clear that non-immune women should be given immunoglobulin. If the pregnant woman is
not immune to VZV and she has had a significant exposure, she should be offered VZIG as
soon as possible. VZIG is effective when given up to ten days after contact (in the case of
continuous exposure, this is defined as ten days from the appearance of the rash in the index
case).

A Intravenous (IV) aciclovir

IV aciclovir is recommended for all pregnant patients presenting with severe chickenpox.

B Oral aciclovir

Oral aciclovir is recommended when patients present within 24 hours of onset of the rash.

C Reassurance

Reassurance is not appropriate as a solo strategy because of the significant risks of maternal
chickenpox infection. Oral aciclovir should be prescribed for pregnant women with
chickenpox if they present within 24 hours of the onset of the rash and if they are 20+0
weeks of gestation or beyond. Use of aciclovir before 20+0 weeks should also be considered.
Congenital varicella infection brings risks of pneumonitis, hepatic inflammation, and
encephalitis and spontaneous fetal loss.

E VZV vaccination

VZV vaccination has no role in this situation in preventing varicella-zoster infection, as the
exposure has already occurred. Where seronegative women are identified, it may be
recommended in the post-partum period.
51605

Rate this question:

Next Question

Previous Question Tag Question

Feedback End Review

Difficulty: Average

Peer Responses %

Q. Answered Flagged

Q1

Q2

Q3

Q4

Q5

Q6

Q7

Q8

Q9

 External Links

Chickenpox in Pregnancy
rcog.org.uk/globalassets/documents/guidelines/gtg13.pdf
(https://www.rcog.org.uk/globalassets/documents/guidelines/gtg13.pdf)
0:00:43/03:00:00

An 18-year-old man from a traveller family comes to the Endocrine Clinic for review. He is
chronically tired and is very frustrated that he cannot do sports like his siblings. He has
always been short and found it difficult to maintain weight. He complains of intermittent
muscle pains affecting his arms and legs and of pins and needles in his hands.
Physical examination confirms a short stature. His body mass index (BMI) is 19.5 kg/m 2.
Investigations:

Haemoglobin (Hb) 12.5 g/dl (13.5–17.5 g/dl)

White cell count (WCC) 7.8 × 10 9/l (4–11 × 10 9/l)

Platelets (PLT) 295 × 10 9/l (150–400 × 10 9/l)

Sodium (Na +) 142 mmol/l (135–145 mmol/l)

Potassium (K +) 4.4 mmol/l (3.5–5.0 mmol/l)

Creatinine (Cr) 95 µmol/l (50–120 µmol/l)

Calcium (Ca 2+) 1.95 mmol/l (2.20–2.60 mmol/l)

Phosphate (PO 4) 1.7 mmol/l (0.7–1.4 mmol/l)

Alkaline phosphatase 212 iu/l (30–130 iu/l)

Parathyroid hormone (PTH) 1.2 pmol/l (1.6–6.9 pmol/l)

Which of the following is the most likely diagnosis?

A Hyperparathyroidism

B Hypoparathyroidism

C Osteomalacia

D Paget’s disease

E Pseudohypoparathyroidism

Explanation 

B Hypoparathyroidism
The clinical scenario of short stature and chronic muscle pain and weakness fits with
congenital hypoparathyroidism, which may have been diagnosed late because this patient
has not engaged with medical services. The clinical scenario of normal renal function in the
presence of hypocalcaemia, hyperphosphataemia and low PTH levels fits well with the
diagnosis. Calcium and vitamin D supplementation is the initial intervention of choice.

A Hyperparathyroidism

Hyperparathyroidism is associated with elevated PTH and calcium levels and decreased
phosphate levels, which are not seen here.

C Osteomalacia

Osteomalacia is associated with borderline low serum calcium, elevated alkaline phosphatase,
and elevated PTH levels, rather than the low levels of PTH seen here.

D Paget’s disease

Paget’s disease is associated with increased alkaline phosphatase and normal calcium, PTH
and phosphate levels. Presentation at such a young age would also be highly unusual.

E Pseudohypoparathyroidism

This fits with the calcium, phosphate and alkaline phosphatase levels seen here, although the
PTH level is elevated because of PTH resistance.
51614

Rate this question:

Next Question

Previous Question Tag Question

Feedback End Review

Difficulty: Average

Peer Responses %
Q. Answered Flagged

Q1

Q2

Q3

Q4

Q5

Q6

Q7

Q8

Q9 
0:00:43/03:00:00

A 36-year-old woman with a long history of Crohn’s disease comes to the Gastroenterology
Clinic for review. She underwent a third bowel resection in the past few years around four
months ago. She is concerned, as her diarrhoea is continuing and she is gradually losing
weight.
Examination reveals a blood pressure of 115/82 mmHg. Her pulse is 67 bpm and regular. She
has a well-healed laparotomy scar on her abdomen. Her body mass index (BMI) is 19 kg/m 2.
Investigations:

Haemoglobin (Hb) 9.8 g/dl (11.5–15.5 g/dl)

White cell count (WCC) 5.9 × 10 9/l (4–11 × 10 9/l)

Platelets (PLT) 191 × 10 9/l (150–400 × 10 9/l)

Sodium (Na +) 143 mmol/l (135–145 mmol/l)

Potassium (K +) 3.4 mmol/l (3.5–5.0 mmol/l)

Creatinine (Cr) 90 µmol/l (50–120 µmol/l)

Albumin (Alb) 30 g/l (35–55 g/l)

Alanine transaminase (ALT) 23 iu/l (5–30 iu/l)

Alkaline phosphatase 192 iu/l (30–130 iu/l)

Calcium (Ca 2+) 2.15 mmol/l (2.2–2.6 mmol/l)

Erythrocyte sedimentation rate (ESR) 11 mm/hour (0–10 mm/hour)

C-reactive protein (CRP) 18 mg/l (0–10 mg/l)

Which of the following is the most likely cause of her diarrhoea?

A Bacterial overgrowth syndrome

B Crohn’s disease activity

C Exocrine pancreatic insufficiency

D Lymphocytic colitis

E Short bowel syndrome

Explanation 
E Short bowel syndrome

This patient is anaemic, with low albumin and borderline low calcium levels. Coupled with the
presence of ongoing diarrhoea, without raised inflammatory markers, this fits with short
bowel syndrome due to her previous surgical procedures. In the first instance, diarrhoea can
be improved with agents which slow gastrointestinal transit, and patients should receive
extensive oral supplementation. If they are unable to maintain their nutritional status, then
total parenteral nutrition may be required.

A Bacterial overgrowth syndrome

Bacterial overgrowth syndrome is more likely to present with symptoms consistent with
irritable bowel syndrome such as intermittent diarrhoea and abdominal bloating.

B Crohn’s disease activity

The normal inflammatory markers here and absence of abdominal pain both count against
active Crohn’s disease as the underlying diagnosis.

C Exocrine pancreatic insufficiency

There are no triggers for pancreatic insufficiency such as excess alcohol consumption or
recurrent pancreatitis, making exocrine pancreatic insufficiency very unlikely.

D Lymphocytic colitis

This is associated with watery diarrhoea, although pharmacological triggers, such as proton
pump inhibitors, are usually identified as a potential underlying cause.
51629

Rate this question:

Next Question

Previous Question Tag Question

Feedback End Review

Difficulty: Average

Peer Responses %
Q. Answered Flagged

Q1

Q2

Q3

Q4

Q5

Q6

Q7

Q8

Q9

0:00:43/03:00:00

A 45-year-old man presents with a VIIth nerve palsy affecting both the lower and upper
muscles of the face on the left-hand side. He has also lost taste sensation over the anterior
two-thirds of the tongue on the affected side.
Where is the most likely site of the lesion?

A Central

B After the chorda tympani

C After the nerve to the digastric

D After the nerve to the stylohyoid

E Just before it emerges from the stylomastoid foramen

Explanation 

E Just before it emerges from the stylomastoid foramen

The stylomastoid foramen is a common site for pressure on the facial nerve, as it exits from
its bony canal. This point of damage is before the branching of the chorda tympani, which
supplies taste sensation to the anterior two-thirds of the tongue, and fits with the diagnosis
here.

A Central

A central lesion is associated with preservation of facial movement of the muscles of the
brow because of double innervation from the facial nerve on the other side.

B After the chorda tympani

This does not fit with the loss of sensation over the tongue. As the chorda tympani supplies
sensation to the tongue, the facial nerve lesion must be before this point.

C After the nerve to the digastric

The nerve to the digastric supplies the digastric muscle, which stretches between the
mastoid process of the cranium to the mandible at the chin. This branches after the chorda
tympani.
D After the nerve to the stylohyoid

The nerve to the stylohyoid supplies the stylohyoid muscle which lies anterior to the digastric.
This branches after the chorda tympani.
51644

Rate this question:

Next Question

Previous Question Tag Question

Feedback End Review

Difficulty: Average

Peer Responses %

Q. Answered Flagged

Q1

Q2

Q3

Q4

Q5

Q6

Q7

Q8

Q9

0:00:43/03:00:00

A 68-year-old man who works as a part-time taxi driver comes to the clinic for review of his
glycaemic control. His HbA1c is 68 mmol/mol, and current medications include metformin
and sitagliptin. He has suffered an inferior myocardial infarction some ten years ago and
takes a range of medications, including bisoprolol and ramipril, for management of mild heart
failure.
His blood pressure is 119/82 mmHg, and pulse 60 bpm and regular. There is mild pitting
oedema affecting both ankles. His body mass index (BMI) is 34 kg/m 2.
Investigations:

Haemoglobin (Hb) 13.0 g/dl (13.5–17.5 g/dl)

White cell count (WCC) 7.0 × 10 9/l (4–11 × 10 9/l)

Platelets (PLT) 203 × 10 9/l (150–400 × 10 9/l)

Sodium (Na +) 143 mmol/l (135–145 mmol/l)

Potassium (K +) 4.5 mmol/l (3.5–5.0 mmol/l)

Creatinine (Cr) 99 µmol/l (50–120 µmol/l)

Which of the following is the most appropriate next step for glucose control?

A Acarbose

B Empagliflozin

C Gliclazide

D Insulin glargine

E Pioglitazone

Explanation 

B Empagliflozin

This patient needs improved glycaemic control without increasing the risk of hypoglycaemia
or worsening cardiac failure. Empagliflozin, a sodium–glucose co-transporter 2 (SGLT-2)
inhibitor, will improve HbA1c, have positive effects on his weight and reduce the risk of
cardiovascular death and admission to hospital with symptomatic cardiac failure. As such, it
is the most appropriate choice here.
A Acarbose

Although acarbose does improve glycaemic control without worsening hypoglycaemia and
does reduce the cardiovascular risk, its adverse event profile in terms of lower
gastrointestinal adverse events is potentially unsuited to this patient’s job as a taxi driver.

C Gliclazide

Gliclazide does improve glucose control in the short term, although this is at the expense of
hypoglycaemia and weight gain. It has not been shown to improve outcomes from ischaemic
cardiovascular disease. In this case, the hypoglycaemic effects would make it inappropriate,
given the patient’s profession.

D Insulin glargine

Insulin glargine is a suboptimal choice here because it will improve glycaemic control, but at
a significant risk of worsening hypoglycaemia and weight gain. In addition, it has not been
proven to impact on cardiac outcomes.

E Pioglitazone

Pioglitazone does improve glycaemic control without significantly increasing the risk of
hypoglycaemia, although this is at the expense of weight gain. It is also contraindicated in
patients who have cardiac failure.
51617

Rate this question:

Next Question

Previous Question Tag Question

Feedback End Review

Difficulty: Average

Peer Responses %
Q. Answered Flagged

Q1

Q2

Q3

Q4

Q5

Q6

Q7

Q8

Q9
0:00:43/03:00:00

A 69-year-old woman is admitted to the Emergency Department, suffering from diarrhoea


and left iliac fossa pain. She tells you that the diarrhoea has been mixed with blood. There
have been two previous admissions with diarrhoea over the past year. She has not lost
weight and admits to constipation over the past few years. She takes ramipril and diltiazem
for the treatment of ischaemic heart disease. Her pulse is regular in the Emergency
Department, and her electrocardiogram (ECG) reveals no evidence of atrial fibrillation.
Examination reveals an ill-defined mass and pain in the left iliac fossa.
Investigations:

Haemoglobin (Hb) 12.3 g/dl (11.5–15.5 g/dl)

White cell count (WCC) 13.9 × 10 9/l (4–11 × 10 9/l)

Platelets (PLT) 284 × 10 9/l (150–400 × 10 9/l)

C-reactive protein (CRP) 271 mg/l (0–10 mg/l)

Sodium (Na +) 142 mmol/l (135–145 mmol/l)

Potassium (K +) 4.9 mmol/l (3.5–5.0 mmol/l)

Creatinine (Cr) 129 µmol/l (50–120 µmol/l)

No evidence of perforation;
Abdominal X-ray
colon loaded with faeces

Which of the following is the most likely diagnosis?

A Crohn’s abscess

B Diverticulitis

C Ischaemic colitis

D Ruptured ovarian cyst

E Sigmoid colon carcinoma

Explanation 

B Diverticulitis
The clinical scenario here with recurrent diarrhoea and left iliac fossa pain (and, on this
occasion, a possible diverticular abscess) fits with diverticulitis as the most likely cause of her
symptoms. Patients with constipation are at increased risk of diverticular disease, and
diltiazem is recognised as a cause of constipation. It is possible that the diarrhoea may
represent overflow against a background of constipation or may be occurring as a result of
local colonic inflammation. Treatment following the acute event should include increasing
fluid intake and soluble fibre.

A Crohn’s abscess

A Crohn’s abscess in the absence of symptoms before this year is very unlikely. The largest
peak in Crohn’s diagnosis occurs between the ages of 15 and 30 years, and small
bowel/right-sided colonic disease is more likely than the left-sided disease described here.

C Ischaemic colitis

Ischaemic colitis is most likely to be seen at the rectosigmoid junction and splenic flexure.
The ill-defined mass here makes ischaemic colitis less likely to be the underlying diagnosis, as
does the normal ECG. A CT would be required to rule this out as a diagnosis.

D Ruptured ovarian cyst

This is less likely, given the history of recurrent admissions with diarrhoea, which is much
more in keeping with a diagnosis of diverticular disease. For a ruptured ovarian cyst, acute-
onset iliac fossa pain, in the absence of marked gastrointestinal symptoms, would be more
usual.

E Sigmoid colon carcinoma

Although this could present with a left iliac fossa mass and a change in bowel habit, other
features which might be expected, including weight loss, chronic pain or subacute
obstruction, are absent here.
51626

Rate this question:

Next Question

Previous Question Tag Question

Feedback End Review

Difficulty: Average

Peer Responses %
Q. Answered Flagged

Q1

Q2

Q3

Q4

Q5

Q6

Q7

Q8

Q9

0:00:43/03:00:00

A 43-year-old woman is referred to the Endocrine Clinic with hypertension, impaired glucose
tolerance and a change in her facial appearance on photographs, including macroglossia and
increased prominence of her lower jaw.
Examination reveals a blood pressure of 152/92 mmHg. Her pulse is 77 bpm and regular. Her
body mass index (BMI) is 23 kg/m 2. Insulin-like growth factor 1 (IGF-1) is significantly
elevated.
Which of the following is the most appropriate confirmatory test for acromegaly?

A Glucose tolerance test with growth hormone measurement

B Insulin tolerance test with growth hormone measurement

C Pituitary magnetic resonance imaging (MRI)

D Random growth hormone measurement

E Serum insulin-like growth factor binding protein 3 (IGFBP3)

Explanation 

A Glucose tolerance test with growth hormone measurement

Although IGF-1 level is significantly raised, it is important to confirm that growth hormone is
not suppressed during the oral glucose tolerance test. If growth hormone fails to suppress
below 1 mg/l, then the diagnosis of acromegaly is confirmed, and a pituitary MRI is the logical
next step.

B Insulin tolerance test with growth hormone measurement

Insulin tolerance test is a possible investigation to confirm growth hormone deficiency. It


carries a significant risk, with glucose levels needing to reach the hypoglycaemic range to
confirm that growth hormone is not stimulated.

C Pituitary magnetic resonance imaging (MRI)

Pituitary MRI is needed to confirm the diagnosis of a pituitary adenoma, although because of
the risk of identifying ‘incidentalomas’, it should not be performed until a biochemical
diagnosis is confirmed.

D Random growth hormone measurement


Growth hormone production is cyclical and transient; measuring random growth hormone
levels therefore does not contribute to a diagnosis of acromegaly.

E Serum insulin-like growth factor binding protein 3 (IGFBP3)

Although IGFBP3 is the main binding protein for IGF-1, it does not significantly contribute to
confirming a diagnosis of the condition.
51618

Rate this question:

Next Question

Previous Question Tag Question

Feedback End Review

Difficulty: Average

Peer Responses %

Q. Answered Flagged

Q1

Q2

Q3

Q4

Q5

Q6

Q7

Q8

Q9

0:00:43/03:00:00

A 50-year-old man, who is a known alcoholic with end-stage cirrhosis, is brought to the
Emergency Department, having been found lying on his sofa with confusion and pyrexia.
According to the paramedics, he had an empty bottle of gin on the table next to him.
Examination reveals a temperature of 38.3 °C. His blood pressure is 95/60 mmHg, and his
pulse is 95 bpm and regular. He has obvious ascites on abdominal examination and moans in
pain in response to palpation.
Investigations:

Haemoglobin (Hb) 9.8 g/dl (13.5–17.5 g/dl)

White cell count (WCC) 13.2 × 10 9/l (4–11 × 10 9/l)

Platelets (PLT) 87 × 10 9/l (150–400 × 10 9/l)

C-reactive protein (CRP) 185 mg/l (0–10 mg/l)

Sodium (Na +) 137 mmol/l (135–145 mmol/l)

Potassium (K +) 5.0 mmol/l (3.5–5.0 mmol/l)

Creatinine (Cr) 95 µmol/l (50–120 µmol/l)

Which of the following is the most useful investigation?

A Ascitic tap

B Blood cultures

C Computed tomography (CT) head

D Lumbar puncture

E Upper gastrointestinal (GI) endoscopy

Explanation 

A Ascitic tap

This patient’s deterioration is suggestive of spontaneous bacterial peritonitis (SBP). A cell


count of > 250 polymorphs per mm 3 is highly suggestive of the diagnosis. Other useful
calculations include the serum–ascites albumin gradient. If the difference between the two is
> 11 g/l, this is suggestive of portal hypertension and supportive of an SBP diagnosis.
Quinolones and third-generation cephalosporins are reasonable options for treatment. Both
Escherichia coli and Klebsiella are commonly recognised causes of SBP in patients with end-
stage cirrhosis, which are likely to be sensitive to these antibiotic options.

B Blood cultures

Blood cultures are likely to have a lower yield, with respect to a bacterial diagnosis, in
comparison to ascitic fluid.

C Computed tomography (CT) head

A subdural haematoma is a possible cause of confusion in this patient, although the fever and
abdominal pain are more in keeping with a diagnosis of SBP.

D Lumbar puncture

Patients with end-stage liver disease are at greater risk of bacterial meningitis, although here
the abdominal pain is a stronger pointer towards SBP as the underlying diagnosis.

E Upper gastrointestinal (GI) endoscopy

Hepatic decompensation can occur due to an upper GI haemorrhage related to oesophageal


varices. If the ascitic tap is unremarkable, an upper GI endoscopy should be considered to
rule out bleeding varices.
51625

Rate this question:

Next Question

Previous Question Tag Question

Feedback End Review

Difficulty: Average

Peer Responses %
Q. Answered Flagged

Q1

Q2

Q3

Q4

Q5

Q6

Q7

Q8

Q9
0:00:43/03:00:00

A 54-year-old man attends the clinic some two months after Helicobacter pylori eradication
therapy for peptic ulcer disease. He has no residual symptoms.
Which of the following is the most appropriate way to confirm eradication of H. pylori?

A Carbon-13 breath testing

B H. pylori serology

C Hydrogen breath testing

D Re-endoscopy and biopsy

E Stool antigen testing

Explanation 

A Carbon-13 breath testing

Carbon-13 breath testing is the only reliable re-test method which is endorsed by the National
Institute for Health and Care Excellence (NICE) guidelines on management of ulcer disease.
The optimised initial therapy for H. pylori should include a seven-day course of a proton
pump inhibitor plus amoxicillin 1 g and either clarithromycin 500 mg or metronidazole 400
mg – all three given twice a day.

B H. pylori serology

Serological testing for H. pylori is unreliable as an assessment of H. pylori clearance.


Therefore, it is not recommended by NICE guidelines.

C Hydrogen breath testing

Hydrogen breath testing is used to diagnose bacterial overgrowth, which is more prevalent in
patients who have undergone gastrointestinal (GI) surgery or suffer from conditions which
affect GI motility such as systemic sclerosis.

D Re-endoscopy and biopsy

Re-endoscopy is not required. Carbon-13 breath testing is accurate in assessing clearance of


H. pylori.
E Stool antigen testing

Although commercial stool antigen tests are available for assessment of H. pylori status, they
are not recommended outside the research context by current NICE guidelines.
51628

Rate this question:

End Session

Previous Question Tag Question

Feedback End Review

Difficulty: Average

Peer Responses %

Q. Answered Flagged

Q1

Q2

Q3

Q4

Q5

Q6

Q7

Q8

Q9

 External Links

National Institute for Health and Care Excellence - Helicobacter pylori testing and eradication in adults
pathways.nice.org.uk/pathways/dyspepsia-and-gastro-oesophageal-reflux-disease/helicobacter-pylori-testing-and-e…
(https://pathways.nice.org.uk/pathways/dyspepsia-and-gastro-oesophageal-reflux-
disease/helicobacter-pylori-testing-and-eradication-in-adults#content=view-
node%3Anodes-h-pylori-testing)

You might also like